Está en la página 1de 44

CICLO DE REVISION EN MEDICINA

Examen Simulacro
Tema A y B

1. Hombre de 48 aos que acude a Urgencias por dolor


abdominal y vmitos. Los datos analticos iniciales son:
GOT: 80 U/I. GTP 54 U/I. Leucocitos 21.800/mm3.
Amilasa 4.500 U/I. La TAC abdominal demuestra
coleccin peripancretica. Se inicia tratamiento con
fluidoterapia y analgsicos. Seale cul de los
siguientes frmacos aadira al tratamiento, en primer
lugar, para mejorar el pronstico de este caso:
A) Metilprednisolona.
B) Antiinflamatorios no esteroideos.
C) Omeprazol.
D) Imipenem.
E) Inhibidores de la secrecin pancretica.
2. Cul es el principal factor responsable de la no
cicatrizacin de una lcera pptica?:
A) Estrs.
B) Infeccin por Helicobacter pylori.
C) Consumo de alcohol.
D) No abandono del hbito tabquico.
E) Determinados hbitos dietticos.

Academia de Preparacin Mdica

Estudios M&C SAC.


Jr. Manuel Corpancho Nro. 313
Urb. Santa Beatriz, Lima.

www.estudiosmyc.com

Sbado, 4 de Agosto del 2012

3. Cul de las siguientes recomendaciones es FALSA


en relacin con el tratamiento de la peritonitis bacteriana
espontnea en un paciente cirrtico?:
A) El diagnstico se basa en la existencia de ms de
250 PMN/mm3 en el lquido asctico.
B) El tratamiento de eleccin lo constituyen las
cefalosporinas de tercera generacin.
C) La administracin de albmina intravenosa previene
el desarrollo de insuficiencia renal.
D) El tratamiento antibitico debe mantenerse durante
15 das.
E) Una vez resuelta la infeccin, debe iniciarse
tratamiento profilctico con norfloxacino.
4. Paciente de 72 aos, que como nico tratamiento
toma antidiabticos orales, presenta anemia ferropnica
crnica, con hemorragias ocultas positivas. Cual es el
mtodo diagnstico ms indicado para localizar la lesin
sangrante?:
A) Trnsito gastroduodenal.
B) Gammagrafa con hemates marcados.
C) Trnsito intestinal.
D) Colonoscopia total.
E) Panendoscopia oral.
5. Cul de las siguientes definiciones corresponde con
mayor precisin al concepto de Metaanlisis?:
A) Es una revisin narrativa en la que la bibliografa se
busca de manera no estructurada.
B) Es una revisin narrativa en la que la bibliografa se
busca de manera estructurada.
C) Es una revisin narrativa en la que se presentan
tablas estructuradas de los resultados de los distintos
estudios incluidos.
D) Es una revisin en la que se combinan
estadsticamente los resultados de los estudios
incluidos.

Examen Simulacro :: Ciclo de Revisin en Medicina :: Sbado 4 de Agosto del 2012 :: www.estudiosmyc.com
E) Es una revisin en las que se presentan tablas
estructuradas de los resultados de los distintos estudios
incluidos.
6. Le encargan el diseo de un ensayo clnico en el que
es muy importante que un factor pronstico se distribuya
por igual en los dos grupos de tratamiento. El mtodo de
aleatorizacin que usted utilizara es:
A) Aleatorizacin simple.
B) Aleatorizacin por bloques.
C) Aleatorizacin estratificada.
D) Aleatorizacin centralizada.
E) Aleatorizacin ciega (ocultacin de la secuencia de
aleatorizacin).
7. En una revista biomdica se publica un estudio, en el
que los autores notifican el resultado en coste/aos de
vida ganados. De qu tipo de anlisis de evaluacin
econmica se trata?:
A) Coste de la enfermedad.
B) Coste-efectividad.
C) Coste-beneficio.
D) Coste-consecuencia.
E) Coste-utilidad.
8. La mejor forma de verificar una hiptesis en
epidemiologa es a travs de:
A) Un estudio descriptivo.
B) Un estudio experimental.
C) Un estudio caso-control.
D) Un estudio de cohortes.
E) Un estudio transversal.
9. Todas estas situaciones, EXCEPTO una, aumentan la
probabilidad de detectar un cncer colorrectal, selela:
A) Enfermedad inflamatoria intestinal de larga evolucin.
B) Endocarditis por Streptococcus bovis.
C) Tabaquismo de ms de 35 aos de duracin.
D) Ureterosigmoidostoma hace 20 aos, para corregir
una malformacin vesical.
E) Ingesta crnica de aspirina o antiinflamatorios no
esteroideos.
10. Cul de las siguientes caractersticas NO es propia
de un ensayo clnico controlado?:
A) Intervencin experimental.
B) Tratamiento asignado segn el criterio del
investigador.
C) Participacin voluntaria del sujeto.
D) Asignacin aleatoria a las distintas posibilidades de
tratamiento en estudio.
E) Enmascaramiento de la medicacin.
11. La vacuna antineumoccica est recomendada para
todas, MENOS una de las siguientes situaciones
clnicas, selela:
A) Alcoholismo crnico.
B) Contactos familiares de un paciente con neumona
neumoccica

C) Fstula crnica de lquido cefalorraqudeo tras un


traumatismo craneal.
D) Infeccin VIH avanzada.
E) Insuficiencia cardaca crnica.
12. Cul de la siguientes vacunas no se incluye en las
recomendaciones
actuales
de
un
paciente
esplenectomizado?:
A) Vacuna neumoccica.
B) Vacuna meningoccica.
C) Vacuna frente a Haemophilus influenzae tipo B.
D) Vacuna frente al virus de la gripe.
E) Vacuna frente al virus hepatitis A.
13. En qu patologa pensara en primer lugar en un
paciente de 65 aos, que presenta disminucin lenta,
progresiva e indolora de su agudeza visual sin signos de
inflamacin ocular?:
A) Error de refraccin.
B) Distrofia corneal.
C) Papilitis.
D) Glaucoma agudo.
E) Degeneracin macular senil.
14.
Un
paciente
diabtico
tratado
mediante
fotocoagulacin focal con lser de Argn tres aos
antes, presenta una prdida brusca e importante de
visin, sin dolor ni alteraciones en la superficie ocular.
La causa ms probable de esta disminucin de agudeza
visual es:
A) Hemorragia vtrea.
B) Edema corneal.
C) Glaucoma crnico simple.
D) Catarata nuclear.
E) Atrofia ptica.
15. Una de los siguientes signos o sntomas, NO es
habitual en una uvetis anterior aguda:
A) Midriasis.
B) Dolor.
C) Inyeccin ciliar.
D) Sinequias posteriores.
E) Fotofobia.
16. Cul es el tratamiento de la obstruccin intestinal
no quirrgica en la fase terminal del cncer?:
A)
Sonda
nasogstrica,
aspiracin
continua,
sueroterapia intravenosa.
B) Administracin de vitamina grupo B, laxante y enema
de limpieza.
C) Alimentacin parenteral total y esteroides i.v.
D) Morfina, buscapina y haloperidol por va subcutnea.
E) Dieta absoluta y sueros por va subcutnea.
17. Cul de los siguientes opioides NO es adecuado
para el tratamiento del dolor crnico de etiologa
cancerosa?:
A) Metadona.
B) Meperidina.
C) Morfina.

Examen Simulacro :: Ciclo de Revisin en Medicina :: Sbado 4 de Agosto del 2012 :: www.estudiosmyc.com
D) Fentanilo.
E) Tramadol.
18. La localizacin ideal para hacer una traqueotoma
es:
A) La membrana crico-tiroidea.
B) La membrana mio-tiroidea.
C) Primer anillo traqueal.
D) Segundo o tercer anillo traqueal.
E) Cuarto o quinto anillo traqueal.
19. Cul de estas afirmaciones es FALSA en relacin
con la otitis media secretoria?:
A) Los nios con paladar hendido son ms susceptibles
a padecerla.
B) En casos unilaterales en adultos, es obligada la
inspeccin del cavum.
C) Su perodo de mayor incidencia en climas templados,
es de junio a septiembre, coincidiendo con la poca de
baos.
D) El 66% de los cultivos de las muestras obtenidas por
miringocentesis presentan bacterias.
E) La otoscopia puede mostrar niveles lquidos o
burbujas de aire tras el tmpano ntegro.
20. Una mujer de 68 aos, sin aparentes factores de
riesgo cardiovascular, ingresa en la Unidad Coronaria
del Hospital por un cuadro agudo de cardiopata
isqumica. En la analtica realizada a su llegada se
objetiva una anemia (Hb 8gr/dl) previamente no
conocida. En este caso, la actitud ms adecuada con
respecto a la anemia es:
A) Actitud expectante, ya que slo se debe transfundir la
anemia sintomtica.
B) Transfundir hemates.
C) Transfundir sangre total.
D) Instaurar tratamiento con eritropoyetina.
E) Administrar hierro intravenoso.
21. Los cuerpos de Howell-Jolly, son inclusiones
eritrocitarias de fragmentos nucleares y se observan en:
A) Asplenia.
B) Mielofibrosis.
C) Leucemia linftica crnica.
D) Dficit de G6PD.
E) Intoxicacin por plomo.
22. Cul de los siguientes hallazgos es el ms
caracterstico de la leucemia mieloide crnica?:
A) El cromosoma Filadelfia.
B) El esplenomegalia palpable.
C) La disminucin de la fosfatasa alcalina granulocitaria.
D) El aumento del cido rico srico.
E) El reordenamiento del gen bcr/abl.
23. Joven de 23 aos, con historia de conducta sexual
de riesgo, consulta por fiebre, malestar general y dolor
en glande. La exploracin fsica muestra mltiples
tatuajes, "piercings" y la presencia de tres lesiones

vesiculosas en glande y adenopatas inguinales


bilaterales. Cul es el diagnstico ms probable?:
A) Herpes genital.
B) Secundarismo lutico.
C) Primoinfeccin VIH.
D) Infeccin por citomegalovirus.
E) Condilomas acuminados.
24. En cuanto a la neumona por Neumococo en
paciente infectados por el virus de la inmunodeficiencia
humana (VIH) es FALSO que:
A) Es una infeccin frecuentemente bacterimica.
B) Tiene mayor incidencia que en la poblacin general.
C) Junto a Haemophilus influenzae es la causa ms
comn de neumona en pacientes con SIDA.
D) Se recomienda vacuna neumoccica en aquellos
pacientes con CD4 < 100 cel/L.
E) Esta neumona puede ser vista en pacientes con
sistema inmune relativamente intacto.
25. La tuberculosis asociada a la infeccin por VIH se
caracteriza por:
A) Presentacin subclnica de la enfermedad.
B) Aparicin caracterstica en los estadios de
inmunodepresin ms severa (>50 CD4/mm3).
C) Elevada frecuencia de afectacin extrapulmonar y
diseminada.
D) Escaso rendimiento de los mtodos microbiolgicos
de diagnstico.
E) Mala respuesta al tratamiento antituberculoso.
26. NO es propio del shock txico estafiloccico:
A) Fiebre elevada.
B) Lesiones cutneas.
C) Metstasis spticas.
D) Fracaso renal.
E) Rabdomilisis.
27. Cul de las siguientes permite diferenciar la
pielonefritis aguda de la cistitis aguda?:
A) Leucocituria.
B) Hematuria.
C) Bacteriuria.
D) Antecedentes de infeccin urinaria.
E) Fiebre de ms de 38.5C.
28. Un paciente de 22 aos, que acude para evaluacin
de una lesin genital ulcerada, presenta un VDRL
positivo a ttulo de 8 diluciones con un FTA-Abs
negativo. La interpretacin ms adecuada de estos
resultados es:
A) Falso positivo de las pruebas no treponmicas.
B) Falso negativo de las pruebas treponmicas.
C) Sfilis curada (pendiente la positivizacin del FTAAbs).
D) Sfilis curada (pendiente la negativizacin del VDRL).
E) Sfilis de larga evolucin.

Examen Simulacro :: Ciclo de Revisin en Medicina :: Sbado 4 de Agosto del 2012 :: www.estudiosmyc.com
29. Seale cul de las siguientes asociaciones de
helmintos y su clnica caracterstica es INCORRECTA:
A) Ascaris lumbricoides - Sndrome de Leffler.
B) Giardia lamblia Mala absorcin intestinal.
C) Ancylostoma duodenale - Anemia megaloblstica.
D)
Strongyloides
stercolaris
Sndrome
de
hiperinfestacin en inmunodeprimidos.
E) Taenia solium - Convulsiones generalizadas.
30. Paciente de 30 aos, seropositivo VIH conocido
desde 5 aos antes, con antecedentes de neumona por
P. jivovecci, que consulta por cefalea desde 10 das
antes. La exploracin fsica muestra como datos ms
relevantes mnima rigidez de nuca y temperatura de
37,5C, fondo de ojo normal, TAC: ligera atrofia cortical.
La puncin lumbar da salida a lquido claro con 40
clulas mononucleares, protenas: 90 mgrs%, glucosa:
30 mg% (glucemia: 90 mg%). Sealar, de entre las
siguientes, la causa ms probable:
A) Herpesvirus tipo 8.
B) Listeria.
C) Criptococo.
D) CMV.
E) VIH.
31. En relacin con Streptococcus pyogenes y la
faringoamigdalitis, cul de las siguientes afirmaciones
NO es correcta?:
A) En tratamiento de la faringoamigdalitis estreptoccica
se efecta con una sola inyeccin i.m. de 1,200,000 UU
de penicilina benzatina.
B) El tratamiento antibitico de la faringoamigdalitis
estreptoccica se efecta con 250,000 UU/6h oral de
penicilina V durante 10 das.
C) El tratamiento antibitico de la faringoamigdalitis
estreptoccica se efecta con una sola inyeccin i.m. de
1,200,000 UU de penicilina procana.
D) El tratamiento antibitico de la faringoamigdalitis
estreptoccica se efecta con amoxicilina oral 500
mg/8h durante 10 das.
E) El tratamiento antibitico recomendado de la
faringoamigdalitis estreptoccica en los casos de alergia
a la penicilina es un macrlido oral durante 10 das.
32. A lo largo de los ltimos 10 aos se ha producido un
cambio muy importante en la etiologa de la Endocarditis
Infecciosa del adulto. El microorganismo ms frecuente
en la actualidad, es:
A) Microorganismos del grupo HACEK.
B) Staphylococcus aureus.
C) Estafilococos coagulasa negativos.
D) Bacilos gram negativos.
E) Streptococcus viridans.
33. Un hombre de 74 aos con un infarto agudo de
miocardio es tratado con estreptoquinasa. Seis horas
despus desarrolla un cuadro de hipotensin arterial
severa y obnubilacin. Cul de las siguientes
complicaciones es MENOS probable que sea la causa?:
A) Infarto de ventrculo derecho.

B) Tromboembolismo pulmonar.
C) Rotura del msculo papilar.
D) Rotura de la pared libre ventricular.
E) Hemorragia cerebral.
34. En el tratamiento de la hipertensin arterial, la
ventaja de los bloqueadores de los receptores de la
Angiotensina II con respecto a los inhibidores del
enzima conversor de la Angiotensina es que:
A) Son ms potentes.
B) Producen menos tos.
C) No producen hiperpotasemia.
D) Se puede dar en embarazadas.
E) Se pueden dar en sujetos con estenosis de la arteria
renal bilateral.

35. El tratamiento ms eficaz para prevenir recurrencias


en el aleteo o flutter auricular comn es:
A) Buen control de la hipertensin arterial que con
frecuencia padecen estos enfermos.
B) Digoxina asociada a un frmaco que disminuya la
conduccin en el nodo A-V (anticlcicos o
betabloqueantes).
C) Ablacin con catter y radiofrecuencia del istmo cavo
tricspide.
D) Insercin de un marcapasos con capacidad
antitaquicardia.
E) Amiodarona.
36. Un paciente obnubilado con una presin arterial de
80/40 mmHg, un gasto cardaco de 3 l/min, una presin
de enclavamiento pulmonar de 14 mmHg y una presin
auricular derecha de 14 mmHg, puede estar sufriendo
cualquiera de las siguientes situaciones patolgicas con
la excepcin de una:
A) Taponamiento cardaco.
B) Deshidratacin.
C) Infarto de ventrculo derecho.
D) Tromboembolismo pulmonar.
E) Constriccin pericrdica.
37. La campana del fonendoscopio es ms adecuada
que la membrana para:
A) Distinguir un soplo diastlico de uno sistlico.
B) Or mejor el chasquido de apertura en pacientes con
estenosis mitral.
C) Valorar la presencia de un tercer y/o cuarto tono.
D) Detectar el click mesosistlico del prolapso mitral.
E) Auscultar a los nios pequeos y bebs, en los que la
membrana no capta bien la tonalidad de los ruidos
cardacos.
38. Cul de las siguientes respuestas sobre el Delirium
Tremens es la correcta?:
A) Se trata del estado peculiar de intoxicacin producido
por el consumo de alcohol.
B) Se caracteriza por la aparicin de conductas
impulsivas tras el consumo de una pequea cantidad de
alcohol.

Examen Simulacro :: Ciclo de Revisin en Medicina :: Sbado 4 de Agosto del 2012 :: www.estudiosmyc.com
C) Es un cuadro clnico con elevado ndice de
complicaciones
psiquitricas
y
con
pocas
complicaciones orgnicas.
D) Aparece con mayor frecuencia entre los hombres
alcohlicos a partir de los 50 aos.
E) Su tratamiento de eleccin son las benzodiacepinas.
39. Cules son los trastornos perceptivos, de entre los
que se enumeran, ms caractersticos de las
esquizofrenias?:
A) Ilusiones hipnaggicas.
B) Alucinaciones visuales zoomrficas.
C) Alucinaciones o pseudoalucinaciones auditivas.
D) Paraeidolias.
E) Alucinosis auditivas.
40. Una paciente de 24 aos acude a la Urgencia
llevada por un familiar por vmitos recidivantes. Parece
bulimia nerviosa. Cul de las pruebas de laboratorio es
ms til para evaluar la gravedad de los vmitos?:
A) Nivel de Hemoglobina.
B) Nivel plasmtico de Amilasa.
C) Nivel srico de Socio.
D) Nivel plasmtico de Calcio.
E) Nivel plasmtico de Creatinina.
41. Acude a urgencias un paciente de 37 aos con gran
postracin, fiebre elevada (38,5C) e intensa cefalea.
Presenta en miembros inferiores, tronco y raz de
miembros superiores un exantema tenue eritematoviolceo, maculoso, escasamente confluente, con
afectacin de palmas y plantas. Al interrogar a
familiares, reconocen haber estado hace siete das en
una excursin. Reexplorado el paciente, en cara
posterior de la piera derecha aparece una lesin
necrtica-costrosa, de 0,5 cm. de dimetro, rodeada por
un halo violceo, edematoso de 0,3-0,4 cm. El
tratamiento ms indicado sera:
A) Doxiclina v.o. (200 mgr. cada 12 horas un da 100
mgr. cada 12 horas por 5 das).
B) Cloxacilina 1 gr. i.v. cada 6 horas durante 10 das.
C) Ceftriaxona 1 gr. i.v. cada 12 horas durante 10 das.
D) TMP-SMX 500 mgr. i.v. cada 8 horas por 10 das.
E) Eritromicina 250 mgr. i.v. cada 6 horas por 7 das.
42. Un paciente de 27 aos ex-ADVP desde hace tres
aos, con muguet oral y antecedentes de neumona por
neumocistis carinii, presenta en el dedo ndice, en
superficie dorsal de la 2. falange, una lesin nica,
lcero necrtica, con crecimiento serpinginoso de
aproximadamente 4-5 cms. de dimetro, bordes
geogrficos y algunas reas costrosas, hemorrgicas.
Es extremadamente dolorosa. Su 1. sospecha clnica
es:
A) Linfoma B cutneo.
B) Ulcera secundaria al tratamiento.
C) Infeccin por herpes simple.
D) Chancro lutico.
E) Picadura sobreinfectada.

43. Un paciente de 67 aos, comienza a presentar


lesiones eritematoedematosas en brazos y abdomen.
Ocasionalmente se observan ampollas salpicando el
rea afectada. No hay afectacin de mucosas. Conserva
el estado general. Se realiz biopsia de piel donde se
observ las imagen de una vescula subepidrmica con
eosinfilos. En inmunofluorescencia directa se observ
una imagen de depsito lineal en UDE de IgG. Tras usar
la tcnica de separacin con sal, la banda de depsito
aparece tanto en el suelo, como en el techo de la
ampolla. El diagnstico ms probable ser:
A) Enfermedad de During-Brocq.
B) Fogo Selvagen.
C) Enfermedad IgA lineal.
D) Epidermlisis ampollosa adquirida.
E) Penfigoide ampolloso.
44. Un paciente de 44 presenta lesiones ppuloerosivas muy pruriginosas en glteo y rodillas, que
comenzaron a salir hace meses, cursando en brotes que
curan espontneamente en una semana. Se ha
realizado biopsia de piel observndose ampolla
subepidrmica en tincin de hematoxilina-eosina y un
depsito de IgA linea en UDE. La biopsia de
vellosidades intestinales en normal. Los anticuerpos
antigliadina IgA son negativos. Los anticuerpos
antigliadina IgG y los anticuerpos antiendomisio son
positivos. Presenta adems anticuerpos antitiroideos. El
tratamiento adecuado ser:
A) Corticoides orales porque es una enfermedad IgA
lineal.
B) Corticoides orales porque es una Dermatitis
Herpetiforme con biopsia intestinal negativa.
C) Sulfonas ms dieta sin gluten, porque es una
Dermatitis Herpetiforme.
D) Sulfonas sin dieta, porque es una Dermtitis
Herpetiforme con biopsia intestinal negativa.
E) Sulfonas ms inmunosupresores.
45. Un varn de 6 aos presenta de manera brusca un
brote de lesiones eritematoescamosas redondeadas, de
pqueo tamao, distribudas por todo el tegumento. La
semana anterior acudi a Urgencias por un cuadro
catarral, con T= 37,5C y amigdalitis. En el cultivo
farngeo se aisl un estreptococo. La patologa cutnea
ms probable corresponde a:
A) Pitiriasis rosada.
B) Eczema numular.
C) Psoriasis gutata.
D) Herpes circinado.
E) Vasculitis sptica.
46. Un joven de 26 aos, con antecedentes familiares
de psoriasis, presenta un brote de lesiones en placas
eritematosas, con centro amarillento y halo perifrico
descamativo, ovaladas, bien delimitadas, en cara
anterior y posterior del tronco. La 1. lesin haba
aparecido haca 1 semana, era de mayor tamao y se
localizaba en el tercio superior de la espalda. El

Examen Simulacro :: Ciclo de Revisin en Medicina :: Sbado 4 de Agosto del 2012 :: www.estudiosmyc.com
paciente conserv un buen estado general en todo
momento. Probablemente el cuadro corresponde a:
A) Psoriasis en pequeas placas.
B) Psoriasis tipo Von Zumbusch.
C) Pitiriasis rosada de Gibert.
D) Rosola sifiltica.
E) Eczema diseminado.
47. Un varn de 16 aos comienza a presentar
vesculas de contenido claro agrupadas, de localizacin
peribucal, que a lo largo de 1 semana se van rompiendo
formndose erosiones y costras. Transcurrido este
perodo, en el dorso de las manos aparecen 2 lesiones
ampollosas con vesculas dispuestas perifricamente y
alguna ppula eritematosa con centro ms oscuro. Se
trata de:
A) Inicio de una varicela.
B) Aparicin de lesiones en manos por contagio directo
desde la zona peribucal.
C) Imptigo estafiloccico.
D) Eritema multiforme minor.
E) Sndrome de Stevens-Johnson.
48. Una mujer de 65 aos viene presentando desde
hace 2 aos brotes de lesiones nodulares < de 2 cm.,
localizadas de forma bilateral en MMII, acompaados de
livedo reticularis, febrcula y artralgias. La histologa
corresponde a una vasculitis leucocitoclstica.
Probablemente se trata de:
A) Sndrome de Sweet.
B) Eritema elevatum diutinum.
C) Panarteritis nodosa cutnea.
D) Vasculitis urticarial.
E) Enfermedad de Kawasaki.
49. Un paciente de 63 aos de edad presenta una
eritrodermia de varios meses de evolucin con
adenopatas generalizadas y ms de 10% de clulas
con ncleo cerebriforme en sangre perifrica. Tiene
adems intenso prurito y edemas pretibiales. Su
diagnstico sera:
A) Eczema seborreico.
B) Exantema medicamentoso crnico.
C) Eritrodermia psoriasica.
D) Sndrome de Sezary.
E) Parapsoriasis en grandes placas.
50. Un nio sin alteraciones cutneas al nacimiento
comienza a presentar hacia los 3 meses de edad
escamas grandes y negruzcas en tronco y
extremidades, afectando pliegues axilares y poplteos.
Tiene un hermano mayor igualmente afecto. Es
probable que se trate de:
A) Una ictiosis ligada a X.
B) Una ictiosis laminar.
C) Una ictiosis vulgar.
D) Enfermedad de Darier.
E) Ninguna de las anteriores.
51. Una mujer acude a nuestra consulta poque tras
haber estado esa maana expuesta al sol presenta una

quemadura solar exagerada localizada en cara


(respetando regin peiorbitaria, retroauricular y
submentoniana), dorso de manos y en ambas piernas
hasta la altura de las rodillas. Estaba en tratamiento con
un diurtico tiacidico.
A) Se trata de una reaccin fototxica.
B) Es una reaccin fotoalrgica por mecanismo tipo IV.
C) Debe eliminarse el agente causante, poner
tratamiento sintomtico y evitar la radiacin lumnica
hasta que remita el cuadro.
D) Es una erupcin polimorfa lumnica.
E) Son correctas A y C.
52. Una joven de 16 aos de edad presenta varias
mculas de color blanco lechoso de varios centmetros
de dimetro, de distribucin simtrica sobre codos,
rodillas, manos y zona peribucal. En la biopsia cutnea
hay ausencia de malanocitos. Tras fotoquimioterapia
sistmica con psoralenos han pigmentado parcialmente
las lesiones. El diagnstico de esta paciente es:
A) Esclerosis tuberosa.
B) Lepra.
C) Hipomelanosis guttata idioptica.
D) Vitligo.
E) Hipomelanosis de Ito.
53. Nio de 10 aos de edad que acude a la consulta
por amigdalitis pultcea con adenopatias cervicales. Se
pone tratamiento con penicilina a dosis correctas. A las
72 horas acude de nuevo al no experimentar mejoria.
Debemos pensar en:
A) Posible Mononucleosis infecciosa.
B) Posible amigdalitis viral de etiologa diferente al
Estreptococo beta hemoltico del grupo A.
C) Linfoma.
D) V. Parainfluenzae.
E) Rubola.
54. La presencia de vmitos en la infancia es uno de los
motivos ms frecuentes de consulta. Pueden ser
debidos a enfermedades digestivas y extradigestivas.
Cul de las siguientes patologas produce menos
frecuentemente vmitos en la etapa de la lactancia?
A) Reflujo gastroesofgico.
B) Estenosis hipertrfica de ploro.
C) Apendicitis.
D) Invaginacin intestinal.
E) Gastroenteritis.
55. Ante un nio que de forma brusca presenta
hipotensin, vmitos y colapso cardiovascular que no
responde a la administracin de drogas vasoactivas o
catecolaminas y en la analtica realizada presenta
hiponatremia e hiperpotasemia, se debe considerar
como probable diagnstico:
A) Insuficiencia cardaca.
B) Insuficiencia suprarrenal.
C) Diabetes juvenil.
D) Diabetes inspida.
E) Intoxicacin por monxido de carbono.

Examen Simulacro :: Ciclo de Revisin en Medicina :: Sbado 4 de Agosto del 2012 :: www.estudiosmyc.com
56. Un varn de 17 aos se encuentra mareado, con
vmitos y en el transcurso de unos minutos se halla
tumbado en el suelo en coma, con una exploracin
neurolgica normal. La causa ms probable sera:
A) Ingestin de barbitricos.
B) Hemorragia subaracnoidea.
C) Coma etlico.
D) Status convulsivo.
E) Tumor cerebral.
57. Un varn de 4 aos tiene lesiones purpricas
palpables, simtricas, de 3 das de evolucin en las
extremidades inferiores. Los estudios hematolgicos
revelan:
Hemoglobina:
10
g/dl;
recuento
leucocitario
16.500/mm3; recuento plaquetario 240.000/mm3 y VSG
de 45 mm/hora. La etiologa ms probable es:
A) Maltrato infantil.
B) Prpura de Schonlein Henoch.
C) Enfermedad de Kawasaki.
D) Meningococemia.
E) Enfermedad de Von Willebrand.
58. Durante la evaluacin previa al ingreso en el colegio
de un nio de 5 aos, se detecta retraso en el habla.
Como antecedentes se recogen episodios reiterados de
cuadros catarrales sin control mdico. Cul de las
siguientes es la casusa ms probabnle de esta
situacin?
A) Trastorno de dficit de atencin con hiperactividad.
B) Hipoacusia de conduccin.
C) Disfuncin de la Trompa de Eustaquio.
D) Retraso mental.
E) Hipoacusia neurosensorial.
59. Nia de 7 aos sin antecedentes de inters que
acude a urgencias por dolor abdominal generalizado y
vmitos desde doce horas antes. Deposicin normal. No
antecedentes quirrgicos previos. A la exploracin
presenta abdomen muy distendido y dolor en zona
periumbilical con aumento de ruidos intestinales,
Blumberg (-). En la Radiografa de abdomen en
bipedestacin se observa obstruccin a nivel de
intestino delgado. El diagnstico ms probable sera:
A) Invaginacin intestinal.
B) Brida intestinal.
C) Malrotacin intestinal.
D) Divertculo de Meckel.
E) Estenosis ileal congnita.
60. Nio de 5 meses que llevan a la consulta por tos y
secrecin nasal desde hace 24 horas. Toma mal los
biberones por presentar fatiga. Se observa febrcula y
retraccion intercostal, con zonas de hipoventilacin y
estertores en la auscultacion respiratoria. Existen otros
familiares con cuadro catarral. Se le pone tratamiento
sintomtico y se aconseja la revisin a las 24 horas. En
la nueva visita el nio ha empeorado, con importante
insuficiencia respiratoria, sibilancias, tos continua y

fiebre de 38C. Cul de los siguientes cuadros cree


que presenta el nio?:
A) Catarro habitual descendente con evolucin a asma.
B) Aspiracin de cuerpo extrao.
C) Bronquiolitis.
D) Epiglotitis.
E) Laringitis.
61. Un nio nacido a trmino de 2.100 gr. de peso
presenta irritabilidad y temblores amplios a las 36 horas
de vida. Se alimenta mal y tiene diarrea y obstruccin
nasal. Cul es el diagnostico ms probable?:
A) Hipocalcemia.
B) Hipomagnesemia.
C) Dficit de piridoxina.
D) Sindrome de abstinencia por adicin materna a
herona.
E) Hipoglucemia.
62. Lactante de dos meses que presenta llanto agudo,
en crisis, desde hace 20 das, sntomas motores y heces
normales para su edad. El diagnstico ms probable
ser?:
A) Gastroenteritis aguda.
B) Intolerancia a la lactosa.
C) Clico del lactante.
D) Otitis media aguda.
E) Hernia inguinal.
63. Lactante de tres meses sin antecedentes previos de
inters, que en el examen de salud se le detecta un
deterioro en las adquisiciones sicomotoras, el resto de la
exploracin por aparatos es normal. En la anamnesis
presenta sacudidas musculares breves de cabeza y
extremidades superiores. Cul sera el diagnstico
ms probable?:
A) Clico del lactante.
B) Hemorragia cerebral.
C) Sndrome de West.
D) Sndrome de Lenaux-Gastaut.
E) Fenilcetonuria.
64. Ante un lactante de 1,5 meses de edad que presenta
ictericia debemos pensar en todos los siguientes
cuadros, excepto en:
A) Infeccin urinaria.
B) Lactancia materna.
C) Atresia congnita de vas biliares.
D) Ictericia fisiolgica.
E) Hipotiroidismo congnito.
65. Un paciente acude por cuadro de dolor intenso
epigstrico, de aparicin brusca, acompaado de
vmitos en los que slo consigue arrojar saliva, gran
distensin abdominal alta e imposibilidad para pasar
ms all del esfago distal con una sonda nasogstrica,
cul sera la principal sospecha diagnstica?:
A) Perforacin gstrica.
B) Estenosis pilrica aguda.
C) Vlvulo gstrico agudo.

Examen Simulacro :: Ciclo de Revisin en Medicina :: Sbado 4 de Agosto del 2012 :: www.estudiosmyc.com
D) Sndrome de Boerhaave.
E) Tricobezoar.
66. Un paciente de 60 aos refiere dolor en epigastrio
desde hace unas tres semanas y en menor medida
desde meses antes, que se alivia con los alimentos y
reaparece 2 horas despus de las comidas, con
irradiacin a hipocondrio derecho. En la gastroscopia
muestra una lcera de 1 cm. en curvadura menor, con
bordes netos bien definidos, ausencia de islotes de
tejido dentro del nicho ulceroso, con mucosa de aspecto
normal. Ante lo cual Ud.:
A) Decide no practicar biopsia de la lcera por tener
caractersticas de benignidad.
B) Slo practicar biopsia del fondo del nicho.
C) Practicara biopsias mltiples independientemente de
las caractersticas del nicho ulceroso.
D) Algunos de los signos descritos son sugestivos de
malignidad, por lo que practicara biopsias mltiples.
E) Por no tener caractersticas de malignidad claras, no
practicara biopsias.
67. Paciente de 84 aos que presenta cuadro de diarrea
mucosa con decaimiento generalizado y prdida de
peso. Analticamente el paciente presenta anemia con
hipopotasemia, hiponatremia e hipocloremia; al tacto
rectal se palpa una masa homognea de consistencia
blanda, recubierta de moco, no dolorosa. Este paciente
presentar con gran probabilidad:
A) Hemorroides.
B) Enteropata pierde protenas.
C) Leiomioma de recto.
D) Adenoma velloso de recto.
E) Hamartoma rectal.
68. Mujer de 42 aos que acude al hospital con historia
de 10 aos de disfagia, primero para lquidos y
posteriormente para slidos, y que en la actualidad
presenta regurgitaciones de carcter principalmente
nocturno. Cul sera la prueba diagnstica esencial?:
A) Rx de trax.
B) Trnsito digestivo.
C) Endoscopia alta.
D) Manometra esofgica.
E) TAC torcica.
69. Un varn de 60 aos sin antecedentes personales ni
familiares de inters presenta sangre roja mezclada con
las heces. El mdico realiza una inspeccin perianal y
un tacto rectal encontrando hemorroides internas grado
III. La actitud ms correcta sera:
A) Medidas higinicas ms pomada anti hemorroidal.
B) Solicitar endoscopia alta.
C) Solicitar un enema opaco.
D) Solicitar rectoscopia y enema opaco.
E) Solicitar colonoscopia total.
70. Paciente de 43 aos en tratamiento por colitis
ulcerosa con corticoides y sulfasalazina; acude a
urgencias por malestar general, fiebre, distensin

abdominal, nuseas y vmitos. Presenta gran distensin


abdominal, intenso dolor a la palpacin y signos de
irritacin peritoneal. En la analtica presenta marcada
leucocitosis con desviacin izquierda. En la RX simple
de abdomen se observa gran dilatacin del colon. El
paso siguiente es:
A) Intervencin quirrgica.
B) Rectocolonoscopia.
C) Inmunosupresores.
D) Sonda nasogstrica, sueros y corticoides iv.
E) TAC abdominal.
71. Una mujer de 52 aos diagnosticada de lcera
gstrica de 2 cm. con biopsia negativa por malignidad, a
las 8 semanas de tratamiento con ranitidina 150
mg cada 12 horas, se somete a control endoscpico
encontrndose una lcera de 0,5 cm, con nueva biopsia
negativa y totalmente asintomtica. Cul es la
conducta preferida?:
A) Cambiar el medicamento a fenotidina.
B) Aadir curalfato a la ranitidina.
C) Remitir por ciruga.
D) Continuar con ranitidina 8 semanas ms.
E) Suspender toda medicacin.
72. Una mujer de 76 aos presenta vmito en posos de
caf. En la endoscopia se encontr un plipo en cuerpo
gstrico. No se identific ningn otro origen de la
hemorragia. El hematcrito es del 28%. Cul es la
mejor opcin teraputica?:
A) Biopsia con pinzas por estudio histolgico, si es
benigna ninn tratamiento adicional.
B) Biopsia con pinzas, si es benigna, antagonistas H2.
C) Polipectoma endoscpica con asas.
D) Reseccin quirrgica.
E) Actitud expectante por si se repitiese la hemorragia.
73. Un paciente de 45 aos con anemia ferropnica y
colonoscopia normal se someti a endoscopia alta
observndose un duodeno con mucosa testoreada sin
lesiones sugerentes de hemorragia. La biopsia mostr
atrofia total de vellosidades, lo cual sugiere:
A) Glardiasis.
B) Mucosa normal.
C) Celaca.
D) Enfermedad de Crohn.
E) Linfoma intestinal.
74. Una mujer de 65 aos con artritis reumatoide
deformante grave se presenta en el hospital por dolor
periumbilical de inicio nocturno, con aumento rpido de
intensidad. La exploracin abdominal era casi normal
con hemocultivo positivo, recuento de leucocitos de
20.000 mm3 con desviacin a la izquierda y VSG > 100
mm/h. Cul es el diagnstico ms probable?:
A) Colitis ulcerosa.
B) Enfermedad de Crohn.
C) Isquemia intestinal.
D) Parasitosis intestinal.
E) Angiodisplasia de colon.

Examen Simulacro :: Ciclo de Revisin en Medicina :: Sbado 4 de Agosto del 2012 :: www.estudiosmyc.com
75. Varn de 78 aos con debilidad, prdida de peso,
diarrea, artritis y fiebre, durante el ltimo ao. En el
examen
fsico:
prdida
de
masa
muscular,
linfadenopata y tumefaccin en rodilla izquierda. En
pruebas de laboratorio: anemia ferropnica y hemault
positivo. Con Rx abdomen y enema opaco banales. El
diagnstico diferencial debe incluir todos los siguientes,
excepto:
A) SIDA.
B) Enfermedad de Crohn.
C) Vasculitis reumatoide.
D) Enfermedad de Whipple.
E) Singellosis.
76. Se hace colonoscopia en un enfermo, donde
aparece un plipo de 8 mm en colon sigmoide que se
extirpa sin observar otras lesiones. Cul de los
siguientes considera el intervalo de vigilancia en este
paciente?:
A) Seis meses.
B) Un ao.
C) Dos aos.
D) Tres aos.
E) Cinco aos.
77. Nia de 6 aos remitida a consulta por hemorragia
vginal, que presenta un desarrollo mamario, en etapa III
de Tanner, estatura alta, con una edad sea de 9 aos
(Rx. de mano y mueca izda.), valores basales de
gonadotropinas y estradiol elevados para la prepubertad
con ovarios aumentados para la edad y con mltiples
quistes de dimetro igual o mayor de 14 mm. Examen
neurolgico clnico-radiolgico normal, sin pigmentacin
cutnea en mancha de caf con leche y estudio
hormonal tiroideo y suprarrenal normal. El tratamiento
de eleccin es:
A) Laparocopia diagnstico-teraputica ya que su
etiologa es un tumor ovrico.
B) Agonistas de GnRH, ya que estamos ante una PPV,
y los agonistas de GnRH son los nicos que retrasan el
desarrollo sexual y la maduracin esqueltica.
C) Danazol.
D) Acetato de medroxiprogesterona ya que producen un
retraso en el desarrollo sexual y muy buenos resultados
en el control del crecimiento.
E) No precisa tratamiento, aunque s una vigilancia
anual.
78. Una paciente de 30 aos, que consulta por
amenorrea secundaria, presenta concentraciones
plasmticas basales de FSH 2 MUI/ml., LH 1,5, MUI/ml.,
prolactina 9 ngr./ml. Tras la administracin de
gestgenos 10 mgr./da durante 5 das no se observa
sangrado vaginal. En cambio, tras la administracin de
estrgenos conjugados durante 21 das y en los 5
ltimos das gestgenos aparece una menstruacin. De
las siguientes causas de amenorrea cul es la que
corresponde con el cuadro clnico?:
A) Sndrome de ovario poliqustico.
B) Fallo ovrico autoinmune.

C) Prolactinoma hipofisario.
D) Sndrome de Asherman.
E) Tumor cerebral.
79. Una mujer de 54 aos solicita tratamiento hormonal
sustitutivo
por
sntomas
neurovegetativos
y
manifestaciones genitourinarias importantes. Refiere
que la >FUR fue hace un ao y medio, pero que hace 2
meses ha empezado a sangar muy abundantemente.
Presenta exploracin fsica y mamografa normal. Cul
es la actitud ms apropiada en esta paciente?:
A) Tratamiento con gestgenos.
B) Tratamiento con estrgenos.
C) Tratamiento con terapia combinada.
D) Ecografa transvaginal para valorar lnea media y si
existe alguna duda de patologa endometrial realizar
histologa endometrial.
E) Citologa (triple toma).
80. Una mujer de 45 aos con el antecedente de un
proceso gripal hace dos semanas por el que fue tratado
con amoxicilina presenta ahora un cuadro de prurito
vulvar y leucorrea. En la exploracin se aprecia
enrojecimiento y edemas de la vulva y del introito y
secrecin vaginal blanca grumosa de aspecto caseoso.
En la mucosa vaginal aparecen unas placas
blanquecinas irregulares que se desprenden con
facilidad y dejan ulceraciones rojas superficiales. La
etiologa probable es?:
A) Candidiasis o moniliasis.
B) Herpes genital.
C) Tricomaniasis.
D) Gardnerella vaginalis.
E) Clamidias.
81. Mujer de 40 aos que consulta por cuadro de
poliartritis simtrica de grandes y pequeas
articulaciones de 15 das de evolucin. En la analtica
destaca un factor reumatoide positivo siendo el resto del
estudio inmunolgico negativo. Qu diagnstico
realizara?:
A) Artritis reumatoide.
B) Artritis paraneoplsica.
C) Artritis no filiada.
D) Enfermedad de Still del adulto.
E) Poliartritis vrica.
82. Mujer de 65 aos que ingresa por cuadro de cefalea,
fiebre y dolor con limitacin de ambos hombros y ambas
caderas de 2 meses de evolucin. El resto de
anamnesis y exploracin fsica no aporta datos
relevantes. La analtica pone de manifiesto una gran
elevacin de los reactantes de fase aguda (VSG y
PCR). La conducta a seguir sera:
A) Diagnosticar a la paciente de polimialgia reumtica e
iniciar tratamiento.
B) Realizar biopsia de la arteria temporal para descartar
la existencia de arteritis pues el tratamiento difiere.

Examen Simulacro :: Ciclo de Revisin en Medicina :: Sbado 4 de Agosto del 2012 :: www.estudiosmyc.com
C) Iniciar tratamiento con prednisona 1 mg./kg./da para
curarse en salud ante la posible existencia de una
arteritis.
D) Realizar artrocentesis de un hombro o una cadera
para descartar primero la existencia de una artritis
sptica.
E) Iniciar tratamiento con 20 mg./da de prednisona y si
no mejorara en 15 das subir a 1 mg./kg./da.
83. Varn de 25 aos afecto clnica y radiolgicamente
de una sacroiletis unilateral de 2 meses de evolucin.
La conducta a seguir es la siguiente:
A) Solicitar analtica con HLA y dar tratamiento con
AINE
ante
la
sospecha
diagnstica
de
espondiloartropata.
B) Adems de lo anterior aadir salazopirina al
tratamiento pues probablemente padecer una
espondilitis anquilosante.
C) Adems de lo anterior realizar una buena anamnesis
y exploracin fsica para descartar la existencia de
psoriasis.
D) Realizar anamnesis y exploracin fsica, solicitar
analtica con HLA as como ppD y serologas a brucella.
E) Lo primero es realizar una artrocentesis de la
articulacin afecta para descartar proceso infeccioso
crnico.
84. Mujer de 45 aos que acude a la consulta por dolor
seo generalizado desde hace varios aos. La
conducta a seguir es:
A) Diagnosticarla de fibromialgia y tratarla con
analgsicos y antidepresivos.
B) Sospechar la posible existencia de proceso
metastsico y realizar un completo estudio de bsqueda
del tumor primario.
C) Realizar una correcta anamnesis, exploracin fsica y
solicitar analtica completa con hormonas tiroideas y
CPK.
D) Adems de lo explicado en el apartado 3., solicitar
una gammagrafa sea para asegurarse de la existencia
de proceso inflamatorio articular.
E) Realizar una anamnesis y exploracin fsica
correctas. Solicitar un estudio analtico con VSG y PCR.
Cuando exista sospecha clnica solicitar determinacin
de hormonas tiroideas, anticuerpos anti-ANA y creatinP-quinasas.
85. Varn de 45 aos que acude a la consulta por
presentar dolor, tumefaccin y edema a nivel de la mano
izquierda. Entre sus antecedentes patolgicos
nicamente destaca la luxacin del hombro izquierdo 1
mes antes. La conducta a seguir es la siguiente:
A) Realizar artrocentesis de la mueca para descartar
artritis sptica.
B) Solicitar analtica y radiografas ante la sospecha de
artritis de mueca.
C) Solicitar analtica, radiografas y gammagrafa sea
con tecnecio ante la sospecha de DSR.
D) Solicitar analtica con inmunologa y radiografas ante
la sospecha de inicio de proceso inflamatorio crnico.

E) Solicitar analtica, radiografas y gammagrafa con


tecnecio y galio ante la sospecha de DSR.
86. Mujer de 20 aos que acude por dolor y tumefaccin
de
ambos
tobillos
junto
lesiones
cutneas
eritematoviolceas dolorosas en ambas EEII de 10 das
de evolucin. El diagnstico ms probable es:
A) Artritis reactiva.
B) Vasculitis.
C) Sarcoidosis.
D) Enfermedad inflamatoria intestinal.
E) Artropata no filiada.
87. Una paciente de 55 aos sin antecedentes
patolgicos de inters consulta por dorsalgia de inicio
brusco y ritmo mecnico de dolor. Aporta Rx simple de
columna en la que se aprecian aplastamientos
vertebrales mltiples. La exploracin fsica es normal
salvo por la presencia de debilidad muscular proximal.
Ante la sospecha clnica de osteomalacia, cul de las
siguientes exploraciones es ms rentable:
A) VSG y hemograma.
B) Determinacin de calcio, fosforo y fosfatasas
alcalinas.
C) Densitometra sea.
D) Gammagrafa sea.
E) Resonancia magntica.
88. Una paciente de 23 aos con antecedentes de ulcus
duodenal presenta una poliartritis simtrica con afeccin
predominante de manos. En la exploracin fsica, aparte
de la poliartritis, presenta aftas orales. Se practica
analtica general que muestra como nicas alteraciones
VSG: 40 mm/1. hora, leucocitos: 3.000/mm3 (1.200
linfocitos) y ANA + 1/320 patrn homogneo. Cul
sera su diagnstico?:
A) Artritis reumatoide.
B) Lupus eritematoso sistmico.
C) Gota poliarticular.
D) Condrocalcinosis.
E) Artritis reactiva.
89. Cul es el tratamiento indicado en la paciente de la
pregunta anterior?:
A) Antiinflamatorios no esteroideos.
B) Antipaldicos.
C) Glucocorticoides orales.
D) Pulsos de metilprednisolona.
E) Inmunosupresores.
90. Un paciente de 45 aos consulta porque en una
analtica de rutina se ha detectado una uricemia de 9
mg./dl. No existen antecedentes de artritis ni clculos
urinarios. Cul es la actitud correcta?:
A) Iniciar tratamiento con uricosricos.
B) Iniciar tratamiento con uricosricos y colchicina.
C) Iniciar tratamiento con alopurinol.
D) Iniciar tratamiento con alopurinol y colchicina.
E) No precisa tratamiento.

Examen Simulacro :: Ciclo de Revisin en Medicina :: Sbado 4 de Agosto del 2012 :: www.estudiosmyc.com
91. Una paciente de 15 aos es remitida por sospecha
de fiebre reumtica: tras un cuadro gripal con
artromialgias generalizadas se determinaron las
antiestreptolisinas (ASLO), que son positivas. Cul de
las siguientes afirmaciones es verdadera?:
A) Ante el cuadro clnico de la paciente y las ASLO +, el
diagnstico de fiebre reumtica es seguro.
B) Con las ASLO +, bastara para diagnosticar la fiebre
reumtica.
C) La fiebre reumtica es un tipo de artritis sptica.
D) Con estos datos, no se puede realizar el diagnstico
de fiebre reumtica.
E) Debemos iniciar rpidamente tratamiento con
penicilina.
92. Una paciente de 40 aos presenta una gonartosis
con afeccin predominante del compartimento interno
de la rodilla e impotencia funcional severa. Seale la
respuesta verdadera:
A) El tratamiento es reposo absoluto y esperar a que
sea ms mayor para colocarle una prtesis.
B) Una osteotoma podra estar indicada.
C) Nunca se debe tratar con antiinflamatorios.
D) No debe realizar fisioterapia ya que podra daarse
ms la articulacin.
E) El uso de un bastn est contraindicado.
93. Un paciente presenta latencia del sueo de menos
de 10 minutos con dificultades para despertarse y
episodios de sueo de 18-20 horas, asociado al
despertarse con hiperfagia, hipersexualidad, aumento
de peso, irritabilidad, depresin, comportamiento
impulsivo, disfuncin vegetativa y alteraciones
neurolgicas. Las siestas diurnas duran varias horas.
Estos episodios duran varias semanas intercalndose
varios meses sin somnolencia. En uno de los episodios
de somnolencia el paciente tuvo un grave accidente. El
diagnstico ms probable es:
A) Epilepsia.
B) Narcolepsia.
C) Simulacin o trastorno conversivo-histeria.
D) Sndrome de Klein-Levin.
E) Apnea del sueo.
94. Una chica de 17 aos acude al servicio de urgencias
trada por su madre por un desmayo. Cuando la vemos
est irritable y dice querer irse de alta porque no le pasa
nada. Va con ropas holgadas y algo ms abrigada que
el resto de pacientes, est delgada y parece ms joven
de lo que le corresponde por su edad. Dice comer
normal y niega usar laxantes o diurticos, su madre dice
que no come con ellos porque es muy activa y est todo
el da fuera. Rompi con el novio hace un ao. Dice que
est harta de tener a su madre siempre pendiente de
ella. El dato que ms nos ayudara a diferenciar una
anorexia nerviosa de otros trastornos sera:
A) Presencia de amenorrea.
B) Peso inferior al 15%.
C) Que la paciente diga que tiene miedo a ganar peso o
que diga que se ve gorda.

D) Que diga que tiene fro.


E) Uas y pelo frgil.
95. Una adolescente de apariencia fsica normal acude
por irritabilidad y tristeza. Comenta que decidi comer
menos en las comidas porque estaba algo gorda.
Haciendo esto consigue adelgazar a temporadas pero
luego se vuelve a engordar. Ahora est en perodo de
transicin, sigue restringiendo las comidas pero se pasa
todo el da picoteando y en ocasiones come ms de lo
que quisiera de forma descontrolada. En esos
descontroles efectivamente la paciente come una gran
cantidad de alimentos especialmente chocolate, galletas
y dulces, aunque en otras ocasiones son salados. Tras
estos atracones se siente muy culpable e irritada y
vomita para evitar engordarse. El diagnstico ms
probable es:
A) Sndrome de Klein-Levin.
B) Depresin bipolar.
C) Bulimia.
D) Esquizofrenia.
E) Anorexia nerviosa tipo compulsivo/purgativo.
96. Una paciente de 20 aos se presenta en la guardia
quejndose al internista de que tiene ataques de
corazn y sensacin de ahogo, sudoracin y sensacin
de mareo que se inicia bruscamente mientras estaba
leyendo relajadamente en su casa, le dur unos
minutos, crey morir y senta que lo que le ocurra no
era real, temiendo perder el control o estar volvindose
loca. Se le realizan pruebas ECG y auscultacin,
detectndose frecuencia cardaca alta y signos
sugerentes de prolapso mitral. Aun as dada la gran
ansiedad de la paciente se llama al psiquiatra ya que
insiste en que se le hagan ms pruebas y en que no se
quiere quedar sola en casa nunca ms, cul es el
posible diagnstico?:
A) Prolapso mitral.
B) Agorafobia.
C) Angor.
D) Trastorno de angustia.
E) A y D.
97. Una paciente de 55 aos acude a consultas por
cansancio especialmente por la maana, falta de
concentracin desde hace 1 mes. Dice estar por las
maanas despierta antes de hora sin poder dormir. Ha
perdido apetito si bien come como siempre pero
forzndose, por lo que no ha perdido peso. Pierde el hilo
de las conversaciones y est irritable. Ha dejado de
hacer las cosas que le gustaba hacer y no sale de casa
ms que lo justo. Viene acompaada por su hermana y
nos ruega que no le digamos nada a su marido ya que
desde que no hace las cosas de la casa tan bien como
antes no quiere darle ms disgustos. Cul es el
diagnstico ms probable?:
A) Ansiedad y depresin.
B) Demencia.
C) Depresin mayor.
D) Trastorno de personalidad.

Examen Simulacro :: Ciclo de Revisin en Medicina :: Sbado 4 de Agosto del 2012 :: www.estudiosmyc.com
E) Neurosis.
98. El paciente, que es trado por un familiar, dice no
saber porqu est aqu y parece irritado y con
agresividad contenida. El familiar nos hace gestos con
los ojos haciendo ver que el paciente no est muy bien.
Entrevistando a ambos por separado el paciente se
muestra contenido y dice estar bien, mejor que nunca.
El familiar dice que ltimamente ha comprado cosas
innecesarias y ha hecho algunos regalos. Dice que
quiere iniciar la carrera de psicologa, se ha apuntado a
un gimnasio y quiere proponer a su jefe algunos
cambios en la empresa si bien no tiene
responsabilidades directivas. Est muy hablador y
agudo en las conversaciones pero irritable, dominante y
con explosiones de mal genio. Debe dormir 4 horas pero
no est cansado al da siguiente, no hay problemas de
apetito. No ha habido problemas en su trabajo, pero
pasa mucho tiempo en la calle. Cul es el diagnstico
ms correcto?:
A) Esquizofrenia.
B) Mana.
C) Depresin ansiosa.
D) Trastorno esquizoafectivo.
E) Trastorno de personalidad.
99. Paciente trado por la polica por haberlo encontrado
andando descalzo por la carretera. El paciente tiene 28
aos, es de otra ciudad y por la documentacin que
lleva parece que pueda ser estudiante. Parece
deshidratado, completamente desaseado y con olor a
orn. Pupilas normales. La piel est tostada sin
quemaduras en las zonas de exposicin al sol. Est
ausente, como absorto de manera que al hablarle ms
fuerte se sobresalta y nos atiende momentneamente
sonrindonos de forma inapropiada hasta que ladea la
cabeza y la gira sin motivo. Murmura algo y mira al
techo, permaneciendo con los brazos extendidos. Cul
es el diagnstico ms probable?:
A) Intoxicacin por LSD.
B) Esquizofrenia.
C) Demencia.
D) Cuadro manaco.
E) Histeria.

100. Acude la madre de un presunto paciente de 18


aos porque su hijo no sale casi nada de casa, parece
evitarles. Se hace su propia comida a partir de
conservas generalmente y come aparte. Nunca usa
conservas o alimentos ya empezados, ni acaba los que
no usa por completo. Alguna vez lo ha visto en el pasillo
haciendo posturas extraas o dando golpes de krate.
Fue aficionado a las lecturas de ovnis y ciencia ficcin.
Fue regular estudiante. No trabaja ni estudia. Cierra
todas las persianas y se queja de que los vecinos estn
demasiado pendientes de ellos. Por las noches a veces
no duerme. Dice que puede or al vecino del tercero
(ellos viven en el primero) hablar mal de ellos y que

siempre est comentando lo que l va haciendo. Cul


es el diagnstico ms probable?:
A) Esquizofrenia.
B) Depresin psictica.
C) Mana.
D) Demencia.
E) Klein-Levin.

TEMA B
1. Paciente de 64 aos ingresado para ciruga diferida
que al da siguiente del ingreso presenta cuadro de
hiperactividad
vegetativa,
temblor,
sobresaltos,
hiperactividad e hiperalerta pero con confusin,
alucinaciones y convulsiones. Cul es la causa ms
frecuente?:
A) Abstinencia a alcohol.
B) Sobredosis de neurolpticos.
C) Sobredosis de benzodiacepinas.
D) Sobredosis de anticolinrgicos.
E) Ingestin de barbitricos.
2. Mujer de 25 aos que acude por intento autoltico con
benzodiacepinas. Es el quinto intento en 7 aos, en esta
ocasin tras una pelea familiar. Su madre dice que es
muy voluble y caprichosa que siempre consigue salirse
con la suya mediante grandes crisis de nervios y peleas.
La madre dice estar harta ya que siempre est
regandola mientras que su marido la perdona todo y
no la castiga lo suficiente. No tiene novio fijo ya que
rompe con ellos en seguida, es mala estudiante y no es
capaz de dedicarse a nada fijo ya que se aburre en
seguida, dice sentirse en ocasiones muy vaca y en
otras muy inspirada en hacer el bien pero nada le dura
mucho, quisiera poder tener una relacin con un hombre
romntico. Sale mucho por la noche y en ocasiones no
llega hasta el da siguiente por la tarde, llegando ebria
en ocasiones. Ha perdido a sus amigas, que se
quejaban entre otras cosas de que las estaba llamando
continuamente. Le han diagnosticado previamente de
hepatitis. El diagnstico de esta paciente es:
A) Esquizofrenia.
B) Mana.
C) Trastorno de personalidad lmite o inestable.
D) Trastorno de personalidad por dependencia.
E) Trastorno esquizoide.
3. El paciente acude a nuestra consulta porque quisiera
dejar de hacer algunas de las cosas que actualmente
hace ya que le quitan mucho tiempo. El paciente tarda
mucho tiempo en vestirse ya que ha de pensar la ropa
que le dar suerte para esa maana. Teme que si no
coge la adecuada algo podra salir mal. No puede evitar
sumar los nmeros que ve en las matrculas de los
coches, as consigue neutralizar los pensamientos que
le vienen sobre la posibilidad de que el coche tenga un
accidente. El sabe que stos son tonteras suyas pero
son cosas que no puede quitarse de la cabeza. Es muy
ordenado en algunos aspectos pero se le acumula el

Examen Simulacro :: Ciclo de Revisin en Medicina :: Sbado 4 de Agosto del 2012 :: www.estudiosmyc.com
trabajo y no consigue hacer las cosas que tiene
preparadas en una lista para ese da. Le gusta corregir
sus escritos hasta que estn impecables, lo mismo le
ocurre con su trabajo, es muy perfeccionista, exigente
consigo mismo y con los dems, es incapaz de delegar
trabajo. El diagnstico del paciente es:
A) Esquizofrenia.
B) Trastorno de personalidad dependiente.
C) Trastorno obsesivo-compulsivo.
D) Fobia.
E) Depresin.
4. Paciente que acude por no poder salir de casa sola.
Dice que le da miedo salir por sitios solitarios por si le
ocurre algo y que en los sitios con mucha gente se
agobia y se pone muy nerviosa teniendo crisis de
ansiedad con palpitaciones, se imagina as misma
necesitada de ayuda y sin poder salir. Si se encuentra
acompaada se encuentra mejor ya que esa persona
podra ayudarla si ocurriera algo. Desde hace unos
meses tiene unas crisis parecidas pero ms cortas
estando relajada, incluso acompaada, sin aviso, la
primera estando en la calle distrada viendo un
escaparate. Cuando le dan en casa es capaz de salir a
la calle para coger aire o bien abre las ventanas. Desde
la primera crisis cogi algo de miedo a estar sola hasta
llegar a la situacin actual. Se ha comprado un telfono
mvil para poder pedir ayuda si fuera necesario. Si entra
en un cine siempre est pendiente de sentarse cerca de
alguna salida. El cuadro clnico de la paciente es:
A) Agorafobia.
B) Hipocondra.
C) Trastorno facticio.
D) Trastorno obsesivo.
E) Agorafobia con crisis de pnico.
5. Ante un paciente con dolor epigstrico irradiado en
hemicinturn, nuseas, distensin abdominal, descenso
de los ruidos hidroareos y ascenso del ST en el
electrocardiograma, cul sera su actitud?:
A) Llamara de inmediato a la Unidad de Cuidados
Intensivos para tratar el infarto agudo de miocardio.
B) Repetira el electrocardiograma a las 8 horas para
confirmar el diagnstico.
C) Lo diagnosticara de pancreatitis aguda.
D) Su diagnstico sera aneurisma disecante de aorta.
E) El paciente tiene una pericarditis aguda.
6. Ingresa en el Servicio de Urgencias un paciente con
dolor epigstrico irradiado en hemicinturn y amilasa
elevada, que pierde de forma brusca visin qu
pensara que ha ocurrido?:
A) Retinopata de Purtscher.
B) Accidente cerebro vascular agudo de la regin
occipital posterior.
C) Desprendimiento de retina.
D) Simulacin.
E) Glaucoma agudo.

7. Ante una paciente que ingresa en el Servicio de


Urgencias con dolor abdominal, elevacin de la amilasa
y la glucosa en suero y con un pH de 7,1 en qu
pensara primero?:
A) Pancreatitis aguda.
B) Rotura de embarazo ectpico.
C) Quiste de ovario.
D) Cetoacidosis diabtica.
E) Ulcera de estmago.
8. Un paciente de 18 aos consulta por ictericia sin
fiebre y coluria sin prurito. En las pruebas
complementarias presenta: Ac. anti VHA IgG positivos,
Ac. antiHBs positivos, Ac. anti HBc IgG positivos,
ecografa heptica normal, hay predominio de la
bilirrubina
directa
con
monoconjugados,
la
colecistografa oral es normal y al dar al paciente
fenobarbital se observa disminucin de las cifras de
bilirrubina total. Cul sera su diagnstico de
sospecha?:
A) Coledocolitiasis.
B) Sndrome de Gilbert.
C) Sndrome de Dubin-Johnson.
D) Hepatitis aguda por VHA, con hepatitis aguda por
VHB curada.
E) Sndrome de Rotor.
9. Ante un paciente con cirrosis alcohlica en estado C10 de Child-Pugh que ingresa por descompensacin
hidrpica, y que se encuentra hipotenso, taquicrdico,
oligrico, con un sodio srico de 125 mEq/l. y de 5
mEq/l. en orina, con sedimento normal, creatinina srica
3 mg./dl. y un aclaramiento de creatinina de 40 ml., en
qu pensara usted?:
A) Deficiente tratamiento diurtico.
B) Insuficiencia renal prerrenal secundaria a tercer
espacio.
C) Sndrome hepatorrenal.
D) Glomerulonefritis mesangial IgA.
E) Insuficiencia renal prerrenal secundaria a
deshidratacin.
10. Paciente varn de 28 aos, VIH(+), en tratamiento
con DDI (Dideoxinosina), que acude al hospital por dolor
intenso en epigastrio irradiado hacia la espalda, que
mejora al flexionar el tronco, acompaado de nuseas y
vmitos. Cul de los siguientes diagnsticos es el ms
probable?:
A) Pericarditis.
B) Obstruccin intestinal.
C) Apendicitis.
D) Pancreatitis aguda.
E) Endocarditis.
11. Tras una semana de ingreso por una pancreatitis
aguda, a pesar del tratamiento mdico sintomtico
persiste la fiebre, leucocitosis e hiperamilasemia. Se
aprecia a la palpacin una masa abdominal localizada
en hipocondrio derecho. Cul es el diagnstico ms
probable?:

Examen Simulacro :: Ciclo de Revisin en Medicina :: Sbado 4 de Agosto del 2012 :: www.estudiosmyc.com
A) Carcinoma de pncreas.
B) Flemn pancretico.
C) Pancreatitis crnica.
D) Plastn secundario a perforacin duodenal.
E) Quiste hidatdico.
12. El principal diagnstico de presuncin ante una
enferma que acude por un cuadro de diarrea acuosa,
dos lceras duodenales resistentes al tratamiento
mdico e hipercalcemia es:
A) Insulinoma.
B) Gastrinoma.
C) Adenocarcinoma de pncreas.
D) Somatostatinoma.
E) Vipoma.
13. Varn de 60 aos que acude por prdida de peso de
10 kilos, dolor sordo en piso abdominal superior de 3
meses de evolucin, acompaado de ictericia
mucocutnea y deposicin de color blanco desde hace 5
das. A la exploracin destaca masa palpable en
hipocondrio derecho. Qu patologa se sospechara en
primer lugar?:
A) Adenocarcinoma pancretico.
B) Tumor gstrico.
C) Coledocolitiasis.
D) Colecistitis.
E) Hepatitis aguda.
14. Varn de 45 aos, etilismo crnico, con dolor
abdominal en el hipocondrio izquierdo, de 3 meses de
evolucin, que empeora con la ingesta y se acompaa
de deposiciones diarreicas pastosas muy mal olientes.
En las exploraciones complementarias destaca:
glucemia de 280, amilasemia en los lmites de la
normalidad y en la placa de abdomen mltiples
calcificaciones a nivel de L2:
A) Pancreatitis aguda.
B) Clico biliar.
C) Pancreatitis crnica.
D) Hepatitis aguda.
E) Ulcus gstrico
15. Paciente de 60 aos con fibrilacin auricular en
tratamiento con amiodarona desde hace meses,
colelitiasis
diagnosticada
ecogrficamente
e
insuficiencia renal crnica moderada. Consulta por
cuadro de anorexia, astenia, nuseas, vmitos e ictericia
mucocutnea. En los datos de laboratorio destaca un
leve aumento de las transaminasas y una creatinina de
2,2. Se realiza una biopsia heptica, observndose al
microscopio electrnico cuerpos lisosmicos lamelares
cargados de fosfolpidos. Qu proceso patolgico le
sugieren estos datos:
A) Hepatitis viral aguda.
B) Clico biliar.
C) Pancreatitis aguda.
D) Hepatitis txica de origen medicamentoso.
E) Hepatitis viral crnica.

16. Un varn de 13 aos que haba presentado un


cuadro febril de vas respiratorias altas de una semana
de evolucin acude a urgencias por vmitos sbitos e
incoercibles. La semana previa haba consumido cido
acetilsaliclico para la sintomatologa respiratoria y tres
semanas antes estuvo en contacto con un paciente con
hepatitis aguda por virus B. Dos das despus del
ingreso el paciente se encuentra estuporoso con
convulsiones sin signos neurolgicos de focalidad y
dolor en hipocondrio derecho con hepatomegalia. Cul
sera su diagnstico?:
A) Cuadro convulsivo en relacin con la fiebre.
B) Hepatitis fulminante vrica.
C) Sndrome de Reye.
D) Reagudizacin del cuadro gripal.
E) Intoxicacin por cido acetilsaliclico.
17. Un hombre de 21 aos recibi un golpe en el escroto
dos horas antes de ser examinado en urgencias. Su
escroto est tenso, hinhado, y equimtico. No se
pueden palpar los testculos. El paso siguiente es:
A) Hacer uretrografa retrgrada.
B) Hacerle un Doppler de flujo color.
C) Realizar una ecografa del escroto.
D) Tratarle con hielo, reposo y suspensorio.
E) Hacer exploracin quirrgica del escroto.
18. Un hombre de 64 aos presenta una hinchazn
indolora del testculo derecho de tres meses de
duracin. Los resultados de los anlisis de orina son
normales y la ecografa testicular muestra un aumento
del tamao de dicho teste.
El diagnstico ms probable es:
A) Linfoma testicular.
B) Leucemia linftica crnica.
C) Seminoma espermatoctico.
D) Teratocarcinoma.
E) Carcinoma de clulas embrionarias.
19. Una mujer sana presenta de forma aguda
polaquiuria y disuria. En el sedimento urinario se
observan ms de 5 leucocitos por campo y el
urinocultivo revela
1.000 colonias de E. coli por ml. El diagnstico ms
probable es:
A) Uretritis por clamydia.
B) Sndrome uretral.
C) Bacteriuria por E. coli.
D) Cistitis intersticial.
E) Cistitis qustica.
20. Un hombre de 74 aos con cncer de prstata
metastsico conocido presenta dolor agudo de cadera
derecha. Hace dos aos se le practic una orquiectoma
pero no ha recibido ningn otro tratamiento. Su estado
general es bueno. Una gammagrafa sea muestra
metstasis difusas y una Rx simple revela una ostelisis
en el acetbulo derecho. La siguiente medida a aplicar
es:
A) Flutamida.

Examen Simulacro :: Ciclo de Revisin en Medicina :: Sbado 4 de Agosto del 2012 :: www.estudiosmyc.com
B) Anlogos de la LH-RH.
C) Radioterapia localizada.
D) Fosfato de estramustina.
E) Ketoconazol.
21. Un recin nacido presenta hematuria, proteinuria y
creatinina elevada. Las presiones de la arteria umbilical
estn significativamente altas y el paciente desarrolla
una insuficiencia cardaca congestiva. Una gammagrafa
renal revela una ausencia de funcin en el rin
izquierdo. El diagnstico ms probable es:
A) Trombosis de la vena renal.
B) Necrosis cortical renal.
C) Hemorragia adrenal.
D) Rotura de un nefroma mesoblstico.
E) Trombosis de la arteria renal.
22. Un hombre de 35 aos presenta un clico renal
izquierdo. En la Rx de abdomen se observa un clculo
de 3 mm. de dimetro alojado en urter medio. La
necesidad de tratamiento quirrgico depende de:
A) La duracin de los sntomas del paciente.
B) Del nmero de episodios de clicos previos.
C) Del nmero de intervenciones quirrgicas previas.
D) De la presencia de fiebre e infeccin urinaria.
E) De una anormalidad metablica subyacente.
23. A un paciente se le somete a una intervencin de
bypass en el intestino delgado y presenta una litiasis
urinaria. Cul ser la composicin ms probable de la
litiasis?:
A) Acido rico.
B) Urato amnico.
C) Fosfato clcico.
D) Oxalato clcico.
E) Estruvita.
24. El factor pronstico ms importante en los nios que
presentan un tumor de Wilms intracava es:
A) La histologa.
B) El volumen del tumor.
C) La extensin atrial del tumor.
D) La afectacin de ganglios linfticos.
E) La diseminacin del tumor durante la intervencin
quirrgica.
25. Un muchacho de 7 aos presenta de forma sbita
dolor escrotal derecho de 4 horas de duracin. Se
sospecha de una torsin testicular. Cul de las
siguientes observaciones es la ms probable?:
A) La ausencia del reflejo cremastrico.
B) El aumento de la captacin por parte del teste
derecho despus de la exploracin radioisotpica.
C) La presencia del reflejo cremastrico.
D) Una transiluminacin correcta del compartimento
escrotal derecho.
E) La estetoscopia con Doppler ser simtrica en ambos
compartimentos escrotales.

26. En un paciente asmtico, cul de los siguientes


frmacos est contraindicado en el tratamiento de la
incontinencia?:
A) La efedrina.
B) Sudafed.
C) El propranolol.
D) La fenilefrina.
E) Las anfetaminas.
27. Ante un paciente de 60 aos con un cncer de
prstata localizado, mal diferenciado y sin metstasis,
qu tratamiento le recomendara para intentar aumentar
su supervivencia:
A) Prostatectoma radical.
B) Anlogos de la LH-RH.
C) Flutamida.
D) Estramustina.
E) Orquiectoma.
28. Ante un paciente de 55 aos que presenta una
tumoracin vesical que infiltra la muscular y que ocupa
la mitad de la vejiga, cul sera el tratamiento que
empleara con intencin curativa?:
A) Cistectoma radical y derivacin urinaria.
B) Reseccin transuretral vesical.
C) Quimioterapia intravesical.
D) Quimioterapia sistmica.
E) Cistectoma parcial.
29. Paciente mujer de 75 aos de edad, que es trada a
la consulta por cambio progresivo de conducta en los
ltimos meses. Previamente haban observado fallo de
memoria reciente sin poder precisar el momento de
inicio. Destacaba a la exploracin lenguaje pobre y fallos
de juicio. Cul es el diagnstico ms probable en esta
paciente?:
A) Depresin.
B) Sndrome confusional agudo.
C) Demencia tipo Alzheimer.
D) Demencia multiinfarto.
E) Trastorno de la personalidad.
30. Varn de 87 aos, con amaurosis bilateral, que
ingresa por neumona basal derecha en la Unidad de
Agudos del hospital. La noche del ingreso presenta
cuadro de agitacin psicomotriz, con alteracin del nivel
de conciencia y desorientacin temporoespacial. Cul
es el diagnstico ms probable?:
A) Demencia.
B) Depresin delirante.
C) ACVA.
D) Sndrome confusional agudo.
E) Ninguno.
31. Paciente de 85 aos de edad con antecedentes de
insuficiencia cardaca en tratamiento con diurticos, e
insomnio que trataba con lorazepam. Presenta nicturia
2-3 veces. Sufre cada al levantarse bruscamente
durante la noche para ir al retrete. Qu factores
pueden haber contribuido e la cada?:

Examen Simulacro :: Ciclo de Revisin en Medicina :: Sbado 4 de Agosto del 2012 :: www.estudiosmyc.com
A) Frmacos.
B) Hipotensin postural.
C) Factores ambientales.
D) Ninguno.
E) Todos ellos.
32. Paciente de 75 aos de edad, varn, que vive solo.
Es encontrado la maana del da 15 de enero cado en
el suelo. A su llegada al hospital se evidencia afasia
motora y hemipleja derecha diagnosticndose de
ACVA. Cul de las siguientes patologas debe ser
descartada en la valoracin inicial?:
A) Rabdomilisis.
B) Hipertiroidismo.
C) Depresin.
D) Hipotermia.
E) A y D.
33. Paciente mujer de 85 aos que ingresa por fractura
pertrocantrea de cadera derecha colocndose traccin.
Se evidencia neumona basal derecha que obliga a
retrasar la intervencin. La paciente sufre de
incontinencia urinaria que se maneja con catter. A los
cinco das se observa enrojecimiento en regin sacra
que no palidece con la presin, diagnosticndose de
lcera por presin grado I. Cul es el principal factor de
riesgo para esta complicacin?:
A) Incontinencia urinaria.
B) Hipoxemia.
C) Edad avanzada.
D) Inmovilidad.
E) Todos.
34. Paciente de 70 aos que sufre cada al suelo
golpendose en la cabeza. Al cabo de unas semanas
sufre trastornos de conducta, prdida de memoria y
posteriormente alteracin del nivel de conciencia. Qu
diagnstico debe descartarse en primer lugar?:
A) Hipotiroidismo.
B) Demencia tipo Alzheimer.
C) Pseudodemencia.
D) Hematoma subdural.
E) Tumor cerebral.
35. Mujer de 75 aos que consulta por incontinencia
urinaria, en la que predomina la urgencia-miccional y
que no sigue ningn tratamiento farmacolgico habitual.
A travs de la exploracin fsica no se objetiva patologa
orgnica, siendo el residuo postmiccional normal. El
estudio analtico es normal. El tratamiento mdico de
eleccin sera:
A) Sondaje vesical intermitente.
B) Calcioantagonistas.
C) Ciruga.
D) Antocolinrgicos.
E) Colector externo.

36. Durante los ltimos 3 meses un varn de 80 aos


presenta una rpida progresin de una demencia
acompaada de signos extrapiramidales y mioclonas.
El diagnstico de sospecha inicial sera:
A) Demencia senil tipo Alzheimer.
B) Enfermedad de Creutzfeldt-Jakob.
C) Enfermedad de Huntington.
D) Hidrocefalia a presin normal.
E) Enfermedad de Parkinson.
37. Cul de las lesiones cutneas siguientes se asocia
ms frecuentemente con neoplasia oculta en un
paciente anciano?:
A) Penfigoide bulloso.
B) Dermatomiositis.
C) Eritema multiforme.
D) Herpes zoster.
E) Pnfigo vulgar.
38. Anciano de 70 aos con un melanoma maligno de
reciente diagnstico y sin otra patologa mdica. Su
pronstico vital estar ms estrechamente relacionado
con una de las siguientes caractersticas:
A) Ausencia de regresin.
B) Tipo histolgico de la lesin.
C) Grado de invasin.
D) Presencia de ulceracin.
E) Lugar de la lesin.
39. Una mujer de 75 aos con historia de diabetes
mellitus no insulindependiente y epilepsa secundaria a
enfermedad cerebro-vascular padece una inflamacin
con retraccin gingival. Se encuentra en tratamiento con
glipizida y fenobarbital. La causa ms probable de su
proceso gingival ser:
A) Caries dental.
B) Edentulismo.
C) Tratamiento con fenobarbital.
D) Dficit de cinc.
E) Pobre higiene oral.
40. Un varn de 76 aos con historia de diabetes
mellitus de larga evolucin y con datos clnicos de
polineuropata perifrica, empieza a tomar amitriptilina
por prescripcin mdica para las parestesias en
miembros inferiores. De forma rogresiva nota
disminucin del volumen de diuresis y ocasionalmente
escapes involuntarios de orina. En la revisin mdica
siguiente se objetiva deterioro del estado general con
insuficiencia renal. Cul es la causa ms lgica de su
deterioro clnico?:
A) Nefrotoxicidad por amitriptilina.
B) Pielonefritis aguda.
C) Infeccin urinaria de vas bajas.
D) Retencin urinaria con fracaso renal secundario.
E) Glomeruloesclerosis diabtica.

Examen Simulacro :: Ciclo de Revisin en Medicina :: Sbado 4 de Agosto del 2012 :: www.estudiosmyc.com
41. Varn de 45 aos con otalgia derecha y sensacin
de taponamiento auditivo, sin otorrea. A los dos das
presenta aumento del dolor, que se hace retroauricular,
y fiebre en agujas. El Rinne es negativo en odo derecho
y el Weber lo lateraliza a la derecha. El diagnstico ms
probable es:
A) Colesteatoma antral invasivo.
B) Petrositis.
C) Tromboflebitis del seno lateral.
D) Otitis externa maligna.
E) Carcinoma de odo derecho.
42. Mujer de 22 aos que presenta parlisis facial
perifrica derecha. A la exploracin presenta otoscopia
normal y lengua geogrfica fisurada. La paciente
comenta haber tenido otro episodio anteriormente,
aquella vez asociado a edema de labio inferior. El
diagnstico ms probable es:
A) Sndrome de Guillain-Barr.
B) Sndrome de Heerfordt-Waaldenstrm.
C) Dficit de C1-inhibidor.
D) Sndrome de Melkerson-Rosenthal.
E) Parlisis facial de Bell.

supraclavicular. A la exploracin presenta taquicardia y


leve cianosis, un tiroides agrandado y, en la
laringoscopia, parlisis de ambas cuerdas vocales en
posicin paramediana. La actitud que debe seguirse es:
A) Bolo de corticoides endovenosos, ante la posibilidad
de carcinoma subgltico.
B) Tiroidectoma de urgencia, pues probablemente el
tiroides agrandado comprime la trquea.
C) Intubacin y observacin.
D) Traqueostoma de urgencia, pues se trata de una
parlisis recurrencial bilateral secundaria a patologa
tiroidea.
E) Administracin endovenosa de espasmolticos.
47. Un nio de 3 aos es trado a urgencias con un
cuadro de fiebre y tos irritativa "perruna", a lo que se
asocia disfona y cierto grado de disnea. El cuadro es
compatible con todas menos:
A) Cuerpo extrao en vas areas.
B) Laringotraquetis aguda.
C) Epiglotitis aguda.
D) Adenoamigdalitis aguda obstructiva.
E) Edema alrgico.

43. Paciente que presenta hipoacusia neurosensorial


izquierda de larga evolucin con cada en agudo, y
acfeno persistente. La actitud correcta en este caso
sera:
A) No hacer nada, pues se trata de un traumatismo
acstico crnico.
B) Potenciales evocados para descartar neurinoma del
VIII.
C) Instaurar terapia vasodilatadora endovenosa, pues se
trata de una hipoacusia sbita.
D) Intervenir el posible colesteatoma.
E) Administrar sedantes vestibulares ante la posibilidad
de vrtigo de Mnire.

48. Paciente mujer de 25 aos que consulta por prdida


de audicin de comienzo insidioso. Su madre era sorda
y gan audicin tras operarse del odo. Otoscopia
normal. Rinne negativo odo izdo., positivo odo dcho.
Weber a la izda. Audiometra: hipoacusia transmisiva
izquierda. Reflejo estapedial abolido en odo izdo. y
presente en derecho. Timpanometra normal. El
diagnstico probable es:
A) Otosclerosis odo izquierdo.
B) Colesteatoma izquierdo.
C) Fijacin de cadena osicular derecha.
D) Timpanosclerosis izquierda.
E) Luxacin de cadena osicular izquierda.

44. Paciente de 65 aos con rinorrea unilateral


purulenta, dolor hemifacial y epistaxis ocasionales. En
Rx se observan lesiones osteolticas en maxilar.
Probablemente se trate de:
A) Cuerpo extrao nasal.
B) Ocena.
C) Granuloma sangrante de tabique.
D) Carcinoma de fosa nasal.
E) Coriza.

49. Un varn de 30 aos acude a urgencias por vrtigo


perifrico intenso con nistagmus a la derecha y acfeno
en odo izquierdo. Recuerda haber tenido dficit auditivo
izquierdo previo, que le desapareci. Audiometra:
hipoacusia neurosensorial izda. leve. Reflejo estapedial:
derecho en 70 dB; izquierdo en 50 dB. Diagnstico
probable:
A) Neuronitis vestibular.
B) Neurinoma del VIII par con reclutamiento positivo.
C) Tumor de tronco cerebral.
D) Otitis media secretora con fstula perilinftica.
E) Sndrome de Mnire.

45. Un varn de 60 aos consulta por tumoracin


indolora en raz nasal, que desplaza la rbita
producindole
diplopia.
El
cuadro
se
debe
probablemente a:
A) Mucocele etmoidal.
B) Quiste de retencin en seno frontal.
C) Enfermedad de Woakes.
D) Papiloma invertido en techo de fosa nasal.
E) Plasmocitoma solitario en seno frontal.
46. Una mujer de 45 aos acude a la urgencia con un
sndrome de disnea y estridor importantes, con tiraje

50. Un nios de 2 aos presenta rinorrea purulenta de


larga evolucin por fosa nasal derecha, con mala
ventilacin nasal. Son diagnsticos posibles todos
menos:
A) Tuberculosis nasal.
B) Cuerpo extrao intranasal.
C) Sinusitis maxilar.
D) Coriza comn.
E) Rinitis alrgica sobreinfectada.

Examen Simulacro :: Ciclo de Revisin en Medicina :: Sbado 4 de Agosto del 2012 :: www.estudiosmyc.com
51. Paciente de 30 aos que acude a urgencias
presentando una ulceracin amigdalar unilateral. Son
diagnsticos probables todos menos:
A) Amigdalitis tifoidea de Duguet.
B) Angina de Ludwig.
C) Carcinoma escamoso de amgdala.
D) Angina de Plaut-Vincent.
E) Agranulocitosis.
52. Un nios de 12 aos acude a urgencias por
obstruccin nasal bilateral crnica y otitis seromucosa
bilateral. Recientemente ha tenido epistaxis importante
por ambas fosas nasales. El diagnstico ms probable
es:
A) Hipertrofia adenoidea.
B) Ototubaritis asociada a rinitis alrgica.
C) Poliposis nasal bilateral.
D) Adenocarcinoma de etmoides.
E) Angiofibroma nasofarngeo.
53. Mujer de 75 aos con historia de cefalea presenta
prdida sbita de visin unilateral con edema de papila
ipsilateral. Qu medida tomaramos en primer lugar?:
A) TAC.
B) VSG.
C) Ingreso hospitalario paratratamiento antibitico iv.
D) Observacin.
E) Radiografa centrada en el agujero ptico.
54. Mujer de 20 aos con ojo rojo bilateral, acompaado
de
quemosis,
folculos
conjuntivales
tarsales,
adenopata preauricular, sin prdida de visin. La
etiologa ms frecuente ser:
A) Adenovirus.
B) S. aureus.
C) H. influenzae.
D) Queratitis herptica.
E) Parainfluenzae virus.
55. Mujer de 23 aos que presenta prdida brusca de
agudeza visual indolora en ojo derecho, en la
exploracin se observa defecto pupilar aferente relativo
en ojo derecho, segmento anterior normal y F.O. normal.
El diagnstico ms probable es:
A) Neuritis ptica retrobulbar.
B) Histeria.
C) Compresin quiasmtica por tumor hipofisario.
D) Compresin del globo ocular por tumor orbitario.
E) Glaucoma agudo de ngulo cerrado.
56. Varn de 50 aos con ojo rojo y doloroso, midriasis
arreactiva y cmara anterior estrecha. Cul de las
siguientes respuestas es falsa?:
A) No sera raro que fuera hipermtrope.
B) Puede presentar cefalea con nuseas y vmitos.
C) Evitar tomar la PIO por posible etiologa infecciosa.
D) El tratamiento definitivo es lser o ciruga.
E) Puede presentar visin en halos de colores.

57. Ante un varn de 32 aos con sndrome febril y


prdida visual unilateral que presenta en fondo de ojo
lesiones en queso y tomate. Cul de las siguientes
afirmaciones es falsa?:
A) Sera aconsejable realizar serologa HIV.
B) El pronstico vital del enfermo es malo.
C) No necesita tratamiento por ser con frecuencia una
alteracin transitoria.
D) Con bastante probabilidad se deber a CMV.
E) La afectacin suele ser bilateral.
58. Paciente de 70 aos que presenta metamorfosias,
micropsia y disminucin de la agudeza visual de varias
semanas de evolucin, el diagnstico ms probable es:
A) Catarata nuclear.
B) Catarata subcapsular posterior.
C) Degeneracin macular senil.
D) Pars planitis.
E) Hemorragia vtrea.
59. Ante un nio de 5 aos con endotropa que ha
seguido tratamiento con correccin ptica y colusiones y
cuya agudeza visual an no es normal. La pauta a
seguir ser:
A) Correccin quirrgica de la endotropa.
B) Continuar las oclusiones sobre ojo con mejor visin.
C) Continuar las oclusiones sobre ojo con peor visin.
D) No ocluir ms y pasar a otro tratamiento.
E) Observacin.
60. Varn de 70 aos que presenta prdida visual
progresiva unilateral, que precisa cambios sucesivos de
correccin ptica mipica. La causa ms probable es:
A) Degeneracin macular senil.
B) Glaucoma crnico simple.
C) Vitritis senil.
D) Coriorretinosis senil.
E) Catarata nuclear.
61. Varn de 51 aos que sufre sbita prdida visual
total e indolora en ojo derecho, aprecindose en el F.O.
una retina de color blanquecino con mcula rojo-cereza.
El diagnstico ms probable es:
A) Desprendimiento de retina con afectacin macular.
B) Enfermedad de Tay-Sacks.
C) Enfermedad de Newman-Pick.
D) Obstruccin de arteria central de la retina.
E) Obstruccin de vena central de la retina.
62. Varn de 10 aos de edad que presenta estrabismo,
disminucin de agudeza visual y leucocoria en ojo
derecho, sin malformaciones oculares. Qu patologa
debemos descartar como ms probable?:
A) Retinoblastoma.
B) Catarata congnita.
C) Vtreo primario hiperplsico persistente.
D) Enfermedad de Coats.
E) Fibroplasia retrolental.

Examen Simulacro :: Ciclo de Revisin en Medicina :: Sbado 4 de Agosto del 2012 :: www.estudiosmyc.com
63. Paciente de 65 aos con Diabetes mellitus tipo II de
10 aos de evolucin con mal control metablico
presenta disminucin de visin brusca unilateral. La
causa ms probable ser:
A) Isquemia macular.
B) Desprendimiento de retina traccional.
C) Hemorragia vtrea.
D) Edema macular.
E) Obstruccin de arteria central de la retina.
64. Varn de 35 aos sano de carcter nervioso
comienza con metamorfopsias y escotoma central
unilateral, disminucin de visin moderada y patrn
angiogrfico en chimenea. El diagnstico ms probable
ser:
A) Distrofia viteliforme de Best.
B) Retinosis pigmentaria.
C) Degeneracin macular ligada a la edad.
D) Enfermedad de Coats.
E) Coriorretinopata central serosa.
65. Una mujer de 35 aos, no fumadora, atleta de fondo
y sin antecedentes personales de inters refiere en los
ltimos 3 meses una menor respuesta al ejercicio
habitual, con disnea de moderados esfuerzos. Tos no
productiva pero niega fiebre. En una ocasin reciente
present la emisin de varios esputos hemoptoicos, que
no se han vuelto a repetir. La Rx de trax muestra un
patrn reticular fino difuso bilateral y un mnimo derrame
pleural derecho. En las pruebas funcionales llama la
atencin el incremento de los volmenes pulmonares.
De los siguientes, cul le parece el diagnstico ms
probable?:
A) Lupus eritematoso sistmico.
B) Tuberculosis.
C) Sarcoidosis.
D) Linfangioleiomatosis.
E) Enfermedad de Hamman-Rich.
66. Varn de 50 aos, no fumador, que refiere disnea de
moderados esfuerzos de unos 4 meses de evolucin
con tos no productiva. Ha recibido tratamiento con
diurticos de asa (furosemida) en el ltimo mes tras
realizarse una Rx de trax (que no aporta). Acude a
urgencias por incremento de la disnea y expectoracin
de esputos claros. Refiere febrcula de predominio
vespertino y prdida de unos 6 Kg de peso. Presenta
hipoxemia con hipocapnia (insuficiencia respiratoria
parcial) y la Rx de trax muestra densidades difusas
bilaterales, confluentes, mal definidas de predominio
parahiliar con un ndice cardiotorcico en el lmite de la
normalidad. FVC: 65%, FEV1: 70%, FEV1/FVC: 75%.
DLCO: 60%.
A) Insuficiencia cardiaca. Edema agudo de pulmn.
B) Enfermedad de Hamman-Rich.
C) Hemorragia pulmonar.
D) Neumona por CMV.
E) Carcinoma bronquioalveolar.

67. Varn de 64 aos, veterinario de profesin, bebedor


espordico que presenta fiebre, cefalea y artromialgias
desde hace 7 das. Durante las ltimas 48 horas refiere
tos productiva, dolor pleurtico en el costado derecho y
disnea progresiva. Ha sido tratado con eritromicina,
pese a lo que se encuentra febril y desorientado. Se
evidencian estertores crepitantes en la base derecha y
una hepatomegalia a 4 cm del reborde costal. En las
pruebas complementarias efectuadas, destaca Hb de
14.8 mg/dL, leucocitosis (17.000/L) con desviacin
izquierda, GOT (AST): 106, GPT (ALT): 82. En la Rx de
trax se aprecia un infiltrado intersticial en lbulo inferior
derecho. Su diagnstico de presuncin debe ser:
A) Neumona neumoccica.
B) Neumonitis por hipersensibilidad.
C) Fiebre Q.
D) Legionella.
E) Tuberculosis.
68. Varn de 37 aos que viene presentando durante
los tres ltimos meses astenia, esputos hemoptoicos y
disnea progresiva hasta hacerse de mnimos esfuerzos,
con intolerancia al ejercicio. Salvo una TA de 90/60, los
datos exploratorios son anodinos. Sin embargo, los
estudios complementarios nos sorprenden: pH: 7.37,
PaO2: 62, PaCO2: 37, HCO3: 27. Hb: 9.2 y creatinina
de 2.3. La Rx de trax muestra infiltrados difusos
parahiliares bilaterales. Ante los hallazgos reseados,
se aade la peticin de un sedimento de orina, que
muestra microhematuria y proteinuria. De las siguientes,
qu prueba diagnstica le parece prioritaria en la
evaluacin del enfermo?:
A) Test de difusin (DLCO).
B) Examen citolgico (esputo o lavado broncoalveolar).
C) Anticuerpos anti membrana basal glomerular, cANCA.
D) Biopsia pulmonar.
E) Biopsia renal.
69. En el caso anterior, la capacidad de difusin del CO
est aumentada y el ttulo de anticuerpos antimembrana
basal glomerular es de 1:128. Cul le parece el
diagnstico ms probable?:
A) Granulomatosis de Wegener.
B) Sndrome de Goodpasture.
C) Tuberculosis.
D) Tromboembolismo pulmonar.
E) Granulomatosis de Churg-Strauss.
70. Mujer de 78 aos que en el curso de un
postoperatorio por fractura de cadera comienza con un
cuadro brusco de disnea y febrcula. Exploracin:
taquipnea a 30 r.p.m., taquicardia a 130 l.p.m., refuerzo
del segundo tono, abolicin del murmullo vesicular en
base de pulmn derecho y extremidades sin edemas, no
dolorosas, sin signos flogticos. Complementarios: GAB:
pH: 7.52, PaO2: 56, PaCO2: 30, HCO3: 25. 13.000
leucocitos con desviacin izquierda.
Rx de trax: pinzamiento del seno costodiafragmtico
derecho. ECG: Taquicardia sinusal con bloqueo

Examen Simulacro :: Ciclo de Revisin en Medicina :: Sbado 4 de Agosto del 2012 :: www.estudiosmyc.com
incompleto de rama derecha. Seale la actitud ms
adecuada:
A) Diurticos.
B) Toracocentesis.
C) Antibiticos.
D) Corticoides.
E) Heparina.
71. Mujer de 37 aos, fumadora, que consulta por
presentar durante el ltimo mes fiebre, malestar general,
artralgias y una erupcin cutnea dolorosa en ambas
piernas. Complementarios. Rx de trax: adenopatas
hiliares bilaterales, sin afectacin del parnquima
pulmonar. Mantoux negativo. Se realiz una FBC con
lavado broncoalveolar (LBA). Lquido del LBA: 22% de
linfocitos con cociente CD4/CD8 de 5.2. El diagnstico
ms probable es:
A) Tuberculosis.
B) Sarcoidosis.
C) Linfoma.
D) Cncer de pulmn .
E) Asbestosis.
72. Varn de 47 aos que acude a urgencias por fiebre,
tos, artralgias y rinorrea purulenta con ulceraciones de la
mucosa nasal de dos semanas de evolucin. Inici
tratamiento antibitico 7 das antes, al ser diagnosticado
por su mdico de cabecera de sinisitis (opacificacin de
ambos senos maxilares), sin obtener una mejora
clnica. La Rx de trax presenta mltiples ndulos
pulmonares bilaterales, algunos de ellos cavitados. En
los anlisis efectuados destaca un sedimento de orina
con 8 hemates por campo con algn cilindro
eritrocitario. La biopsia de la mucosa nasal mostr
inflamacin granulomatosa con necrosis. El diagnstico
ms probable es:
A) Granulomatosis de Wegener.
B) Granulomatosis de Churg-Strauss.
C) Cncer de cavum con metstasis pulmonares.
D) Granulomatosis linfomatoide.
E) Tuberculosis.
73. Un grave problema de las unidades de cuidados
intensivos son las infecciones (neumonas y sepsis) por
grmenes
gramnegativos
multirresistentes
(pseudomonas, serratias, citrobacter, morganella,
acinetobacter, etc.). Ya se han identificado previamente
cepas multirresistentes en nuestra UCI. A falta de un
antibiograma, cul sera el tratamiento emprico de
eleccin?
A) Ceftazidima, amicamicina y vancomicina.
B) Ceftriaxona y tobramicina.
C) Imipenem o ciprofloxacino.
D) Imipenem y amikamicina.
E) Esperar hasta los resultados del antibiograma.

74. Un varn de 30 aos, fumador de 20 cigarrillos/da


desde los 20 a los 25 aos y ex fumador desde
entonces, presenta, en un reconocimiento laboral, un
ndulo pulmonar solitario (NPS) de unos 2 cm. de
diametro en la periferia del LSD. La Rx de trax muestra
un mediastino normal y no permite identificar
calcificaciones en el NPS. El paciente se encuentra
asintomtico y niega la posibilidad de recuperar
radiografas anteriores antes de 6 meses (por cambio de
domicilio). Qu actitud mantendra ante este enfermo?
A) Informar de la baja probabilidad de malignidad y Rx
de trax en 3 meses.
B) Fibrobroncoscopia.
C) Realizacin de una TAC torcica.
D) Realizacin preferente de una PAAF con control de
TAC.
E) Insistir en la recuperacin de las Rx previas y nueva
cita en la consulta entonces (6 meses).
75. En el caso anterior, la TAC no muestra nuevos datos
(confirma la ausencia de calcificaciones, no adenopatas
ni afectacin mediastnica y no existe afectacin
pleural). La familia del enfermo ha localizado las Rx de
trax previas (14 meses antes) en la que se identifica el
mismo NPS con un dimetro de 1,4 cm. Cul sera la
actitud ms adecuada?
A) Actitud expectante y repetir pruebas de imagen en 2
meses.
B) Fibrobroncoscopia con citologa en las muestras
obtenidas.
C) Fibrobroncoscopia y biopsia transbronquial (BTB).
D) PAAF con control de TAC.
E) Toracotoma.
76. En caso de que la PAAF obtenga material suficiente
y el resultado sea de malignidad, qu tipo histolgico le
parece el ms probable?
A) Ca. epidermoide.
B) Adenocarcinoma.
C) Ca clulas pequeas (CCP).
D) Ca. bronquioalveolar.
E) Carcinoide.
77. Paciente de 36 aos con amenorrea de 10 semanas.
Tiene un antecedente de infertilidad por factor tubrico.
Refiere episodios de dolor clico hipogstrico desde
hace aproximadamente un mes. El test de embarazo en
orina es positivo.
La prueba diagnstica que solicita a continuacin es:
A) Amniocentesis precoz.
B) Ecografa.
C) Laparoscopia.
D) Triple screening.
E) Biopsia de vellosidades corinicas.

Examen Simulacro :: Ciclo de Revisin en Medicina :: Sbado 4 de Agosto del 2012 :: www.estudiosmyc.com
78. Gestante de 16 semanas sin antecedentes de
inters que presenta los siguientes resultados en la
analtica de triple screening: alfafetoprotena 0,3 MM
(disminuido), betahcg 1,7 MM (normal), riesgo estimado
de T21 1/43. A continuacin se le realiza:
A) Ecografa.
B) Amniocentesis.
C) Biopsia de vellosidades corinicas.
D) Funiculocentesis.
E) Fetoscopia.
79. Gestante de 26 semanas que consulta por fiebre de
39 C y dolor lumbar unilateral. La analtica de sangre
presenta 16.000 leucocitos y desviacin izquierda. El
tratamiento indicado es:
A) Abundante ingesta de lquidos.
B) Analgsicos orales y abundante ingesta de lquidos.
C) Analgsicos endovenosos y forzar diuresis.
D) Antibiticos orales y reposo domiciliario.
E) Antibiticos endovenosos intrahospitalarios.
80. Gestante de 30 semanas con aumento excesivo de
peso (ganancia de 18 kg. hasta la actualidad) a la que
se realiza un test de OSullivan que resulta patolgico.
La actitud mdica ha de ser a continuacin:
A) Vigilancia fetal estricta, con registros semanales de la
frecuencia cardaca fetal.
B) Controles de glucemia capilar (BMtest) en desayuno,
comida y cena.
C) Ecografas seriadas para diagnosticar a la mayor
brevedad posible un hidramnios o macrosoma fetal.
D) Confirmar el diagnstico mediante una prueba de
tolerancia oral a la glucosa.
E) Tratar con insulina rpida segn los resultados del
test de OSullivan.
81. Gestante de 9 semanas que consulta por
metrorragia menor que una regla y dolor abdominal.
Todava no ha acudido a ninguna visita de control por su
toclogo. En la exploracin se obseva un tero de
aproximadamente 8 semanas de gestacin, abdomen
blando y depresible y crvix permeable a un dedo. Poco
despus
la
paciente
empieza
a
sangrar
abundantemente, mucho ms que una regla. El
tratamiento indicado es:
A) Ingreso y observacin.
B) Laparoscopia.
C) Legrado.
D) Laparotoma.
E) Venoclisis de oxitocina.
82. Primigesta de 27 aos, sin antecedentes mdicos de
inters, que consulta por metrorragia insidiosa y
recurrente en semana 36 de embarazo. No presenta
dolor abdominal, el tero est relajado y el latido fetal se
escucha vigoroso.
El diagnstico ms probable ser:
A) Vasa praevia.
B) Desprendimiento prematuro de placenta.
C) Expulsin del tapn mucoso.

D) Placenta previa.
E) Prdida de lquido amnitico hemtico.
83. Tercigesta isoinmunizada que presenta test de
Coombs indirecto de 1/10 en semana 30 de embarazo.
Se realiza un amniocentesis en que se determina la
madurez fetal (se confirma la presencia de fosfatidil
glicerol en lquido amnitico) y el nomograma de Liley,
que se encuentra en la zona II. La actitud indicada es:
A) Nueva amniocentesis en una semana.
B) Seguimiento ecogrfico con la paciente hospitalizada.
C) Administracin de corticoides para inducir la madurez
fetal.
D) Extraccin fetal.
E) Administracin endovenosa materna de IgG contra
Ac anti D.
84. Gestante de 33 semanas que consulta por dinmica
uterina, disminucin de movimientos fetales y febrcula.
Al ingreso presenta una analtica de sangre con
leucocitosis moderada y el resto de los parmetros
normales. En la
ecografa se observa un oligoamnios y un perfil biofsico
fetal de 9. El tratamiento consiste en:
A) Antibiticos endovenosos.
B) Antibiticos intracavitarios.
C) Induccin del parto.
D) Cultivo del lquido amnitico y tratamiento segn
antibiograma.
E) Administracin de corticoides para favorecer la
madurez fetal e induccin del parto en semana 36.
85. Gestante de embarazo gemelar que presenta
dinmica espontnea en semana 36. Ambos gemelos se
encuentran en presentacin ceflica. El parto del
primero se produce sin complicaciones, pero el segundo
se encuentra en posicin occipito iliaca derecha
transversa al cabo de 25 minutos desde el nacimiento
del primero, sin progresar desde un III plano de Hodge.
Para finalizar el parto est indicada la realizacin de:
A) Vacuum.
B) Frceps.
C) Cesarea.
D) Maniobra de Kristeller.
E) Cualquiera de las anteriores segn el estado fetal.
86. Primigesta de 36 aos que presenta un aumento de
la tensin diastlica de 30 mmHg respecto a tomas
iniciales, y albuminuria con edemas generalizados.
Actualmente se encuentra en la 34 semana de
embarazo. Sbitamente inicia un cuadro de cefalea y
transtornos visuales. Vd. le inicia tratamiento mdico
con:
A) Nifedipina.
B) Dihidralacina.
C) Alfametildopa.
D) Sulfato de magnesio.
E) Labetalol.

Examen Simulacro :: Ciclo de Revisin en Medicina :: Sbado 4 de Agosto del 2012 :: www.estudiosmyc.com
87. En la paciente del caso anterior el estudio fetal
descubre un feto afecto de crecimiento intrauterino
retardado, en el que la relacin entre el rea ceflica y el
rea abdominal es mayor de uno. El feto se encuentra
en situacin ceflica.
La conducta obsttrica adecuada es:
A) Expectante, con parto vaginal como via de eleccin.
B) Controles de bienestar fetal (perfil biofsico)
semanales hasta la semana 40.
C) Valoracin de la madurez fetal e induccin del parto,
si el feto es maduro.
D) Cesrea inmediata.
E) Evaluacin de la funcionalidad placentaria mediante
ecografa Doppler.
88. Purpera que acaba de parir mediante parto
eutcico un varn de 3,450 kg. a los 30 minutos no ha
alumbrado, por lo que se realiza una maniobra de Cred
para extraer la placenta. Transcurridos unos minutos
presenta un cuadro de disnea y hemorragia profusa. El
tero est bien contrado, pero el sangrado no cesa. El
diagnstico ms probable es:
A) Ruptura uterina.
B) Desgarro de crvix.
C) Hipotona uterina.
D) Retencin de restos placentarios.
E) Coagulacin intravascular diseminada.
89. Un paciente de 26 aos de edad, adicto a drogas
por va parenteral, consulta por malestar general, fiebre
de 39con tiritona, escalofros y dolor y tumefaccin en
rodilla derecha. En la exploracin llama la atencin un
soplo cardco panfocal que previamente no estaba en la
historia del enfermo y artritis de rodilla derecha. No se
pudo realizar artrocentesis diagnstica. La cobertura
emprica antibitica ms segura sera:
A) Vancomicina.
B) Vancomicina y Gentamicina.
C) Ciprofloxacina.
D) Eritromicina y cefuloxima.
E) Ceftriaxona.
90. Un joven de 18 aos sin antecedentes de inters,
consulta por tos, fiebre y otalgia. En la Rx de trax
presenta infiltrado intersticial derecho. La exploracin
ORL demuestra miringitis ampollosa. Iniciaramos
tratamiento con:
A) Ceftriaxona.
B) Vancomicina.
C) Norfloxacina.
D) Isoniazida, rifampicina y pirazinamida.
E) Eritromicina.
91. Un paciente al que le detecta una infeccin urinaria
por Pseudomona inicia tratamiento con ceftacidima.
Tras 2 das de tratamiento el paciente comienza a
encontrarse peor, la fiebre aumenta hasta los 39 y
desarrolla hipotensin. En el hemograma destaca
leucopenia e importante trombopenia que previamente

no estaban. Bioqumicamente presenta hiperglucemia


de 198 g./dl. Probablemente nos encontramos ante:
A) Efecto secundario de la ceftacidima.
B) Evolucin natural del proceso.
C) Situacin de shock sptico.
D) Probable asociacin de una hemopata.
E) Cetoacidosis diabtica.
92. Un varn de 27 aos consulta por presenta en la
regin balanoprepucial una lcera de 1 cm. de tamao
de borde indurado, no doloroso y que secreta serosidad.
Presenta
adicionalmente
adenopatas
inguinales
bilaterales y fiebre. Estaramos obligados a solicitarle en
el estudio:
A) Aglutinaciones a salmonella.
B) Aglutinaciones a brucella.
C) Serologa de VIH.
D) Serologa lutica.
E) Serologa de micoplasma.
93. Una paciente de 19 aos ha sido diagnosticada de
mononucleosis infecciosa, confirmada mediante PaulBunnel. Ha sido tratada con paracetamol a dosis de 2,5
g./da. A los cinco das del diagnstico comienza de
forma brusca con un cuadro de abdomen agudo y
shock. El proceso ms probable es:
A) Rotura espontnea del bazo.
B) Hepatitis por virus de Epstein-Barr.
C) Complicacin infecciosa intraabdominal.
D) Complicacin del tratamiento.
E) Otra cualquier causa de abdomen agudo: p. ej.
apendicitis aguda.
94. Un lactante de 7 meses de edad, con Tetraloga de
Fallot intervenida, desarrolla un cuadro de dificultad
respiratoria, sibilancias y tos. No se termometra fiebre.
Adems de las medidas de soporte, el tratamiento de
eleccin sera:
A) Reintervencin quirrgica de su cardiopata.
B) Esteroides a dosis plenas.
C) Ribavirina en aerosol.
D) Eritromicina intravenosa.
E) Ceftriaxona intravenosa.
95. Una paciente de 49 aos consulta en un Servicio de
Urgencias por dolor, enrojecimiento y tumefaccin de la
parte distal de su miembro inferior derecho. Por
sospecha de trombosis venosa profunda se le realiza
flebografa que resulta ser negativa. La actitud
teraputica a seguir es:
A) Tratamiento antiinflamatorio.
B) Heparina de bajo peso molecular.
C) Cobertura emprica antibitica con Oxacilina.
D) Reposo del miembro sin ms.
E) Repetir flebografa pasadas 48 horas.

Examen Simulacro :: Ciclo de Revisin en Medicina :: Sbado 4 de Agosto del 2012 :: www.estudiosmyc.com
96. Un adicto a drogas por va parenteral consulta por
tumoracin fluctuante y con signos inflamatorios en la
flexura del codo donde ha realizado inyecciones
intravenosas. Se debe realizar:
A) Tratamiento quirrgico de drenaje.
B) Cobertura con ceftriaxona.
C) Cobertura con vancomicina.
D) Cobertura con Oxacilina.
E) Medidas antiinflamatorias.

97. Una pareja de turistas que han regresado a Lima,


luego de estar en la Selva Central del Per, comienzan
a presentar deposiciones en cantidad abundante de
caractersticas lquidas, similares al agua, en nmero de
20-25 al da. No se acompaa de dolor abdominal ni
fiebre. El cuadro obliga a una rehidratacin y sta se
consigue con dificultad dada la gran prdida de agua y
electrlitos. El cuadro clnico ms probable es:
A) Gastroenteritis por Salmonella.
B) Disentera bacilar.
C) Disentera amebiana.
D) Clera.
E) Gastroenteritis viral.
98. Un enfermo leucmico, muy inmunodeprimido,
desarrolla tras un tratamiento antibacteriano de amplio
espectro, cuadro importante de insuficiencia respiratoria
y fiebre. En la Rx. de trax se aprecia una masa densa,
cubierta por un menisco delgado de aire en el interior de
una cavidad. El diagnstico ms probable es:
A) Tuberculosis.
B) Neumona bacteriana.
C) Masa tumoral sobreinfectada.
D) Neumona por cndidas.
E) Aspergiloma.
99. Un nio de 3 aos desarrolla un cuadro de infeccin
respiratoria de vas altas con fiebre elevada.
Posteriormente aparece tos paroxstica con gallo
inspiratorio. El tratamiento de eleccin sera:
A) Ceftriaxona.
B) Cefonicid.
C) Cefalotina.
D) Eritromicina.
E) Amoxicilina-clavulnico.
100. Un paciente de 56 aos acude a su mdico de
cabecera por presentar en el ltimo mes fiebre diaria,
con una distribucin de dos picos, matutino y vespertino.
A
la
exploracin
llama
la
atencin
una
hepatoesplenomegalia muy importante. No se objetivan
adenopatas a ningn nivel. El hemograma muestra
pancitopenia. La prueba diagnstica que se debera
realizar sera:
A) Estudio de mdula sea.
B) Ecografa abdominal.
C) Marcadores tumorales.
D) Biopsia heptica.
E) Marcadores de hepatitis.

Examen Simulacro :: Ciclo de Revisin en Medicina :: Sbado 4 de Agosto del 2012 :: www.estudiosmyc.com

CLAVE DE RESPUESTAS TEMA A


PREGUNTA N 1
RESPUESTA: D
Se trata de un caso clnico, laboratorial y radiolgico tpico de pancreatitis aguda. Luego de iniciar el tratamiento con dieta absoluta,
fluidoterapia y analgesia. Es beneficioso el tratamiento con Carbapenems (imipenem) por tratarse de pancreatitis Baltazar E
(Coleccin), esto ha demostrado ser capaz de disminuir la incidencia de sepsis y de mortalidad (respuesta D correcta).
PREGUNTA N 2
RESPUESTA: D
Nos preguntan por el principal factor responsable de la no cicatrizacin. Las opciones A, B y C son causas de gastritis por s i mismas y
los hbitos dietticos no se han relacionado con la patogenia de la lcera. El tabaco aumenta la incidencia de la lcera duodenal,
empeora la cicatrizacin de las lceras (opcin D correcta) e incrementa el riesgo de complicaciones y la necesidad de ciruga. Los
mecanismos por los que acta son: aumento del vaciamiento gstrico, disminucin de la secrecin pancrtica de bicarbonato,
alteracin del flujo sanguneo o disminucin de la sntesis de prostaglandinas.
PREGUNTA N 3
RESPUESTA: D
El tratamiento debe comenzarse si la cifra de PMN del lquido es superior a 250/mm3. Los frmacos ms utilizados de forma emprica
son las cefalosporinas de tercera generacin. La duracin del tratamiento es de 7 a 10 das, existiendo trabajos que respaldan los
tratamientos de 5 das como eficaces (respuesta D falsa).
PREGUNTA N 4
RESPUESTA: D
Ante un paciente anciano varn con anemia ferropnica crnica debemos sospechar un cncer de colon, aqu adems nos dan el dato
de la sangre oculta en heces, que es el test de screening en mayores de 50 aos. La actuacin es la misma, realizar una colonoscopia
completa (puesto slo el 50% de los cnceres estn al alcance del sigmoidoscopio) para buscar la lesin causante de la hemorragia
crnica que le hace al paciente perder hierro. Esta prueba es el mtodo de diagnstico ms sensible y siempre debe hacerse ante la
sospecha de un cncer de colon.

Examen Simulacro :: Ciclo de Revisin en Medicina :: Sbado 4 de Agosto del 2012 :: www.estudiosmyc.com
PREGUNTA N 5
RESPUESTA: D
Para llegar a la respuesta correcta, hay que conocer la diferencia entre una revisin narrativa y una sistemtica. En la revisin
narrativa, el autor revisa bibliografa sobre un tema y compara los resultados, pero el resultado obtenido depende en buena medida de
la voluntad del revisor, en funcin de qu artculos se incluyan (respuestas A, B y C incorrectas). En la revisin sistemtic a, los
artculos se incluyen o no en funcin de criterios prefijados; es la aplicacin del mtodo cientfico a la revisin bibliogrfica. El
metaanlisis es una revisin sistemtica en la que adems los datos se analizan estadsticamente, combinando los diferentes
artculos; no todas las revisiones sistemticas son metaanlisis (respuesta E incorrecta, D correcta).
PREGUNTA N 6
RESPUESTA: C
La aleatorizacin simple nos asegura una misma probabilidad de estar en un grupo u otro, con independencia del nmero de pacientes
que ya haya en cada grupo y de las caractersticas de base. La aleatorizacin por bloques nos evita la desigualdad numrica, a costa
de dejar predeterminada la asignacin del ltimo elemento del bloque. La aleatorizacin estratificada busca homogeneizar la
distribucin de una variable de inters; para ello, la muestra se divide en grupos homogneos respecto a la variable, y posteriormente
se aleatoriza segn nuestras necesidades (respuesta C correcta). Cabe recordar que la aleatorizacin no asegura una homogeneidad
de los grupos, slo tiende a ella, por eso es necesario en la fase de anlisis verificar que la aleatorizacin ha sido eficaz.
PREGUNTA N 7
RESPUESTA: B
El anlisis en el cual se compara el coste en unidades monetarias con resultados que se miden en unidades naturales o fsicas o
unidades de efectividad (aos de vida ganados, incidencia prevenida, Incapacidad evitada) se denomina anlisis de
COSTE/EFECTIVIDAD (respuesta B correcta).
PREGUNTA N 8
RESPUESTA: B
La mejor forma en epidemiologa para corroborar una hiptesis etiolgica es un estudio experimental (respuesta B correcta), y si no,
pues un estudio analtico observacional; los descriptivos slo generan hiptesis causales. Si nos preguntan de los estudios
experimentales, el mejor de ellos es el ensayo clnico controlado y aleatorizado. Si nos preguntaran de los estudios observacionales, el
mejor es el de cohortes prospectivas.
PREGUNTA N 9
RESPUESTA: E
Son situaciones asociadas a mayor riesgo de cncer colorrectal: endocarditis por S. bovis (se asocia, pero no es causa),
ureterosigmoidostoma, tabaquismo de larga evolucin, obesidad, dieta rica en grasas e hipercalrica, enfermedad inflamatoria
intestinal. Por lo tanto la opcin falsa es la alternativa E. De hecho puede que el consumo de aspirina disminuya el riesgo de cncer de
colon.
PREGUNTA N 10
RESPUESTA: B
El ensayo clnico es la herramienta ms potente para obtener conclusiones. Es un estudio analtico experimental, en el que el paciente,
una vez que firma el consentimiento informado, es asignado a un grupo u otro de tratamiento no por el criterio del experimentador sino
por un proceso matemtico llamado aleatorizacin, que intenta que ambos grupos sean homogneos en cuanto a caractersticas
basales (respuesta B falsa). Dichos experimentos suelen ser ciegos, con enmascaramiento de la medicacin para evitar sesgos de
clasificacin, pero eso no es estrictamente indispensable.
PREGUNTA N 11
RESPUESTA: B
Las principales indicaciones de vacunacin antineumoccica son: personas mayores de 65 aos, pacientes con insuficiencia cardaca
crnica, infeccin por VIH avanzada, fstula crnica de LCR, alcohlicos. Es importante recordar tambin su indicacin en pacientes
esplenectomizados. Por lo tanto en esta pregunta la opcin que no recoge una indicacin aceptada es la B.
PREGUNTA N 12
RESPUESTA: E
Tras la esplenectoma existe una inmunodepresin frente a grmenes encapsulados, por ello ya sabemos que las opciones A, B y C
estn efectivamente indicadas. Por otra parte tambin se administra la vacuna contra la gripe ya que nos encontramos ante un
enfermo crnico con cierta inmunodepresin y adems sabemos que la complicacin ms frecuente de la gripe es la neumona, sobre
todo la causada por estos grmenes encapsulados. Por lo tanto la vacuna que no est indicada es la opcin E, la hepatitis A.
PREGUNTA N 13
RESPUESTA: E
En este caso nos plantean el diagnostico diferencial de la disminucin de la agudeza visual (AV). En primer lugar nos dicen que la
prdida es lenta y progresiva, e indolora lo que nos permite descartar el glaucoma agudo y la papilitis (respuestas C y D). Como nos
preguntan que es lo ms probable, descartamos directamente la distrofia corneal ya que no son muy frecuentes (respuesta B). Cabe
pensar que el error de refraccin, el paciente lo habra tenido toda su vida, no le va a aparecer a los 65 aos (respuesta A). Por tanto
la opcin correcta es la degeneracin macular asociada a la edad (Alternativa E)
PREGUNTA N 14
RESPUESTA: A
Se plantea una disminucin brusca de la visin, con lo que se descarta el glaucoma crnico simple, la catarata y la atrofia ptica
(respuestas C, D y E) y con la superficie ocular normal, con lo que descartamos el edema corneal (respuesta B). Adems debemos
recordar que la hemorragia vtrea es una causa frecuente de prdida de agudeza visual en la retinopata diabtica.
PREGUNTA N 15
RESPUESTA: A
En la uvetis anterior aguda la pupila se encuentra en MIOSIS, mientras que en el glaucoma agudo encontramos MIDRIASIS. El resto
de opciones nos completan la clnica tpica de la uvetis anterior.

Examen Simulacro :: Ciclo de Revisin en Medicina :: Sbado 4 de Agosto del 2012 :: www.estudiosmyc.com
PREGUNTA N 16
RESPUESTA: D
En este caso nos estn preguntando por el tratamiento de una obstruccin intestinal por un cncer en fase terminal, es decir, nos
estn preguntando por un tratamiento paliativo y no por uno curativo. De todas las opciones, la nica que cumple estas caractersticas
es la opcin D. La morfina es un opiceo de referencia para el tratamiento del dolor del paciente terminal, debido a su gran potencia
analgsica y su alta afinidad y actividad por los receptores mu, careciendo, prcticamente de techo analgsico. La buscapina
(escopolamina) es una anticolinrgico que se utiliza como espasmoltico con la intencin de disminuir el dolor producido por la
contraccin refleja de la musculatura lisa del intestino que aparece en la obstruccin intestinal. El haloperidol es un neurolptico que
acta bloqueando el sistema dopaminrgico y que se usa, fundamentalmente, por su efecto sedante.
PREGUNTA N 17
RESPUESTA: B
Es una pregunta directa en la que hay que conocer que la meperidina (respuesta B) es un agonista opiceo no recomendado en el
tratamiento del dolor crnico de cualquier etiologa por varias razones. Tiene un metabolito con la mitad de potencia analgsica y una
vida media de 15-20 horas, tras administraciones repetidas puede acumularse y producir hiperexcitabilidad del SNC con alteraciones
del humor, temblor, mioclonas e incluso convulsiones. El resto de opciones son agonistas opiceos que s se usan en le trata miento
del dolor crnico, concretamente la metadona, la morfina y el fentanilo son de alta potencia, mientras que el tramadol es un opiceo
dbil.
PREGUNTA N 18
RESPUESTA: D
La traqueotoma es una maniobra quirrgica indicada en casos de obstruccin respiratoria alta (abscesos, tumores...), pacientes
intubados en los que se espera mantener la ventilacin artificial durante ms de 48-72 horas, y en pacientes con retencin o riesgo de
aspiracin de secreciones. Se ha de realizar siempre entre el 2 y 3 anillos traqueales (respuesta D correcta) y nunca en el 1
(respuesta C falsa) por el alto riesgo de estenosis traqueales residuales (complicacin tarda fundamental, la precoz ms frecuente es
la hemorragia). En la membrana cricotiroidea (opcin A falsa) es donde se realizan las coniotomas, que slo se han de realizar en
caso de extrema urgencia en el medio extrahospitalario.
PREGUNTA N 19
RESPUESTA: C
La opcin A es correcta, ya que es ms frecuente en nios (por la hipertrofia adenoidea), especialmente si tienen malformaciones
velopalatinas, as como en el sndrome de Down. La opcin B es correcta, ya que toda otitis media secretora unilateral persistente en
el adulto obliga a descartar un cncer de cavum (el 25 % de estos cnceres tienen esta forma de presentacin). La opcin D puede
plantearnos algunas dudas, pero est descrito que entre 2/3 y 3/4 de los cultivos de muestras obtenidas por miringocentesis dan
positivo. La opcin E tambin es correcta, ya que el aspecto tpico en la otoscopia es un tmpano ntegro, retrado y opaco, con niveles
hidroareos y burbujas en la caja. La opcin C es la falsa, ya que el dato epidemiolgico que nos presenta es el caracterstico de la
otitis del nadador (otitis externa difusa bacteriana).
PREGUNTA N 20
RESPUESTA: B
Esta situacin requiere sin duda alguna una actitud teraputica inmediata, por lo que descartamos la opcin 1 (actitud expectante) y
las opciones D y E, en las que tanto el hierro IV como la EPO requeriran das para comenzar a mejorar la cifra de hemoglobina. La
respuesta correcta es la B (transfundir hemates) porque en el enunciado no se nos dice que la paciente tenga hipotensin o shock
hipovolmico por hemorragia, en cuyo caso s estara indicado transfundir sangre total (opcin C).
PREGUNTA N 21
RESPUESTA: A
Los cuerpos de Howell-Jolly son inclusiones redondeadas, densas y en general nicas, debidas a fragmentos de cromosomas
procedentes de mitosis eritroblsticas anmalas. Se observan en pacientes esplenectomizados, en el hiposplenismo, en el saturnismo
y en las anemias megaloblsticas y refractarias. Por tanto la respuesta correcta es la A, asplenia. Para el mir se deben conocer otros
datos morfolgicos del estudio de extensin de la sangre perifrica. En el dficit de G6PDH son los cuerpos de Heinz, en los procesos
hemolticos aparecen los esferocitos.
PREGUNTA N 22
RESPUESTA: E
La leucemia mieloide crnica est claramente relacionada con un marcador citogentico, el cromosoma Philadelphia, que aparece en
cerca del 95% de los casos. Dicho cromosoma consiste en una translocacin del material gentico entre los cromosomas 9 y 22. Dicha
translocacin cromosmica da lugar a la unin del oncogn abl del cromosoma 9 con el oncogn bcr del 22, originando un hbrido
anormal bcr/abl, que es el causante de la enfermedad. De todos modos el cromosoma Philadelphia no es exclusivo de las clulas
precursoras de la serie blanca, sino que tambin de la roja y megacariocitos, aunque es exclusivo de las clulas hematolgica s.
Aunque el cromosoma Philadelphia aparece en el 95%, lo caracterstico es la traslocacin, puesto que es el causante de la sntesis
anmala y por tanto la respuesta correcta es la E.
PREGUNTA N 23
RESPUESTA: A
En este paciente, el diagnstico ms factible es el herpes genital, muy probablemente una primoinfeccin, dada la repercusin
sistmica de la enfermedad. En las recurrencias, no suele haber fiebre ni malestar general. La presencia de las lesiones vesiculosas
es muy sugerente de esta infeccin. Estas vesculas suelen ulcerarse ms tarde; de hecho, la causa infecciosa ms frecuente de
lceras genitales es el virus herpes simple. La opcin B es claramente falsa, pues de un secundarismo lutico no cabra esper ar la
aparicin de lesiones de este tipo. Debes asociar a la sfilis secundaria manifestaciones como la rosola, la lengua en pradera segada
y los condilomas planos. La primoinfeccin por VIH puede ser asintomtica, pero suele cursar como un sndrome mononuclesico, con
un rash maculopapular (opcin C falsa). El citomegalovirus puede transmitirse por va sexual, pero no se incluye en el diagnstico
diferencial de los chancros (opcin D falsa). Por ltimo, el condiloma acuminado es una infeccin por papilomavirus que produce unas
lesiones de aspecto sobreelevado y verrucoso (opcin E falsa).

Examen Simulacro :: Ciclo de Revisin en Medicina :: Sbado 4 de Agosto del 2012 :: www.estudiosmyc.com
PREGUNTA N 24
RESPUESTA: D
Las infecciones bacterianas son una importante causa de muerte en pacientes con infeccin por VIH. El neumococo y el H. influenzae
representan dos de las infecciones bacterianas ms frecuentes en este tipo de pacientes (respuesta C correcta). Los enfermos con
infeccin por VIH muestran un incremento de seis veces en la incidencia de neumona neumoccica, y de 100 veces en la de
bacteriemia por neumococo (respuestas A y B correctas). Todos los pacientes con infeccin por VIH deben recibir la vacuna
antineumoccica desde el momento en que se diagnostica; no hay que esperar a que tengan un grado determinado de
inmunodepresin (respuesta D falsa). Los pacientes VIH positivos, cuando sobrepasan determinado nivel de inmunodepresin,
comienzan a sufrir infecciones por microrganismos oportunistas. Debes conocer muy bien a partir de qu nivel de linfocitos CD4
comienzan a ser ms probables los distintos grmenes, lo que te facilitar la resolucin de muchos casos clnicos. Por ejemplo, en el
caso del criptococo, el nivel de CD4 suele oscilar en torno a 100 o menos. Sin embargo, otras infecciones no precisan un grado tan
intenso de inmunodepresin, pudiendo aparecer con el sistema inmunolgico relativamente intacto, como es el caso del neumococ o
(respuesta E correcta). Aprovechamos la ocasin para recordarte las complicaciones clnicas que definen la categora C en la infeccin
por VIH, entre las cuales se encuentra el presentar dos o ms episodios de neumona en un ao.
PREGUNTA N 25
RESPUESTA: C
En muchas ocasiones, pueden interrelacionarse, por la especial susceptibilidad de los pacientes VIH positivos a ser infectados por
micobacterias. Este tipo de enfermos suelen padecer formas ms graves de tuberculosis que los inmunocompetentes, debido
precisamente a la incompetencia de su inmunidad celular (opcin A falsa, C correcta). No es necesario un grado de inmunosupresin
muy severo para que el paciente VIH sufra una tuberculosis (respuesta B falsa). El lmite de 50 CD4/mL es donde suelen estar los
pacientes con SIDA que son infectados por micobacterias atpicas. Desde el punto de vista del diagnstico, los mtodos
microbiolgicos para el diagnstico de tuberculosis no tienen por qu tener un menor rendimiento que en pacientes
inmunocompetentes. De hecho, el nmero de bacilos en las muestras puede incluso ser mayor en el paciente con SIDA, por el
deterioro de sus defensas (opcin D falsa). En cuanto al tratamiento, la respuesta al mismo de los pacientes VIH positivos es la misma
que en inmunocompetentes, es decir, suele ser satisfactorio (opcin E falsa).
PREGUNTA N 26
RESPUESTA: C
Debes recordar que es una enfermedad producida por toxinas del Staphylococcus aureus, la clnica que produce y su asociacin al
uso de tampones intravaginales. Centrndonos en la pregunta, la respuesta falsa es la C, puesto que este cuadro se debe a la
diseminacin hematgena de toxinas (TSST-1), no de la bacteria en s. Clnicamente, el cuadro tpico consiste en la aparicin de
fiebre, exantema con posterior descamacin (tpicamente palmoplantar), hipotensin y afectacin de al menos tres sistemas orgnicos
(digestivo, muscular, renal, heptico, trombopenia o sistema nervioso), en ausencia de otro diagnstico alternativo. El tratamiento debe
hacerse con antibiticos antiestafiloccicos (Oxacilina o, si es resistente, vancomicina) y mantenimiento hemodinmico del paciente.
PREGUNTA N 27
RESPUESTA: E
El diagnstico diferencial entre una ITU alta y una baja es fundamentalmente clnico. Una cistitis aguda suele cursar con ten esmo
vesical, aumento de la frecuencia miccional y escozor al orinar, a veces con hematuria terminal. Puede haber dolor hipogstrico y, ms
raramente, febrcula. En cambio, la pielonefritis se presenta como un cuadro febril, con hipersensibilidad de la fosa lumbar, fiebre,
nuseas, vmitos y puo percusin positiva en la exploracin fsica. De las cuatro primeras opciones que nos presenta la pregunta, no
podramos emplear ninguna para distinguir un cuadro de otro. La diferencia fundamental es la opcin E, pues en una cistitis es
rarsima la presencia de fiebre, mientras que en una pielonefritis aguda sera excepcional que no apareciese.
PREGUNTA N 28
RESPUESTA: A
Existen dos tipos de pruebas serolgicas, treponmicas (FTA-Abs, TPHA) y no treponmicas. (VDRL, RPR). Las primeras en
positivizarse son las treponmicas, persistiendo positivas de por vida. En cambio, en las no treponmicas puede observarse una
disminucin del ttulo de anticuerpos cuando se instaura un tratamiento correcto. En el caso que nos presentan, para considerar que la
lesin ulcerada es consecuencia de una sfilis, debera tener la prueba treponmica (FTA-Abs) positiva. Al ser negativo, debes pensar
en otra causa. La positividad del VDRL tiene un valor muy relativo, porque es una prueba poco especfica, pudiendo positivizarse por
circunstancias muy distintas a la sfilis (lupus eritematoso, embarazo, ciertas conectivopatas,...). Por tanto, recuerda en adelante que
un paciente con FTA-abs negativo debe hacerte pensar que muy probablemente no padece (ni ha padecido) una sfilis. (Alternativa A)
PREGUNTA N 29
RESPUESTA: C
Centrndonos en esta pregunta, la respuesta incorrecta es la C. El Ancylostoma duodenale es un helminto que se fija a la pared
intestinal, de cuya sangre se alimenta, gracias a un gancho con el que se fija a la mucosa. El hecho de ser un helminto y poseer un
gancho, le ha valido el carioso apodo de gusano ganchudo. Dado que se alimenta de sangre, el tipo de anemia que produce es
FERROPNICA, no megaloblstica; de ah la falsedad de la respuesta C. Existe otro helminto que s es capaz de producir anemia
megaloblstica, por consumo de vitamina B12, llamado Diphyllobotrium latum.
PREGUNTA N 30
RESPUESTA: C
El caso que nos plantean es el de un paciente con cefalea y leves signos menngeos, aparte de que es VIH positivo y tiene un lquido
cefalorraqudeo con las caractersticas que ya has visto en el enunciado. De las opciones que nos plantean, hay dos posibilidades
compatibles con el anlisis del LCR que nos dan como dato, que son la B y la C (Listeria y Criptococo). El elemento que nos hace
orientar por la segunda es que se trata de un paciente VIH positivo con un grado muy importante de inmunosupresin (50 CD4/mL), lo
que hace realmente probable la infeccin criptoccica como causa de su meningitis; por otro lado el marcador clnico de la
criptococosis cerebral es la cefalea de das de evolucin (Ms de una semana en el enunciado). El dato que dan de la TAC (lig era
atrofia cortical) no debe confundirte. Esto es algo relativamente frecuente en los pacientes VIH positivos, por efecto del propio VIH.

Examen Simulacro :: Ciclo de Revisin en Medicina :: Sbado 4 de Agosto del 2012 :: www.estudiosmyc.com
PREGUNTA N 31
RESPUESTA: C
Esta bacteria puede producir infecciones muy variadas, como farngeas, escarlatina, imptigo... Adems, pueden producirse
complicaciones no supurativas en relacin con ella, como la glomerulonefritis postestreptoccica y la fiebre reumtica. El tratamiento
de la faringoamigdalitis por S. pyogenes admite varias alternativas. En caso de alergia a betalactmicos, los macrlidos orales son una
opcin vlida (respuesta E correcta). La amoxicilina oral es tambin un tratamiento eficaz (respuesta D correcta). Date cuenta que la
respuesta dice amoxicilina, y no amoxicilina-clavulnico. Si se sabe a ciencia cierta que se trata de S. pyogenes, no es necesario
aadir cido clavulnico, porque hasta la fecha no se ha visto que esta bacteria produzca betalactamasas. La nica penicilina activa
va oral es la penicilina V (fenoximetilpenicilina), que es til como tratamiento si se siguen las pautas que dice la opcin B. Para
prolongar la vida media de la penicilina en el organismo, una opcin es unirla a una sal, que puede ser procana o benzatina. La
combinacin penicilina G benzatina permite una nica inyeccin intramuscular como tratamiento de esta infeccin (opcin A
correcta). En cambio, la penicilina G procana tiene una vida media considerablemente inferior, por lo que la opcin C no es vlida.
PREGUNTA N 32
RESPUESTA: B
Los estreptococos eran la causa ms frecuente de endocarditis infecciosa, con gran diferencia, especialmente S. viridans. Sin
embargo, desde los aos 90 ha existido una disminucin de la endocarditis por estreptococos, coincidiendo con un aumento de las
producidas por estafilococos, que suponen del 30-50% del total. Dentro de este grupo, el S. aureus es 5-10 veces ms frecuente que
el S. epidermidis, que s deberas tener en cuenta si te preguntan por la endocarditis protsica precoz. La alternativa correcta es la B.
PREGUNTA N 33
RESPUESTA: B
La opcin menos probable es la B, el TEP, que es una patologa trombtica, puesto que la estreptoquinasa, como cualquier
tromboltico, hace muy improbable que se formen trombos tan slo seis horas despus de haberla administrado. Todo lo contrario
ocurre con la opcin E (hemorragia cerebral), que es una complicacin conocida de estos frmacos. Las opciones A, C y D son
tambin posibles puesto que son complicaciones mecnicas del IMA que pueden cursar con hipotensin y obnubilacin.
PREGUNTA N 34
RESPUESTA: B
Los antagonistas de los receptores de la angiotensina II tienen un efecto y mecanismo de accin similares a los de los IECA, aunque
en vez de bloquear la produccin de angiotensina II, inhiben competitivamente su unin al receptor AT1 de la angiotensina II. Las
indicaciones, utilidad y tolerancia son similares a las de los IECA, aunque NO producen tos ni angioedema, al no interferir sobre la
degradacin de las bradiquininas (opcin B correcta). La cininasa II es la enzima encargada de metabolizar las bradiquininas, y su
estructura es idntica a la enzima convertidora de angiotensina, por lo que los IECA tambin la inhiben, aumentando los niveles de
bradiquininas. Por todo esto se suelen reservar para los pacientes con intolerancia a los IECA, sobre los que existen ms estudios que
prueban el aumento de supervivencia. Recordamos que ambos grupos farmacolgicos estn contraindicados en embarazadas,
pacientes con hiperpotasemia, estenosis renal bilateral o estenosis de la arteria renal con rin nico.
PREGUNTA N 35
RESPUESTA: C
El flutter se define electrofisiolgicamente como una macrorreentrada a nivel de aurcula derecha, en la que la activacin auricular se
produce de manera crneo-caudal en la pared lateral de la aurcula derecha, para alcanzar la regin del itsmo cavotricuspdeo y
propagarse posteriormente de manera caudo-craneal a nivel del septum hasta alcanzar el techo de la aurcula derecha. Por lo tanto, la
ablacin del istmo cavotricuspdeo obtiene un porcentaje de xitos cercano al 95% en el flutter istmo-dependiente o comn (el ms
frecuente), en el que la reentrada utiliza esta estructura como parte del circuito (opcin C correcta). No obstante, se puede realizar
tambin la prevencin de recurrencias con los mismos frmacos que se usan en la FA, por ejemplo la amiodarona (opcin E), que en
este caso es menos eficaz que la ablacin con catter y radiofrecuencia.
PREGUNTA N 36
RESPUESTA: B
El caso clnico presenta un paciente con disminucin de conciencia e hipotenso, que tiene una presin de enclavamiento disminuida
(<18 mmHg) y una presin en la aurcula derecha elevada (o presin venosa yugular). Esta situacin se corresponde con una
disfuncin del VD, que es incapaz de bombear sangre al VI por lo que tendremos dos tipos de sntomas: los derivados del defic iente
aporte de sangre a los tejidos (obnubilacin e hipotensin con bajo gasto cardaco) y los secundarios a la sobrecarga retrgrada de
lquidos (aumento PVC). La presencia de lquido en el pericardio, que ocurre en el taponamiento cardaco, y la obliteracin de la
cavidad pericrdica por fibrosis, en el caso de la constriccin pericrdica; ocasionan ambas un aumento de la presin pericrdica. Esta
presin se transmitir primero a las cmaras derechas, por ser stas las que trabajan a menor presin, lo que producira la disfuncin
diastlica descrita anteriormente, al dificultar el llenado de los ventrculos. Por lo tanto las opciones A y E son ciertas. El infarto de
ventrculo derecho ocurre hasta en un 30% de los IMA inferiores y cursa como en esta pregunta con clnica de insuficien cia cardiaca
aguda (aumento de PVC y disminucin del gasto cardaco). Es importante destacar que el tratamiento de esta situacin sera la
administracin de lquidos intravenosos y frmacos inotrpicos positivos, dato muy preguntado en el MIR. Por lo tanto la opcin C es
cierta. Respecto al TEP (tromboembolismo pulmonar), se produce un aumento de las resistencias vasculares pulmonares debido a la
disminucin del rea transversal del lecho arterial y a la produccin de sustancias neurohumorales como la serotonina. Este aumento
de las resistencias produce hipertensin pulmonar e insuficiencia derecha aguda, que es la causa ms frecuente de muerte inmediata
por TEP. Si el VD se dilata, el tabique puede abombarse hacia el VI, con la consiguiente restriccin al llenado y aparicin de sntomas
de bajo gasto (presencia de pulso paradjico). Opcin D cierta. Adems en este paciente no habra un estado de deshidratacin ya
que la PVC est elevada. Por lo tanto la opcin B es la falsa.
PREGUNTA N 37
RESPUESTA: C
En esta ocasin se trata de una pregunta terica acerca del manejo del fonendoscopio, la cual se responda sabiendo que la campana
detecta mejor bajas frecuencias y la membrana detecta mejor las altas. De este modo, el 3 tono, el 4 tono y el retumbo de la
estenosis mitral se detectaran mejor con la campana al ser tonos de baja frecuencia. Opcin C correcta. El tercer ruido se produce por
un llenado ventricular rpido y aunque puede ser fisiolgico en personas jvenes, generalmente es indicativo de una insuficiencia

Examen Simulacro :: Ciclo de Revisin en Medicina :: Sbado 4 de Agosto del 2012 :: www.estudiosmyc.com
cardiaca. El cuarto ruido es siempre patolgico y se debe a la contraccin de la aurcula contra un ventrculo con una disten sibilidad
reducida. El chasquido de apertura es un tono diastlico de alta frecuencia que se debe a estenosis de una vlvula aurculo-ventricular,
por lo que se oira mejor con la membrana. El prolapso mitral tiene un click al final de la sstole que se sigue de un soplo de alta
frecuencia, piante, por lo tanto tambin se oira mejor con la membrana. Es importante recordar que el soplo del prolapso igual que el
de la miocardiopata hipertrfica se hace ms intenso cuando disminuye la precarga o la postcarga. Opcin B y D falsas. Para
determinar en qu momento se produce el ruido o soplo hay que considerar la relacin con otros fenmenos del ciclo cardaco como el
pulso carotdeo, la PVY o el impulso apical, pero no depende de la parte del fonendoscopio con la que se ausculte. Opcin A f alsa. En
nios pequeos se puede utilizar tanto la membrana como la campana, aunque en muchas ocasiones se utiliza la campana por razn
de tamao. La membrana es ms amplia y abarcara gran parte del trax del nio, por lo que no podramos determinar en qu foco
estamos auscultando un soplo o ruido en el caso en que ste existiese. Opcin E es falsa.
PREGUNTA N 38
RESPUESTA: E
El delirium tremens se trata de un cuadro producido por la abstinencia al alcohol de forma brusca. Cursa con un sndrome conf usional
con desorientacin, alteraciones perceptivas como ilusiones y alucinaciones de gran intensidad y vivencia angustiosa, sobre todo
visuales (tpicas las microzoopsias). Tambin se pueden producir delirios, inquietud, agitacin y clnica vegetativa. En el tratamiento,
se usan fundamentalmente las benzodiacepinas (cloracepato, diacepam) y el clometiazol. (Alternativa E) En las formas leves, puede
usarse tambin el tiapride.
PREGUNTA N 39
RESPUESTA: C
Dentro del cuadro de la esquizofrenia, es frecuente la existencia de pseudoalucinaciones o alucinacin psquica falsa, caracterizada
por tratarse de una percepcin sin objeto y sin crtica de la misma (al contrario de la alucinosis en la que el paciente duda o crtica la
veracidad o autenticidad de esta), y que el paciente la sita en su espacio interior, siendo sobre todo de tipo auditivo. Por ejemplo, el
paciente oye voces en el interior de su cabeza que le dan rdenes o le incitan a que haga algo. Si estas ocurrieran en el espacio
exterior, el paciente no las oira dentro de su cabeza, sino que se acercara a las esquinas o mirara al cielo u otro lugar desde donde
cree que le estn hablando. Las pareidolias, son un tipo de ilusiones (deformacin de una percepcin real), que se desencadenan por
sugestin. La alternativa correcta es la C.
PREGUNTA N 40
RESPUESTA: B
Entre las alteraciones que se exponen, es el aumento de la amilasa srica la nica determinacin til que apoya el diagnstico de sta
enfermedad, la cual adems estar ms aumentada cuanto mayor sea la incidencia de maniobras purgativas, y que se debe a un
aumento de la amilasa tanto salivar como pancretica. Adems, no olvidar las alteraciones ms tpicas por vmitos de repeticin como
son la hipopotasemia, la hipocloremia, hipomagnesemia y alcalosis metablica. La alternativa correcta es la B.
PREGUNTA N 41
RESPUESTA: A
La clnica descrita tanto por el mal estado general, la fiebre, la distribucin y morfologa del exantema, los perodos tanto de prdromo
como de evolucin, son los tpicos de una fiebre botonosa. Para que no quede duda, tras exposicin en el campo, se observa una
picadura (tache noir). El tratamiento ms eficaz es con tetraciclinas o ms preferiblemente con doxiciclina a las dosis indicadas.
PREGUNTA N 42
RESPUESTA: C
En la infeccin por VIH, cuando la inmunidad est seriamente daada (y nuestro paciente con esos antecedentes, la tiene) cualquier
lcera, sea cual sea su forma, localizacin, aspecto, evolucin, es una infeccin por herpes simple mientras no se demuestre lo
contrario. El dolor intenso y el crecimiento geogrfico descartan cualquier otra opcin.
PREGUNTA N 43
RESPUESTA: E
La edad de aparicin de las lesiones en la vejez, la morfologa y distribucin de las mismas as como la microscopia ptica, sealan
hacia un penfigoide ampolloso. No obstante, y ante la rara, pero posible confusin con una epidermlisis ampollosa adquirida, se ha
realizado la nica prueba que las diferencia que es IFD tras tcnica de separacin de la UDE con sal, el antgeno de la Epide rmlisis
ampollosa adquirida slo quedar en dermis, en el penfigoide ampolloso, quedar bien en techo slo, bien como en nuestro caso, en
ambos lados.
PREGUNTA N 44
RESPUESTA: C
La edad de inicio, la morfologa y distribucin de las lesiones apuntan a una Dermatitis Herpetiforme. Ningn dato va en cont ra de ella.
La afectacin intestinal es parcheada por lo que no tiene un significado en contra del diagnstico que sea negativa. Los Ac. antigliadina
IgA slo aparecen en un tercio de los casos, y los Ac. antigliadina IgG, menos especficos pero presentes en ms de la mitad de los
enfermos s lo son, lo que afirma el diagnstico. La dieta est indicada en cualquiera de los casos, porque disminuye la cantidad de
sulfonas necesaria para controlar la enfermedad.
PREGUNTA N 45
RESPUESTA: C
La vasculitis se descarta por el tipo de lesiones cutneas. El resto de los cuadros s cursan con lesiones eritematoescamosas, sin
embargo: el eczema numular suele ser un proceso muy crnico, en el herpes circinado (tinea corporis) las lesiones aumentan
lentamente tanto en nmero como en tamao, aunque la pitiriasis rosada podra encajar en el cuadro expuesto, es rara en las edades
extremas de la vida, y adems es bastante tpico que los brotes de psoriasis gutata sean desencadenados en la infancia por
infecciones estreptoccicas.
PREGUNTA N 46
RESPUESTA: C
La psoriasis tipo Von Zumbusch es un pustulosis aguda generalizada. En la rosola sifiltica es caracterstica la afectacin de palmas
y/o plantas. En el eczema suelen existir vesculas, erosiones y costras. La lesin descrita es caracterstica de la pitiriasis rosada, la

Examen Simulacro :: Ciclo de Revisin en Medicina :: Sbado 4 de Agosto del 2012 :: www.estudiosmyc.com
placa inicial correspondera al medalln herldico, adems en la pitiriasis rosada las lesiones se localizan en tronco y regin proximal
de miembros, lo cual no tiene por qu suceder en la psoriasis.
PREGUNTA N 47
RESPUESTA: D
Las lesiones peribucales corresponderan a un herpes simple, tras el cual se desencadena un eritema exudativo multiforme con
lesiones en diana y lesiones tipo herpes iris de Bateman. No se trata de un Stevens-Johnson (o eritema multiforme major), pues no
hay lesiones en mucosas.
PREGUNTA N 48
RESPUESTA: C
Aunque todas las patologas entran en el grupo de las vasculitis necrotizantes, en ninguna de ellas salvo en la PAN la lesin elemental
es el ndulo
PREGUNTA N 49
RESPUESTA: D
Varios procesos como el eczema seborreico o psoriasis pueden causar eritrodermia debido a su extensin. Es caracterstico del
sndrome de Sezary la eritroderma con adenopatas y ms de 10% de clulas de Sezary en sangre perifrica. Se acompaa tambin
de alteraciones de pelo y uas. La parapsoriasis en grandes placas es una forma inicial de micosis fungoide.
PREGUNTA N 50
RESPUESTA: A
La ictiosis es una genodermatosis caracterizada por piel seca y cubierta de escamas. La ictiosis vulgar es la ms frecuente, suele
comenzar en la infancia mejorando con la edad. La ictiosis ligada a X la padecen slo los varones, se asocia a un dficit de esterio de
sulfatasa, comienza en la primera infancia y no respeta las flexuras. La ictiosis laminar aparece ya en el momento del nacimiento (bebe
coloidon) y suele acompaarse de ectropin y alteracin de los anejos. La enfermedad de Darier consiste en pequeas ppulas
marrones localizadas en reas seborreicas.
PREGUNTA N 51
RESPUESTA: E
La erupcin polimorfa lumnica es una reaccin de inmunidad retardada frente a un antgeno cutneo que suele presentarse
anualmente en primavera o verano y suelen disminuir las manifestaciones a medida que avanza la estacin. Las reacciones
fototxicas aparecen tras la primera exposicin, en horas, se localizan en reas fotoexpuestas y clinicamente son una quemadura
solar. Las reacciones fotoalrgicas son por reaccin tipo IV y tardan ms de 24 horas en aparecer. Clnicamente es una dermatitis
eczematosa que desborda reas expuestas. Hay frmacos que producen ambos tipos de reacciones.
PREGUNTA N 52
RESPUESTA: D
Pensaramos en lepra si estuviramos en un rea endmica y las mculas tuvieran anestesia. La biopsia, localizacin de las lesiones y
la respuesta al tratamiento son caractersticas de vitligo.
PREGUNTA N 53
RESPUESTA: A
El hecho de que un paciente con garganta estreptoccica no mejore con tratamiento en 48 horas har sospechar la posibilidad de una
mononucleosis infecciosa.
PREGUNTA N 54
RESPUESTA: C
La apendicitis es poco frecuente en la etapa de la lactancia, aunque puede manifestarse por vmitos, afectando principalmente a nios
en edad prescolar y escolar. El resto de las respuestas incluye las causas ms frecuentes de vmitos en la lactancia, unidas a las
infecciones por ejemplo urinarias. Los errores dietticos o psicolgicos figuran como una de las causas ms importantes de vmitos en
el lactante que crece, gana peso y se desarrolla normalmente.
PREGUNTA N 55
RESPUESTA: B
La insuficiencia suprarrenal cursa habitualmente de forma gradual, pero puede presentarse de forma brusca con deterioro rpido,
hipotensin, hipoglucemia, hiponatremia e hiperpotasemia. La recuperacin resulta llamativa tras la administracin de corticoides
intravenosos, junto con glucosa al 5% y suero salino para reponer la volemia y la natremia. Para confirmar el diagnstico deben
recogerse niveles de cortisol antes de iniciar el tratamiento, que estarn disminuidos tanto en sangre como en orina.
PREGUNTA N 56
RESPUESTA: C
Los datos clnicos sugieren un coma etlico. La ingestin de barbitricos se acompaa de pupilas miticas, las anfetaminas provocan
pupilas midriticas, el infarto cerebral y la hemorragia subaracnoidea presentan una exploracin neurolgica anormal, el status
convulsivo tendra una duracin mnima de 30 minutos.
PREGUNTA N 57
RESPUESTA: B
La evaluacin de un nio con prpura y petequias de aparicin aguda debe incluir un recuento plaquetario para descartar una
trombopenia. Una cifra de plaquetas normales sugiere vasculitis de pequeos vasos, alteraciones funcionales plaquetarias o trastorno
de la coagulacin. La distribucin simtrica, lesiones palpables y el recuento plaquetario normal son ms compatibles con prpura de
Schonlein Henoch.
PREGUNTA N 58
RESPUESTA: B
La recurrencia de cuadros catarrales sugiere la posibilidad de afectacin tica (Otitis medias serosa), que es la causa ms frecuente
de problemas auditivos en la edad prescolar

Examen Simulacro :: Ciclo de Revisin en Medicina :: Sbado 4 de Agosto del 2012 :: www.estudiosmyc.com
PREGUNTA N 59
RESPUESTA: D
Un nio fuera del periodo neonatal que tenga una obstruccin intestinal, sin haber sufrido una operacin previa, y que no tenga una
invaginacin, lo ms probable es que presente un divertculo de Meckel.
PREGUNTA N 60
RESPUESTA: C
La bronquiolitis es un cuadro respiratorio asociado con frecuencia al V.R.S. que se produce en nios menores de dos aos con cuadro
catarral previo. Produce insuficiencia respiratoria que puede evolucionar hacia un cuadro grave. La no existencia de estridor, disfagia,
la edad, el ser primer episodio, y no tener antecedentes previos descarta el resto de los cuadros.
PREGUNTA N 61
RESPUESTA: D
El cuadro descrito se encuadra dentro de la abstinencia por adiccin materna a herona, que asocia al bajo peso otros trastornos:
neurolgicos, digestivos y de comportamiento. Se presenta en el 80 % de los hijos de madres heroinmanas.

PREGUNTA N 62
RESPUESTA: C
El clico del lactante aparece en el primer trimestre de vida. Su etiologa y fisiopatologa son desconocidas. El
diagnstico es fundamentalmente clnico caracterizado por llanto de intensidad variable, de predominio vespertino,
irritabilidad, distensin abdominal, flexin de piernas, y falta de respuesta a estmulos calmantes. El desarrollo sicomotor
y somatomtrico son normales.
PREGUNTA N 63
RESPUESTA: C
El Sndrome de West se produce en lactantes entre los 2-6 meses de edad. Ocasiona en ellos prdida de adquisiciones
sicomotoras, retraso sicomotor, y presenta un patrn electroencefalogrfico tpico (Hipsarritmia). Hemos de diferenciarlo
de los clicos del lactante (que cursa sin prdida de conocimiento ni retraso sicomotor) por la similitud entre la crisis de
ambos. Tambin hay que diferenciarlo de otros cuadros epilpticos mioclnicos benignos que no afectan al desarrollo
sicomotor.
PREGUNTA N 64
RESPUESTA: D
Todas las entidades citadas pueden producir ictericia persistente a la edad del lactante citado, excepto la ictericia
fisiolgica. La ictericia fisiolgica aparece entre el segundo y tercer da desapareciendo hacia el octavo da, suele ser de
escasa intensidad cuya etiologa principal es una inmadurez del sistema enzimtico heptico (Glucuronil transferasa)
junto al aumento de la destruccin de hemates.
PREGUNTA N 65
RESPUESTA: C
La perforacin gstrica dara clnica diferente, con un dolor ms difuso, no slo localizado en epigastrio, y s que nos
permitira pasar la SNG. La estenosis pilrica aguda se da en el contexto de una lcera antral o pilrica y tambin permite
pasar la SNG. El sndrome de Boerhaave se asocia con vmitos bruscos previamente, dolor mediastnico tras la ruptura
esofgica y, no obstaculizara la SNG. El tricobezoar no da clnica de obstruccin brusca, sino crnica, con problemas de
vaciamiento gstrico de ms evolucin. El vlvulo se caracteriza por dolor intenso, epigstrico y de aparicin brusca.
PREGUNTA N 66
RESPUESTA: C
Toda lcera gstrica es potencialmente maligna independientemente de su aspecto, por lo que est indicado biopsiar
todas ellas y tomando al menos tres muestras (del nicho y de la periferia).
PREGUNTA N 67
RESPUESTA: D
La presencia de cuadro diarreico con masa palpable nos hace pensar en las respuestas 3, 4 y 5. Estos tres tumores
podran ser ms o menos blandos, y no dolorosos, pero el nico que tpicamente cursa con un cuadro de deplecin
electroltica por ser abundamentemente secretor, es el adenoma tipo velloso de recto.
PREGUNTA N 68
RESPUESTA: D
Todo cuadro de disfagia de larga evolucin sin sndrome constitucional acompaante y sin ninguna otra alteracin hace
pensar en su trastorno motor de esfago, y por tanto tendr una endoscopia normal, y slo podr ser filiado mediante
una manomera esofgica.
PREGUNTA N 69
RESPUESTA: E
Todo varn adulto que presenta cuadro de hemorragia digestiva baja, aunque presente causa aparente del sangrado
como podran ser las hemorroides internas, debe hacerse revisar todo el colon mediante colonoscopia, para descartar
otra causa asociada, y sobre todo el cncer colorrectal.

Examen Simulacro :: Ciclo de Revisin en Medicina :: Sbado 4 de Agosto del 2012 :: www.estudiosmyc.com
PREGUNTA N 70
RESPUESTA: D
El cuadro descrito se trata de un megacolon txico en un paciente con colitis ulcerosa. Aunque el tratamiento del cuadro
suele ser quirrgico requiriendo panproctocolectoma, el primer paso siempre debe ser conservador intentando revertir el
cuadro, y as obviar los riesgos de la ciruga.
PREGUNTA N 71
RESPUESTA: D
Las biopsias gstricas se mantienen negativas y se ha producido una cicatrizacin sustancial. Mientras las biopsias sean
negativas debe darse ms tiempo a que cicatrice la lcera antes de remitir al paciente a ciruga por temor a que la lcera
sea maligna. La conducta apropiada, debido a que la paciente est asintomtica y con buena evolucin en la
cicatrizacin, es continuar con el tratamiento con ranitidina 8 semanas ms y despus repetir la endoscopia. Otra
conducta, en especial si la lcera no muestra este progreso satisfactorio, sera cambiar a omeprazol.
PREGUNTA N 72
RESPUESTA: C
Los plipos gstricos son de origen epitelial y de ellos el 90% son hiperplsicos y del 10 al 20% adenomatosos. si bien
pueden causar dolor abdominal a hemorragia, hasta la mitad son asintomticos. La extirpacin total de la lesin mediante
polipectoma con asa a fin de obtener el espcimen total por estudio histolgico es diagnstica y curativa.
PREGUNTA N 73
RESPUESTA: C
Los pacientes con celaca pueden mostrar anemia ferropnica aislada. El festoneo de los pliegues duodenales sugiere
esta afeccin y puede ser uno de sus primeros signos endoscpicos, lo cual viene apoyado por los hallazgos de la
biopsia.
PREGUNTA N 74
RESPUESTA: C
El paciente presenta los datos analticos y de exploracin caractersticos del infarto intestinal en evolucin, en probable
relacin con arteritis.
PREGUNTA N 75
RESPUESTA: E
El desarrollo de un sndrome txico en personas de edad avanzada, en especial cuando se acompaa de pruebas de
afeccin sistmica, debe hacer pensar en Enfermedad de Whipple.
PREGUNTA N 76
RESPUESTA: D
La recomendacin previa para vigilancia por plipos del colon metacnicos ha sido colonoscopia anual. Datos recientes
del National Colon Polyp Study apoyan un intervalo de 3 aos para el procedimiento, excepto en persona con plipo
adenomatoso > 1 cm. de 9, en quienes hay mltiples plipos y en pacientes con antecedente de plipo maligno. Tambin
se ha sugerido que los adenomas tubulares < 1 cm no requieren vigilancia adicional.
PREGUNTA N 77
RESPUESTA: B
Esta pregunta hace referencia a una pubertad precoz verdadera, con aumento de la talla con una edad sea mayor a la
cronolgica, telarqua, as como valores hormonales ms altos que los que corresponden a la prepubertad. Por otra parte
descartar otras causas de pubertad precoz como son las debidas al sndrome de Albright y alteraciones del tiroides,
suprarrenal o primitivas del ovario (tumores ovricos). El tratamiento de eleccin son los anlogos de GbRH ya que
retrasan el desarrollo sexual y la maduracin esqueltica. El acetato de progesterona tiene escasos resultados en cuanto
al control del crecimiento. La respuesta 1 sera la correcta si sospechsemos patologa ovrica tumoral y la 5 en caso de
tratarse exclusivamente de una telarqua precoz.
PREGUNTA N 78
RESPUESTA: E
La ausencia de menstruacin tras la administracin de gestgenos descarta la anovulacin como causa de amenorrea
(SOP) y la opcin 3 se elimina ya que tiene niveles de prolactina normales. La presencia de menstruacin al administrar
estrgenos y gestgenos nos orienta a que el fallo pueda estar a nivel del compartimento II o III-IV, descartando el
sndrome de Asherman. Los niveles de gonadotropinas bajos nos permite descartar la opcin 2 y orientar el caso clnico
como opcin correcta la nmero 5.
PREGUNTA N 79
RESPUESTA: D
Ante una metrorragia postmenopusica es esencial realizar una ecografa vaginal que nos permita valorar la lnea media
endometrial y si existe alguna duda de patologa endometrial, debemos realizar legrado fraccionado para diagnstico
anatomopatolgico endometrial. Despus de descartar patologa endometrial podemos valorar la THS ms adecuada.
PREGUNTA N 80
RESPUESTA: A
La cadidiasis vaginal se caracteriza por la aparicin de prurito y edema de vulvas e introito, con secrecin vaginal
blanquecina grumosa de aspecto caseoso. Placas blanquecinas irregulares en mucosa vaginal que se desprenden,

Examen Simulacro :: Ciclo de Revisin en Medicina :: Sbado 4 de Agosto del 2012 :: www.estudiosmyc.com
dejando ulceraciones rojas superficiales. Entre las fracturas favorecedoras ms importantes est el tratamiento antibitico
prolongado, embarazo, anovulatorios...
PREGUNTA N 81
RESPUESTA: C
Aunque probablemente se trate de una artritis reumatoide, en este momento no cumple criterios para el diagnstico pues
slo lleva 15 das de evolucin.
PREGUNTA N 82
RESPUESTA: B
Pues aunque la paciente cumple tres criterios para el diagnstico de arteritis de la temporal (edad, VSG y cefalea)
debemos realizar una biopsia de la arteria temporal para confirmar el diagnstico pues en el tratamiento de la arteritis de
la temporal las dosis de prednisona son mayores que las que usaramos en la polimialgia reumtica.
PREGUNTA N 83
RESPUESTA: D
Pues antes de diagnosticar una espondiloartropata debemos descartar los dos procesos infecciosos crnicos capaces
de provocar sacroiletis con frecuencia en nuestro medio, la TBC y la brucella.
PREGUNTA N 84
RESPUESTA: E
Pues las patologas que pueden simular una fibromialgia son el LES de inicio, el hipotiroidismo o la miositis como ms
importantes.
PREGUNTA N 85
RESPUESTA: C
Pues el diagnstico ms probable es una DSR (sd. hombro-mano) que por clnica, radiografa y gammagrafa sea se
diagnosticar. La gammagrafa con galio no est indicada pues no se sospecha un proceso infeccioso.
PREGUNTA N 86
RESPUESTA: C
Pues la paciente presenta un eritema nudoso junto a un peritobillo bilateral caracterstico de la sarcoidosis.
PREGUNTA N 87
RESPUESTA: B
El diagnstico diferencial de los aplastamientos vertebrales debe realizarse fundamentalmente entre la osteoporosis, la
osteomalacia, las metstasis seas y el mieloma mltiple. La presencia de debilidad muscular debe hacer sospechar el
diagnstico de osteomalacia. En este caso, si el calcio, el fsforo y la vitamina D son normales, puede descartarse
razonablemente el diagnstico; si estn alteradas, la biopsia sea proporcionar el diagnstico definitivo. En caso de
mieloma mltiple, la VSG est elevada y suele existir anemia. La densidad mineral sea est disminuida en osteoporosis,
osteomalacia y mieloma mltiple. La gammagrafa sea y la resonancia magntica son tiles para detectar lesiones
subclnicas en caso de metstasis seas.
PREGUNTA N 88
RESPUESTA: B
La paciente cumple 4 criterios para el diagnstico de lupus eritematoso sistmico (aftas orales, artritis, leucolinfopenia y
ANA +), por lo que se puede establecer el diagnstico de esta enfermedad.
PREGUNTA N 89
RESPUESTA: A
En este caso, sin ninguna manifestacin mayor de lupus eritematoso sistmico, el tratamiento indicado son
antiinflamatorios no esteroideos. En caso de persistencia o empeoramiento de la clnica, podra valorarse la utilizacin de
otros tratamientos.
PREGUNTA N 90
RESPUESTA: E
La hiperuricemia asintomtica no es tributaria de tratamiento hipouricemiante a menos que las cifras de cido rico en
sangre predispongan al desarrollo de una nefropata.
PREGUNTA N 91
RESPUESTA: D
La fiebre reumtica es un tipo de artritis reactiva postestreptoccica que se diagnostica con los criterios de Jones.
PREGUNTA N 92
RESPUESTA: D
El tratamiento conservador de la artrosis incluye analgesia y/o antiinflamatorios, descarga articular y fisioterapia. Cuando
estas medidas han fallado y la impotencia funcional es severa, puede plantearse una solucin quirrgica. La osteotoma
de alineacin puede retrasar sensiblemente la evolucin del proceso artrsico.

Examen Simulacro :: Ciclo de Revisin en Medicina :: Sbado 4 de Agosto del 2012 :: www.estudiosmyc.com
PREGUNTA N 93
RESPUESTA: D
La latencia de sueo se objetiva mediante el MSLT (mltiple sleep latency test), que en cuanto es inferior a 10 minutos
indica hipersomnia. La dificultad para despertarse y la falta de sueo reparador son caractersticas de la hipersomnia que
la diferencian de los grandes dormidores que s tienen sueos reparadores aunque su tiempo de sueo sea mayor que
la media. Una duracin inadecuada del sueo nocturno (inferior a 7 horas) puede producir sntomas similares a la
hipersomnia primaria pero los pacientes recuperan el sueo cuando tienen das libres. En la narcolepsia el sueo es ms
reparador, existen menos dificultades para levantarse, la somnolencia diurna es en forma de crisis y no continua,
presentan siestas cortas, e intrusiones recurrentes de fases REM durante el da. En la apnea del sueo no suele haber
un patrn recidivante, suele haber alteraciones cardiovasculares y obesidad y no suele haber alteraciones emocionales o
de conducta descritas que corresponden claramente a las de un sndrome de Klein-Levin o hipersomnia idioptica
recidivante, sin embargo, a veces es necesario hacer un registro polisomnogrfico para diferenciar una apnea obstructiva
de una hipersomnia idioptica. En la epilepsia los fenmenos postcrticos no duran tanto y en las auras o crisis
psicomotoras, si bien existe cierta disminucin del nivel de conciencia no se le puede despertar como a un hipersomne.
Siempre que un diagnstico es difcil se plantea la posibilidad de un sndrome conversivo o simulacin, los estados
disociativos se dan sobre todo en relacin con una situacin que contara al paciente, las lesiones o accidentes que se
simulan no suelen ser graves salvo error de clculo. La personalidad del paciente conversivo o disociativo no es normal
entre crisis.
PREGUNTA N 94
RESPUESTA: C
La paciente padece posiblemente anorexia nerviosa. Los pacientes con dicho trastorno suelen negar datos necesarios
para el diagnstico, por lo que ante la evidencia de la delgadez debemos sospecharlo inmediatamente y buscar algn
dato objetivo como el peso, presencia de amenorrea, signos de desnuricin como el fro, la fragilidad de piel y mucosas,
etc. Sin embargo estos signos no son especficos. La familia no es tampoco una fuente fiable de informacin, existe en
ocasiones excesiva tolerancia o incomprensin hacia lo que est pasando. La paciente desconfa de los mdicos a los
que ve como entrometidos en su libertad de comer como quiera. Por otro lado la irritabilidad de estas pacientes crea un
rechazo por parte de los profesionales. La nica manera de establecer un diagnstico cierto es mediante un dilogo
abierto, dejando a un lado las preguntas cerradas a fin de averiguar si la paciente est preocupada de forma desmedida
por engordar o si se ve gorda a pesar de su delgadez.
PREGUNTA N 95
RESPUESTA: C
Todos los trastornos mencionados pueden presentar hiperfagia. En el sndrome de Klein-Levin la hiperfagia se presenta
tras episodios de hipersomnia. En la depresin bipolar y en la esquizofrenia los sntomas predominantes son los afectivos
y psicticos respectivamente y adems la hiperfagia no es de forma tan recurrente en forma de atracones seguidos de
vmitos. En la anorexia nerviosa tipo compulsivo el cuadro es prcticamente el mismo pero suele haber disminucin de
peso, amenorrea y signos de desnutricin que faltan en la bulimia.
PREGUNTA N 96
RESPUESTA: E
El trastorno de angustia suele presentarse en crisis repetidas de breves minutos de duracin, salvo que se tengan varias
crisis en salvas, situacin en la cual la crisis parece durar ms tiempo, pero una exploracin detallada nos revelar
altibajos. Las crisis aparecen sin relacin a estar en situaciones temidas como en las fobias (agorafobia), con un cuadro
clnico muy similar al que presenta la paciente. Se desarrolla temor anticipatorio a sufrir nuevas crisis y generalmente
aparece temor a estar en situaciones donde sea difcil obtener ayuda si se tiene la crisis (agorafobia). La presencia de
conectivopatas (prolapso mitral, hiperlaxitud ligamentaria) parece ser mayor en estos pacientes y no slo porque sean
explorados ms por la presencia de quejas somticas. De todas maneras tambin los pacientes psiquitricos pueden
tener patologa mdica asociada, en este caso prolapso mitral. La frecuencia cardaca y la tensin arterial alta debido a la
descarga noradrenrgica del locus ceroleous es comn en estos pacientes.
PREGUNTA N 97
RESPUESTA: D
No hay signos somticos ni psquicos de ansiedad, sino ms bien enlentecimiento psicomotor. En los trastornos de
personalidad y en la neurosis existe un patrn prolongado en el tiempo de quejas muy variadas, a veces remendando un
cuadro depresivo. Tanto en la demencia como en la depresin hay disminucin de concentracin, el inicio de una
demencia puede simular una depresin pero no suele existir un inicio tan brusco, salvo en las vasculares que se suelen
acompaar de cadas, dficit neurolgicos o episodios confusionales. La paciente presenta anhedonia (incapacidad para
disfrutar de las cosas que antes le daban placer), este sntoma o bien la tristeza han de estar presentes para poder hacer
un diagnstico de depresin mayor. En este caso la depresin presenta sntomas de tipo endgeno como insomnio de
tercera fase, disminucin de apetito, etc. Salvo que la paciente haya tenido contacto con mdicos o bien presente otra
patologa diferente de la depresin no es habitual que la paciente use la palabra depresin o triste, sino trminos ms
descriptivos como estoy por los suelos, lo veo todo negro, estoy ms cansada, etc. De forma similar el paciente con
depresin mayor no viene por s slo sino que ha de ser trado por alguien de confianza al que ha relatado alguna de sus
experiencias.

Examen Simulacro :: Ciclo de Revisin en Medicina :: Sbado 4 de Agosto del 2012 :: www.estudiosmyc.com
PREGUNTA N 98
RESPUESTA: B
El paciente no presenta al parecer alucinaciones, delirios bizarros o sntomas defectuales que nos puedan hacer dudar
con la esquizofrenia o con un cuadro esquizoafectivo. No existe disminucin del funcionamiento normal como en la
depresin, todo lo contrario, el funcionamiento es superior al normal, si bien en este caso no mejor, ya que su
comportamiento le crea problemas (discapacidades) familiares que en un futuro sern tambin sociales y laborales. Los
pacientes ansiosos se saben enfermos y piden ayuda en vez de no ser conscientes de ser enfermos como es el caso del
paciente. Los trastornos de personalidad suelen tener una evolucin prolongada en el tiempo. Los pacientes manacos
pueden tener un nimo expansivo (simptico, de buen humor contagioso) o bien como es este caso al menos durante la
consulta, irritable. El funcionamiento general es superior a la media pero inestable, con realizacin de proyectos cuyo
xito es difcil o imposible, los proyectos que se inician difcilmente se terminan ya que otro inters lo suele desplazar, el
habla suele ser rpida en consonancia con la actividad mental. Si el nivel de actividad alto no le produce alteraciones
sociales, laborales o familiares se llama hipomana, si las produce se llama mana y requiere tratamiento con litio y a
veces adems neurolpticos y/o ingreso.
PREGUNTA N 99
RESPUESTA: B
El paciente presenta un aspecto llamativo sugerente de patologa grave. El hecho de que haya estado vagabundeando
largo tiempo hasta tener una piel tostada y su aspecto desaseado hace improbable una histeria o una intoxicacin por
LSD que tienen crisis de inicio y duracin cortas. Un paciente manaco no llega a tener tal deterioro del autocuidado
personal. El paciente est ms que con un nivel de conciencia bajo, perplejo y parece estar absorto oyendo voces, su
afecto es inapropiado y su conducta bizarra. Parece que el paciente pueda haberse fugado. Estas fugas pueden darse en
la epilepsia, en la histeria y en las psicosis esquizofrnicas, siendo en los dos primeros casos de duracin breve, no
hasta el extremo de poder llevar varios das con tal grado de abandono de autocuidados.
PREGUNTA N 100
RESPUESTA: A
El paciente presenta un comportamiento muy llamativo, su deseo de aislarse no parece motivado por una depresin, que
se acompaara de quejas de nimo bajo, cansancio, inhibicin psicomotriz, etc. El paciente parece tener ideas de
referencia (hablan de l), ideas de vigilancia por los vecinos y parece tener alucinaciones auditivas comentadoras de la
propia actividad (oye a un vecino dos pisos por encima de l comentar lo que hace a cada momento). Es frecuente que
este y otro tipo de pacientes no podamos hacer un diagnstico a partir de sntomas referidos por el paciente sino a partir
de indicios que como en este caso son muy sugerentes.

CLAVE DE RESPUESTAS
PREGUNTA N 1

TEMA B

RESPUESTA: A

Las intoxicaciones o sobredosis de frmacos dan un cuadro de disminucin de nivel de conciencia que puede producir un cuadro
similar, concretamente el sndrome neurolptico maligno si bien se acompaara de rigidez. Es improbable que se le haya
administrado un exceso de frmacos mientras est ingresado, salvo las benzodiacepinas, que a veces se suelen dar como hipnticos
o como tranquilizantes con excesiva facilidad a todo tipo de pacientes, incluso ancianos en los que el riesgo de un mayor efecto es
ms alto. En cualquier caso el cuadro clnico sera completamente diferente, con sopor, sin convulsiones, sin temblor. Siendo
improbable que sea por algn frmaco que se le est administrando en exceso parece ms probable que sea algo que haya dejado de
tomar, en este caso las sospechas se dirigen especialmente al alcohol o las benzodiacepinas cuyo cuadro de abstinencia es identico al
del paciente incluso aunque la familia diga que no bebe mucho o lo normal. Otra causa de desorientacin al poco de ingresar son los
cuadros demenciales pero no se suelen acompaar de convulsiones o alucinaciones.

PREGUNTA N 2

RESPUESTA: C

La paciente presenta un patrn de comportamiento anormal prolongado en el tiempo caracterizado por inestabilidad emocional en las
relaciones en sus objetivos personales, etc. Muestra inters por relacionarse con la gente al revs que el trastorno esquizoide, si bien
este inters es egosta y con el nico objetivo de evitar la soledad, tener alguien a quien contar los problemas, sin tener conciencia de
que estas relaciones han de ser mutuamente satisfactorias. Busca en general su autosatisfaccin llevando las situaciones al lmite
hasta producir la ruptura de las relaciones. No cede a las exigencias de otros con tal de mantener la relacin como en el caso del
trastorno de personalidad por dependencia.

PREGUNTA N 3

RESPUESTA: C

El paciente presenta pensamientos intrusivos, que invaden la conciencia y sin embargo el paciente sabe que esos pensamientos son
suyos y no impuesto por el exterior como por ejemplo en la esquizofrenia, le producen malestar e intenta quitrselos realizando
algunos actos conpulsivos como ponerse cierta ropa, etc. Presenta adems rasgos de personalidad obsesiva como el perfeccionismo,
etc. Es frecuente que existan fobias asociadas a estmulos que desencadenan los pensamientos obsesivos o depresin pero no
parece ser el caso de este paciente.

Examen Simulacro :: Ciclo de Revisin en Medicina :: Sbado 4 de Agosto del 2012 :: www.estudiosmyc.com
PREGUNTA N 4

RESPUESTA: E

La paciente presenta miedo a espacios cerrados, abiertos o llenos de gente, que tienen en comn que son sitios en los que la paciente
piensa que es ms difcil poder obtener ayuda si le ocurriera algo. El solo hecho de exponerse a esos sitios desencadena una crisis de
ansiedad, lo cual es tpico de las fobias, en este caso agorafobia. Adems presenta crisis de pnico, que fue la primera manifestacin
psicopatolgica y que caus secundariamente la agorafobia. Los trastornos simulados o facticios pueden reproducir la mayora de los
trastornos psiquitricos en general con el fin de obtener un beneficio, sin embargo debemos dejarlos siempre como un diagnstico a
hacer una vez excluidos otros ms probables.

PREGUNTA N 5

RESPUESTA: C

En ocasiones en la pancreatitis aguda el electrocardiograma es anormal con alteraciones del ST y onda T semejantes a la isquemia
miocrdica.

PREGUNTA N 6

RESPUESTA: A

La retinopata de Purtscher es una complicacin de la pancreatitis aguda relativamente rara que cursa con prdida sbita e importante
de la visin. En el fondo de ojo muestra manchas algodonosas y hemorragias circunscritas a un rea limitada de paila y mcula. Se
cree que se debe a la oclusin de la arteria retiniana posterior con granulocitos agregados.

PREGUNTA N 7

RESPUESTA: D

La cetoacidosis diabtica se suele acompaar de dolor abdominal y aumento de la amilasa srica, pero la lipasa e isoamilasa
pancretica en suero no se elevan. Los pacientes con pH inferior a 7,32 pueden presentar falsas elevaciones de la amilasa srica.

PREGUNTA N 8

RESPUESTA: E

El sndrome de Rotor es un trastorno del metabolismo de la bilirrubina que produce hiperbilirrubinemia mixta, parecido al sndrome de
Dubin-Johnson pero sin pigmento en la clula heptica y con predominio de monoconjugados en la bilirrubina conjugada. La
colecistografa oral es normal y no existe elevacin tarda en la curva de eliminacin de bromosulftanena. Se hereda de forma
autosmica recesiva.

PREGUNTA N 9

RESPUESTA: C

El sndrome hepatorrenal es una complicacin grave, propia de pacientes cirrticos con ascitis que se caracteriza por insuficiencia
renal progresiva con marcada retencin de sodio y oliguria en ausencia de cualquier causa especfica identificable de disfuncin renal.

PREGUNTA N 10

RESPUESTA: D

Las manifestaciones clnicas de la pancreatitis son dolor en epigastrio irradiado hacia la espalda que mejora al flexionar el tronco y se
acompaa frecuentemente de nuseas y vmitos. El DDI es una causa de pancreatitis de origen medicamentoso.

PREGUNTA N 11

RESPUESTA: B

Entre las complicacions locales de la pancreatitis aguda, se encuentra el flemn pancretico. Se caracteriza por ser una masa
inflamatoria que parece entre la 2.a y 3.a semana tras el episodio de pancreatitis. Se manifiesta por leucocitosis, fiebre,
hiperamilasemia persistente a pesar del tratamiento correcto.

PREGUNTA N 12

RESPUESTA: B

Dentro del grupo de los tumores pancreticos el gastrinoma (productor del sndrome de Zollinger-Ellison) se caracteriza clnicamente
por la existencia de lceras ppticas resistentes al tratamiento mdico, hipercalcemia y diarrea acuosa.

PREGUNTA N 13

RESPUESTA: A

El adenocarcinoma de pncreas es ms frecuente en varones, mayores de 50 aos, los sntomas iniciales son insidiosos y suele
aparecer por lo menos 2 meses antes de que se diagnostique el tumor. Se caracteriza por dolor sordo abdominal y prdida de peso en
ms del 75% de los pacientes. La ictericia debida a la obstruccin biliar aparece en el 80% de los enfermos.

PREGUNTA N 14

RESPUESTA: C

Las causas de pancreatitis crnica son similares a las de la pancreatitis aguda, siendo el alcoholismo una de las principales. Es
caracerstico el dolor persistente, que empeora con la ingesta y no responde a anticidos. La trada clsica consiste en calcificaciones
pancreticas, esteatorrea y diabetes mellitus.

PREGUNTA N 15

RESPUESTA: D

La amiodarona es un antiarrtmico que puede ocasionar lesin heptica, sta se puede producir tanto al inicio del tratamiento como
despus de varios meses. El cuadro clnico que produce es semejante al de cualquier hepatitis viral (astenia, anorexia, etc.). Es
caracterstica que la anatoma patolgica presente cuerpos lisosmicos lamelares de fosfolpidos al microscopio electrnico.

PREGUNTA N 16

RESPUESTA: C

El sndrome de Reye es una entidad que aparece en la edad peditrica que se caracteriza por la asociacin de encefalopata aguda,
rpidamente progresiva y esteatosis visceral de predominio heptico. La etiopatogenia no es clara pero se ha relacionado con una
infeccin viral previa y el consumo de cido acetilsaliclico.

Examen Simulacro :: Ciclo de Revisin en Medicina :: Sbado 4 de Agosto del 2012 :: www.estudiosmyc.com
PREGUNTA N 17

RESPUESTA: C

La ecografa escrotal es la ms til de las pruebas mencionadas para determinar si el testculo est intacto o no. Si presente una
rotura, precisar exploracin quirrgica y reparacin de la lesin. En el caso contrario, podr tratarse de forma conservadora. Si la
ecografa es normal, las posibilidades que se trate de una rotura sern muy escasas.

PREGUNTA N 18

RESPUESTA: A

El linfoma es el tumor testicular ms comn en este grupo de edad, mientras que los tumores de clulas germinales no
seminomatosos son poco frecuentes. Los linfomas testiculares habitualmente son de clulas tipo B. La leucemia linftica crnica es
una causa poco comn de masa testicular.

PREGUNTA N 19

RESPUESTA: C

Los urocultivos de aproximadamente el 15% de las mujeres con bacteriuria revelarn la presencia de bacterias uropatgenas en
concentraciones de 1.000 colonias inferior por ml. La historia clnica y los hallazgos bacteriolgicos indican una infeccin significativa
que debera tratarse, ms que una contaminacin de la orina.

PREGUNTA N 20

RESPUESTA: C

El paciente tiene una clara progresin de la enfermedad despus de un tratamiento adecuado. La radioterapia localizada sera la forma
ms efectiva para reducir su dolor seo. Debera considerarse la posibilidad de aplicar otras medidas una vez paliado el dolor. No hay
razn alguna para utilizar un tratamiento con anlogos de la LH-RH despus de la orquiectoma y tras haber fracasado el tratamiento
hormonal.

PREGUNTA N 21

RESPUESTA: E

Gracias al uso cada vez ms frecuente del cateterismo arterial umbilical, se ha podido documentar bien la trombosis de la arteria renal
como causa significativa de hipertensin infantil que a menudo provoca insuficiencia cardaca congestiva. Este diagnstico debe
sospecharse en todo nio con hematuria inexplicada, hipertensin o insuficiencia cardaca congestiva.

PREGUNTA N 22

RESPUESTA: D

La necesidad de tratamiento quirrgico en estos pacientes viene dictada principalmente por la presencia de infeccin y fiebre. De no
ser as, los clculos pequeos de 3 mm. se deja que se eliminen espontneamente.

PREGUNTA N 23

RESPUESTA: D

Los procedimientos de by-pass intestinal pueden producir esteatorrea. La esteatorrea provoca precipitacin de calcio de la dieta en la
grasa intraluminal (saponificacin). Esta saponificacin reduce el nivel de calcio inico intraluminal a valores ms bajos de los
normales. Normalmente el calcio inico en la luz intestinal precipita con el oxalato de la dieta. El dficit de calcio inico originado por la
saponificacin se traduce en un aumento de la absorcin de oxalato de la dieta. Como consecuencia de ello, el paciente desarrolla
hiperoxalemia, hiperoxaluria y mayor frecuencia de litognesis de oxalato clcico.

PREGUNTA N 24

RESPUESTA: A

Al igual que en los pacientes con tumor de Wilms, pero sin afectacin de la cava, el factor pronstico ms importante para la
supervivencia es el patrn histolgico. El ndice de supervivencia a los tres aos con histologa favorable era del 86%, frente a un 35%
con histologa desfavorable (es decir, variables anaplsicas).

PREGUNTA N 25

RESPUESTA: A

Existe una correlacin del 100% entre la presencia del reflejo cremastrico ipsilateral y la ausencia de torsin testicular. El reflejo
cremastrico es un reflejo de la piel superficial que consiste en la contraccin del msculo cremster ipsilateral y la elevacin del
testculo ipsilateral al golpear la piel de la cara interna del muslo. La ausencia de dicho reflejo, aunque no es diagnstico de torsin
testicular, aumenta las sospechas de torsin testicular e indica la necesidad de proseguir con ms exploraciones complementarias.

PREGUNTA N 26

RESPUESTA: C

El propranolol aumenta la presin uretral por su efecto bloqueante betaadrenrgico. Sin embargo, puede tambin provocar
broncoconstriccin debido al bloqueo de los receptores bronquiales beta-adrenrgicos. Por ello, este frmaco est contraindicado en
pacientes que padecen asma o enfermedad pulmonar obstructiva.

PREGUNTA N 27

RESPUESTA: A

La nica forma de aumentar la supervivencia en el cncer de prstata es mediante la ciruga radical ante tumores localizados. En
principio se considera que en una ciruga de cncer de prstata, para aumentar la supervivencia el paciente debe de tener menos de
70 aos. El resto de los tratamientos slo consiguen un efecto paliativo de la enfermedad mejorando la sintomatologa, pero no
consiguen erradicar la enfermedad.

PREGUNTA N 28

RESPUESTA: A

Ante un tumor de esa extensin y con afectacin de la muscular vesical, el nico tratamiento con intencin curativa es la cistectoma
radical, y derivacin urinaria con interposicin de intestino. La cistectoma parcial debido al tamao del tumor no sera viable. La
quimioterapia es un tratamiento paliativo y no curativo en el cncer vesical.

Examen Simulacro :: Ciclo de Revisin en Medicina :: Sbado 4 de Agosto del 2012 :: www.estudiosmyc.com
PREGUNTA N 29

RESPUESTA: C

El inicio insidioso sin poder precisar el momento, su curso progresivo y el fallo de memoria reciente como sntoma de presentancin es
ms caracterstico de la enfermedad de Alzheimer.

PREGUNTA N 30

RESPUESTA: D

El cuadro clnico es caracterstico y adems se dan unos factores predisponentes como son la edad avanzada, la deprivacin
sensorial, la infeccin y el cambio ambiental. La presentacin nocturna tambin es caracterstica de este cuadro.

PREGUNTA N 31

RESPUESTA: E

Tanto los diurticos como las benzodiazepinas se asocian a cadas en el anciano. La hipotensin postural es ms frecuente en esta
poblacin y puede verse favorecida por los diurticos y el cambio brusco de posicin. Probablemente la falta de iluminacin contribuy
a la cada.

PREGUNTA N 32

RESPUESTA: E

La rabdomilisis es una complicacin frecuente en sujetos que permanecen tiempo tumbados, produce elevacin marcada de la CPK y
puede causar fracaso renal agudo. La hipotermia es un diagnstico dficil si no se piensa en ello. Este paciente ha pasado la noche en
el suelo, es invierno y padece lesin neurolgica que puede deteriorar la termorregulacin.

PREGUNTA N 33

RESPUESTA: D

Todos los dems factores contribuyen a la aparicin, pero el principal siempre es la inmovilidad con la consiguiente presin mantenida
sobre la misma regin anatmica.

PREGUNTA N 34

RESPUESTA: D

Ante un paciente con deterioro cognitivo de inicio subagudo, con alteracin del nivel de conciencia debe investigarse el hecho de un
antecedente traumtico por banal que parezca y descartar mediante TAC de crneo un hematoma subdural. Su importancia radica en
que es tratable, y si no se detecta a tiempo puede ser mortal.

PREGUNTA N 35

RESPUESTA: D

Una vez descartadas las causas transitorias de incontinencia urinaria, la inestabilidad vesical es la causa ms frecuente de
incontinencia establecida, manifestndose de forma caracterstica por urgencia-incontinencia. El diagnstico se realiza a travs del
estudio urodinmico. El tratamiento mdico se basa en la utilizacin de frmacos con accin anticolinrgica. La ciruga estara indicada
en el caso de incontinencia de estrs (debilidad del suelo plvico o alteracin esfinteriana). Obviamente las medidas paliativas del
catter vesical o el colector externo no son planteables.

PREGUNTA N 36

RESPUESTA: B

Cuando aparece la enfermedad la evolucin es rpida, asocindose con signos cerebelosos y extrapiramidales, apareciendo
mioclonas inducidas por los estmulos. Los signos extrapiramidales y las mioclonas pueden presentarse tambin en la enfermedad de
Alzheimer, aunque en fases ms avanzadas de la enfermedad. Si bien la enfermedad de Parkinson y de Huntington no presentan un
deterioro mental rpido, s que pueden presentar signos extrapiramidales. Los pacientes con hidrocefalia a presin normal presentan
apraxia para la marcha e incontinencia sin mioclonas o signos extrapiramidales.

PREGUNTA N 37

RESPUESTA: B

La dermatomiositis se asocia en ms de un 60% de los casos a una neoplasia. Habitualmente existe un paralelismo entre la extensin
de la dermatomiositis y la tumoral. El penfigoide bulloso se asocia en un porcentaje muy bajo (5-10%) a una neoplasia oculta.

PREGUNTA N 38

RESPUESTA: C

La incidencia del melanoma maligno se est incrementando rpidamente en todos los grupos de edad. El pronstico de esta lesin es
peor en el grupo de ancianos que en el grupo de los jvenes, lo cual se atribuye ms directamente al grado de invasin en el momento
del diagnstico. Si bien las otras caractersticas (localizacin, histologa, ulceracin, ausencia de regresin), tambin se asocian a un
peor pronstico, ninguna de ellas ha demostrado ser tan buen predictor de la supervivencia como el grado de invasin en el momento
de la reseccin quirrgica.

PREGUNTA N 39

RESPUESTA: E

Esta paciente padece una enfermedad periodontal, causada en mayor frecuencia por una inadecuada higiene oral. Algunos pacientes
con enfermedades sistmicas (diabetes mellitus), pueden sufrir una progresin ms rpida de la enfermedad periodontal y ser
asimismo ms severa. Algunos frmacos, como la fenitona, producen hiperplasia gingival, condicionando un mayor riesgo para la
enfermedad priodontal. El edentulismo (prdida de las piezas dentarias) es una consecuencia de la enfermedad periodontal.

PREGUNTA N 40

RESPUESTA: D

Este paciente con historia de diabetes mellitus de larga evolucin tiene muchas probabilidades de padecer una neuropata autonmica
asociada. Como resultado de la neuropata puede existir alteracin de la contractilidad vesical. La adicin de amitriptilina ha podido
comprometer an ms la contractilidad muscular, provocando una arreflexia vesical, lo cual justificara la repercusin sobre el tracto
urinario superior y los escapes involuntarios de orina debido a rebosamiento.

Examen Simulacro :: Ciclo de Revisin en Medicina :: Sbado 4 de Agosto del 2012 :: www.estudiosmyc.com
PREGUNTA N 41

RESPUESTA: C

El cuadro es el de una otitis media aguda sin supuracin, que se complica provocando tromboflebitis del seno lateral. Los datos clave
son la fiebre en picos y el dolor retroauricular. Debera hacerse diagnstico diferencial con mastoiditis mediante escner.

PREGUNTA N 42

RESPUESTA: D

El edema de labio inferior y la lengua fisurada asociada a parlisis facial recurrente definen este sndrome.

PREGUNTA N 43

RESPUESTA: B

No se trata de un vrtigo de Mnire porque la hipoacusia no es fluctuante y faltan las crisis de vrtigo. El colesteatoma no produce
hipoacusia neurosensorial. La larga evolucin del proceso excluye la teraputica de choque frente a una hipoacusia sbita. Debera
interrogarse la posibilidad de un trauma sonoro crnico, pero la positividad de este hecho en la historia clnica no excluira la
posibilidad del neurinoma del VIII par.

PREGUNTA N 44

RESPUESTA: D

El dato clave es la existencia de lesiones osteolticas en Rx.

PREGUNTA N 45

RESPUESTA: A

El plasmocitoma solitario podra producir el mismo cuadro, pero es infrecuente. La enfermedad de Woakes (poliposis nasal
deformante) es propia de la infancia.

PREGUNTA N 46

RESPUESTA: D

El cuadro es una parlisis recurrencial bilateral por crecimiento tiroideo. Estas parlisis podran intubarse, pero la maniobra puede
traumatizar la glotis y afectar al habla, adems de necesitar traqueostoma diferida para evitar estenosis subgltica por intubacin
prolongada.

PREGUNTA N 47

RESPUESTA: E

Las cuatro primeras opciones son compatibles con un cuadro infecciosoirritativo. El edema alrgico de glotis produce disnea y
ronquera, pero rara vez da fiebre o tos.

PREGUNTA N 48

RESPUESTA: A

El colesteatoma puede simular los hallazgos audiolgicos de la otosclerosis, pero la otoscopia es normal. La timpanosclerosis no
afecta al reflejo estapedial; la luxacin de cadena traduce un aumento de la compliance timpnica en la timpanometra.

PREGUNTA N 49

RESPUESTA: E

El cuadro de hipoacusia neurosensorial fluctuante con reclutamiento positivo, unido a acfeno y vrtigo perifrico, es tpico de
sndrome de Mnire. El nistagmo a la derecha traduce hiporreflexia vestibular izquierda, congruente con el resto de los datos.

PREGUNTA N 50

RESPUESTA: C

Los senos maxilares no suelen dar clnica hasta los 4 aos.

PREGUNTA N 51

RESPUESTA: B

Las angina de Ludwig es una inflamacin del suelo de la boca de origen dentario; nada tiene que ver con las amgdalas palatinas.

PREGUNTA N 52

RESPUESTA: C

El dato clave sera la existencia de epistaxis importantes, lo que en principio excluira el resto de diagnsticos; el adenocarcinoma de
etmoides es excepcional en nios.

PREGUNTA N 53

RESPUESTA: B

Ante un edema de papila unilateral en un sujeto adulto es preciso descartar rpidamente una N.O.I.A. en relacin a una arteritis de
clulas gigantes, porque, aunque poco frecuente, el pronstico sin tratamiento es fatal para ambos ojos. La VSG est claramente
elevada en la arteritis y no en otras causas de edema papilar.

PREGUNTA N 54

RESPUESTA: A

La clnica es tpica de conjuntivitis. Los folculos y la adenopata preauricular aparecen en conjuntivitis vricas y la ms frecuente es la
causada por adenovirus.

PREGUNTA N 55

RESPUESTA: A

La prdida de visin de instauracin rpida acompaada de un defecto pupilar aferente nos indica patologa del nervio ptico. En la
neuritis retrobulbar la prdida de visin no se acompaa de otros sntomas o signos, salvo la respuesta pupilar anmala. Tpicamente
ni el paciente ni el mdico ven.

PREGUNTA N 56

RESPUESTA: C

Ante un ojo rojo doloroso con pupila en midriasis arreactiva y estrechamiento de cmara anterior, el diagnstico ms probable es un
glaucoma agudo de ngulo estrecho. La PIO muy elevada nos confirma el diagnstico.

Examen Simulacro :: Ciclo de Revisin en Medicina :: Sbado 4 de Agosto del 2012 :: www.estudiosmyc.com
PREGUNTA N 57

RESPUESTA: C

En la retinitis por CMV aparecen lesiones exudativas y hemorrgicas en patrn de queso y tomate, produce disminucin de la visin si
hay afectacin macular. Se da en pacientes inmunodeprimidos, incluido el SIDA. Requiere tratamiento con Foscarnet o Ganciclovir.

PREGUNTA N 58

RESPUESTA: C

La afectacin macular provoca un sndrome macular caracterizado por metamorfopsias, micropsias, o macropsias junto con
disminucin de la agudeza visual. De las opciones dadas, la correcta es la nica con afectacin macular primariamente.

PREGUNTA N 59

RESPUESTA: B

Hasta los 10-12 aos de edad podemos revertir la ambliopa mediante oclusin del ojo con mejor visin siempre que no exista una
patologa orgnica de fondo. La ciruga sobre los msculos extraoculares se debe retrasar hasta recuperar la visin del ojo amblope.

PREGUNTA N 60

RESPUESTA: E

La catarata nuclear senil provoca disminucin de agudeza visual progresiva y miopizacin del ojo por cambios en el ndice de
refraccin del cristalino (miopa de ndice). Se caracteriza porque empiezan a quitarse las gafas de cerca para trabajos de visin
prxima.

PREGUNTA N 61

RESPUESTA: D

La obstruccin de la arteria central de la retina se caracteriza por disminucin brusca de agudeza visual, retina edematosa,
blanquecina, contrastando con el color rojizo de la mcula, que est respetada. Las enfermedades metablicas con mancha rojocereza se presentan en la infancia.

PREGUNTA N 62

RESPUESTA: D

Ante un cuadro clnico de estas caractersticas, a esta edad, el diagnstico ms probable es el de enfermedad de Coats, ya que el
resto de las patologas mencionadas se diagnostican ms precozmente, o bien presentan malformaciones signos oculares de ms
temprana aparicin.

PREGUNTA N 63

RESPUESTA: C

La hemorragia vtrea es una complicacin frecuente de la retinopata diabtica. La isquemia macular y el edema macular (causas de
disminucin de visin) no aparecen bruscamente.

PREGUNTA N 64

RESPUESTA: E

La coriorretinopata central serosa aparece frecuentemente en varones jvenes-adultos nerviosos, con disminucin de agudeza visual
variable, sndrome mcula (metamorfopsias) y afecta mcula. Constituye un desprendimiento localizado de la neurorretina, producido
por disfuncin del EPR. El patrn angiogrfico en chimenea es poco frecuente pero tpico.

PREGUNTA N 65

RESPUESTA: D

La paciente, una mujer joven, presenta un patrn pulmonar intersticial. La ausencia de sintomatologa sistmica (fiebre, artralgias o
sndrome constitucional) hacen poco sugerente la existencia de una tuberculosis, un LES o una sarcoidosis. Es poco probable que la
sarcoidosis presente exclusivamente una afectacin parenquimatosa (estadio IIIA, afectacin exclusivamente parenquimatosa y IIIB, o
IV, fibrosis pulmonar). El sexo inicialmente nos orientara hacia LES, sarcoidosis y linfangioleiomiomatosis. Es particularmente tpico el
incremento de los volmenes pulmonares, al igual que ocurre en la granulomatosis de clulas de Langerhans (histiocitosis X). Esta
peculiaridad se explica, en ambos casos, por la formacin de mltiples quistes en la va respiratoria distal (en la linfangioleiomatosis,
por la proliferacin de las clulas musculares). El derrame pleural y el neumotrax son complicaciones frecuentes en el momento de
establecer el diagnstico.

PREGUNTA N 66

RESPUESTA: E

Se trata de un varn no fumador que presenta un patrn alveolar bilateral sin cardiomegalia. La enfermedad de Hamman-Rich se
caracteriza por un patrn intersticial, y conduce a una fibrosis pulmonar de forma rpida. La ausencia de cardiomegalia, adems de
una presin venosa yugular normal, descarta la insuficiencia cardiaca. No hay datos para sospechar una infeccin por CMV, virus que
caractersticamente afecta a individuos inmunodeprimidos. La espirometra muestra un patrn restrictivo y, dato crucial para descartar
la presencia de sangre en el espacio alveolar, la capacidad de transferencia del CO est disminuida. El carcinoma bronquioalveolar es
un tipo de adenocarcinoma (no relacin con el tabaco) con tres formas clnicas: masa nica (ms de la mitad de los casos),
multinodular o difuso (patrn alveolar por diseminacin endobronquial).

PREGUNTA N 67

RESPUESTA: C

El cuadro impresiona de infeccioso y de curso agudo. Las neumonas tpicas, como la neumoccica, presentan un patrn radiolgico
de ocupacin de espacio areo (alveolar) y las manifestaciones extrarespiratorias son raras. Aunque clnicamente es ms frecuente un
sndrome gripal, ante toda neumona atpica con factores epidemiolgicos sugerentes (contacto con ganado y/o sus productos, como
granjeros, matarifes o veterinarios) debe sospecharse la fiebre Q (Coxiella burnetii). La afectacin heptica (con elevacin moderada
de transaminasas) apoya este diagnstico. La neumona por legionella puede producir un cuadro superponible, aunque la falta de
respuesta a eritromicina hace menos probable este diagnstico. Fundamentalmente la agudeza del cuadro y la localizacin del
infiltrado sin afectacin ganglionar hacen improbable que se trate de una primoinfeccin tuberculosa. La neumonitis por
hipersensibilidad (aunque en su forma aguda puede provocar fiebre, sin tanta afectacin general) puede descartarse por la ausencia
de antecedente de exposicin.

Examen Simulacro :: Ciclo de Revisin en Medicina :: Sbado 4 de Agosto del 2012 :: www.estudiosmyc.com
PREGUNTA N 68

RESPUESTA: C

El caso sugiere la existencia de una hemorragia alveolar difusa (HAD) con afectacin renal. La existencia de siderfagos (macrfagos
cargados de hemosiderina) en esputo o una capacidad de difusin del CO (DLCO) aumentada confirmaran la existencia de una HAD,
pero no aporta informacin sobre la etiologa del proceso. Cualquiera de las tres siguientes respuestas pueden permitir establecer el
diagnstico, pero la realizacin de biopsias suele diferirse en el tiempo. Por el contrario, la determinacin de anticuerpos
antimembrana basal glomerular es crucial, ya que puede permitir el diagnstico del sndrome de Goodpasture. La positividad en los
anticuerpos anti-citoplasma de neutrfilo con patrn citoplasmtico (c-ANCA), dirigidos contra la proteinasa-3, es altamente sugerente
de una granulomatosis de Wegener.

PREGUNTA N 69

RESPUESTA: B

En el contexto de una hemorragia alveolar con afectacin renal, tanto tuberculosis como tromboembolismo pulmonar quedan
prcticamente descartadas. Por otra parte, en la granulomatosis de Churg-Strauss puede existir afectacin renal (menos frecuente que
en el Wegener), pero son muy frecuentes los antecedentes de asma y la eosinofilia perifrica. El cuadro clnico es muy sugerente de
un sndrome de Goodpasture. Aunque la granulomatosis de Wegener suele presentar otro patrn radiolgico (ndulos y/o masas
mltiples) y la afectacin de las vas areas superiores es muy frecuente, en principio no puede descartarse el diagnstico sin alguna
prueba complementaria. El ttulo de anticuerpos antimembrana basal glomerular es compatible con un sndrome de Goodpasture.
Debe iniciarse lo ms precozmente posible la plasmafresis (adems de corticoides para mejorar la situacin respiratoria).

PREGUNTA N 70

RESPUESTA: E

El tromboembolismo pulmonar (TEP) es la consecuencia ms grave de la trombosis venosa profunda (TVP). Ambas entidades son
aspectos de una misma enfermedad, la enfermedad tromboemblica venosa. En su patogenia se implican el estasis venoso
(inmovilizacin, encamamiento), la lesin vascular (ciruga de cadera) y las situaciones de hipercoagulabilidad. Clsicamente cursa
con disnea de instauracin brusca y dolor torcico. El signo clnico ms frecuente es la taquipnea. Aunque en un 90-95% el origen del
trombo se localiza en miembros inferiores, hasta en un 50% de los casos no se aprecian signos clnicos de TVP. La Rx de trax suele
ser normal (la alteracin ms frecuente es la aparicin de atelectasia o derrame -en un 10% de los casos, e indica infarto pulmonar-).
El ECG ms frecuentemente identificado es normal (lo ms tpico es el patrn SIQIIITIII -que sugiere sobrecarga ventricular derecha-).
Los datos gasomtricos ms sugerentes son la hipoxemia e hipocapnia con un gradiente alveolo-arterial aumentado. La prueba
diagnstica de primera eleccin es la gammagrafa pulmonar de ventilacin-perfusin. El tratamiento consiste en anticoagulacin con
heparina (hay tendencia a iniciar simultneamente heparina y dicumarnicos y, en TEP masivo, pueden emplearse fibrinolticos).

PREGUNTA N 71

RESPUESTA: B

El sndrome de Lfgren es la forma ms frecuente de presentacin de la sarcoidosis aguda, particularmente en mujeres nrdicas e
irlandesas (la otra es el sndrome de Heerfordt-Waldenstrm). Consiste en la asociacin de eritema nodoso (paniculitis septal sin
vasculitis) y adenopatas hiliares bilaterales. Puede existir linfocitosis en el LBA (>15%) de pacientes con sarcoidosis, tuberculosis,
asbestosis y linfomas, pero el cuadro clnico es muy sugerente de sarcoidosis. El cociente CD4/CD8 alto (>3.5) junto a la presentacin
tpica son casi diagnsticos de sarcoidosis. Sin embargo, el diagnstico de certeza exige evidenciar los granulomas no caseificantes
en cualquiera de los tejidos afectos. En 2/3 de los casos de sarcoidosis existe elevacin de la enzima convertidora de la angiotensina
(ECA) e hipercalciuria (con/sin hipercalcemia). El pronstico del sndrome de Lfgren es favorable y, en principio, slo requiere
tratamiento sintomtico (AINEs).

PREGUNTA N 72

RESPUESTA: A

El diagnstico de granulomatosis de Wegener se establece por la presencia de vasculitis granulomatosa en la va area superior
(sinusitis, rinitis, otitis media), inferior (ndulos pulmonares) con una glomerulonefritis (tpicamente microhematuria con cilindros
eritrocitarios). Adems, distintos grados de vasculitis en otros tejidos (piel, articulaciones, ojos, etc). El cuadro clnico que se describe
es tpico, aunque el diagnstico requiere la confirmacin histolgica (preferiblemente en tejidos ms accesibles, como la mucosa
nasal). Sin embargo, el ttulo elevado de c-ANCA es altamente sensible (88%) y especfico (95%). El curso clnico y la biopsia
descartan la etiologa neoplsica y la tuberculosis (granulomas caseificantes con bacilos cido-alcohol resistentes). La granulomatosis
linfomatoide es en realidad un proceso linfoproliferativo (con infiltrado de clulas linfocitoides y plasmocitoides atpicas en los vasos) en
pulmn, piel, rin y SNC (realmente no es una vasculitis inflamatoria). La granulomatosis de Churg-Strauss, o angetis alrgica, afecta
preferente la va respiratoria inferior (y frecuentemente piel, riones) de pacientes asmticos. La evidencia eosinofilia (>1.000/L) en
un 80% de los casos (aunque ste no es un dato diferencial).

PREGUNTA N 73

RESPUESTA: D

Imipenem (imipenem/cilastatina) es un antibitico beta-lactmico del grupo de los carbepenems. Los carbapenems (imipenem,
meropenem) presentan caractersticas diferenciales de las penicilinas y de las cefalosporinas, que los hace idneos para el problema
de la la neumona nosocomial y, principalmente, de la multirresistencia. Aunque pueden usarse como monoterapia, es recomendable
la asociacin (con aminoglucsidos o ciprofloxacino). La monoterapia con ciprofloxacino es una mala alternativa por la aparicin
precoz de nuevas resistencias. La asociacin ceftazidima, amicamicina y vancomicina es una pauta clsica para el sndrome febril en
la neutropenia severa. La ceftazidima es una cefalosporina con actividad antipseudomona, y probablemente el ms empleado en el
caso de neumonas nosocomiales en UCIs, recuperaciones, etc.

PREGUNTA N 74

RESPUESTA: C

La presencia de alguna calcificacin en el NPS es probablemente el dato ms sugerente de benignidad. Su ausencia debe alertar al
clnico, aunque no existan otros datos sugerentes de malignidad (adulto joven, tabaquismo acumulado escaso y tamao menor de 3
cm.). En la actualidad, la segunda prueba diagnstica suele ser una TAC (aunque la rentabilidad diagnstica en este paciente es, a

Examen Simulacro :: Ciclo de Revisin en Medicina :: Sbado 4 de Agosto del 2012 :: www.estudiosmyc.com
priori, muy baja, tambin podramos solicitar citologa de esputo simultneamente). La recuperacin de Rx previas es particularmente
importante, pero no se ha de diferir la realizacin de pruebas complementarias.

PREGUNTA N 75

RESPUESTA: D

La duplicacin del volumen entre 11 das y 2 aos sugiere malignidad (30 y 490 das), aunque el crecimiento lento no lo descarta (el
carcinoide, el adenocarcinoma muy diferenciado y las metstasis pueden crecer muy lentamente). Se estima que existe duplicacin del
volumen si se evidencia el incremento del radio en un 33%. Generalmente la primera exploracin cruenta es la broncoscopia con/sin
BTB (y posteriormente la PAAF si no se obtiene un diagnstico). La rentabilidad de la BTB depende de la capacidad de acceso del
fibrobroncoscopio (limitada por el calibre del rbol bronquial) y el tamao del NPS (dimetro superior a 1 cm.). La localizacin tan
perifrica decant hacia la realizacin de la PAAF. La morbilidad de la PAAF (neumotrax, hemorragia) es muy baja y la rentabilidad
diagnstica, en manos experimentadas, alta.

PREGUNTA N 76

RESPUESTA: B

En principio el diagnstico ms probable es el adenocarcinoma. El escaso tabaquismo y la presentacin perifrica hacen menos
probables el carcinoma epidermoide y el carcinoma de clulas pequeas (que tambin se asocia al tabaquismo, pero suele
presentarse como masa central con afectacin mediastnica). Si los hallazgos de la PAAF sugieren un adenocarcinoma hay que
intentar descartar que el NPS no se trate de una metstasis de un adenocarcinoma de otro origen (en principio la falta de otros
sntomas no lo sugiere). El carcinoma bronquioalveolar es un tipo de adenocarcinoma con una presentacin clnica y radiolgica tpica
(expectoracin muy abundante y patrn alveolar, aunque es ms frecuente la presentacin como ndulo o masa nica - 60% de las
ocasiones-). En caso de un resultado de la PAAF no concluyente (presenta una rentabilidad diagnstica en lesiones malignas del 8090%), debe realizarse una biopsia del NPS mediante toracotoma (microtoracotoma, y si no es posible, toracotoma abierta) y biopsia
intraoperatoria para decidir la cuanta de la reseccin pulmonar (tumor, segmento, lbulo).

PREGUNTA N 77

RESPUESTA: B

En una gestente con antecedentes de infertilidad y 36 aos de edad est indicado el diagnstico precoz de anomalas congnitas
mediante pruebas invasivas, como pueden ser la amniocentesis o la biopsia del vellosidades corinicas. Sin embargo, en esta
paciente es prioritario descartar una complicacin de la gestacin por el dolor clico hipogstrico que refiere. Dado el antecedente del
factor tubrico, hemos de descartar un embarazo ectpico. La primera prueba necesaria para ello es la ecografa, mediante la cual
visualizaremos la vescula embrionaria dentro o fuera del tero.

PREGUNTA N 78

RESPUESTA: A Antes de realizar cualquier tipo de prueba invasiva por un triple screening anmalo
como el que se presenta en este caso clnico es necesario comprobar que efectivamente la gestacin corresponde al tiempo de
evolucin que le suponemos. Es decir, si no hay una ecografa previa que nos d la edad gestacional, hay que realizarla. Si entonces
se confirman los datos de que disponemos, est indicada una amniocentesis ante la probabilidad de una comosomopata.
PREGUNTA N 79

RESPUESTA: E

Esta paciente presenta una pielonefritis, complicacin mdica bastante frecuente en la gestacin. El tratamiento es antibitico
endovenoso, hasta conseguir un mnimo de 48 horas sin fiebre. Luego, dependiendo del estado general de la paciente puede concluir
el tratamiento oral en su domicilio. El primer diagnstico diferencial es con el clico nefrtico, en el que no se presenta fiebre ni signos
analticos de infeccin.

RESPUESTA: D El test de OSullivan es un mtodo de screening que se aplica a todas las embarazadas,
y es aproximativo, pero no diagnstico de la diabetes gestacional. Una vez obtenido un resultado patolgico, es necesario confirmar la
enfermedad mediante una curva de glucemia (prueba de tolerancia oral a la glucosa, basal y horaria en tres horas), que si es excesiva
en al menos dos resultados diagnostica la diabetes.
PREGUNTA N 80

PREGUNTA N 81

RESPUESTA: C

Por la clnica que se define en el enunciado, podramos incluir este cuadro entre las metrorragias del primer trimestre. Para realizar el
diagnstico diferencial entre las causas ms habituales de metrorragia, debemos tener en cuenta que la enfermedad trofoblstica
produce sangrado indoloro con tero mayor que la edad gestacional; el embarazo ectpico produce una hemorragia interna con
escaso sangrado al exterior, y habitualmente el crvix est cerrado. En este caso el diagnstico es aborto, por lo que se debe realizar
legrado quirrgico para su tratamiento.

PREGUNTA N 82

RESPUESTA: D

La causa ms frecuente de metrorragias en el tercer trimestre es la placenta previa. Adems, en este caso la ausencia de contraccin
uterina y las caracetersticas de la metrorragia apoyan el diagnstico. El tapn mucoso se diferencia claramente de la sangre por su
consistencia, asi como el lquido amnitico hemtico. En cuanto a los Vasa Praevia, es muy infrecuente su rotura espontnea, que
siempre se acompaa de afectacin fetal.

PREGUNTA N 83

RESPUESTA: D

En un feto cuya madre est isoinmunizada, el seguimiento supone determinaciones seriadas del test de Coombs indirecto, que cuando
presenta valores ascendentes implica afectacin fetal. El nomograma de Liley valora la presencia de bilirrubina de predominio
indirecta, que en este caso se encuentra en un escaln intermedio. La indicacin es de esperar la madurez fetal y extraer el feto; la
madurez viene marcada por la presencia de fosfatidil glicerol en lquido amnitico, hecho que se ha confirmado. Por ello el siguiente
paso es finalizar el embarazo.

Examen Simulacro :: Ciclo de Revisin en Medicina :: Sbado 4 de Agosto del 2012 :: www.estudiosmyc.com
PREGUNTA N 84

RESPUESTA: C

El cuadro clnico que se describe se corresponde con una corioamnionitis. El feto est afectado (perfil biofsico = 9), por lo que se debe
finalizar la gestacin lo antes posible, adems de realizar tratamiento antibitico endovenoso a la madre en el parto y postparto. El
resto de las opciones se invalidan al no contemplar esta posiblidad.

PREGUNTA N 85

RESPUESTA: B

Entre el nacimiento de un primer y segundo gemelo deben transcurrir entre 5 y 20 minutos. Por encima de este tiempo debe realizarse
la instrumentacin indicada segn la posicin y el plano que alcanza la ceflica fetal. En este caso podemos finalizar el parto va
vaginal ya que se ha llegado al tercer plano de Hodge, y el frceps es el instrumento ms indicado dada su capacidad rotadora. La
maniobra de Kristeller est proscrita dadas las complicaciones maternas y fetales que puede acarrear.

PREGUNTA N 86

RESPUESTA: D

En el caso de esta paciente debemos prevenir la aparicin de una eclampsia, para lo que el nico medicamento til de esta lista es el
sulfato de magnesio. El resto son hipotensores que se utilizan principalmente para disminuir los valores de la tensin arterial diastlica
en las crisis hipertensivas.

PREGUNTA N 87

RESPUESTA: C

En la primiparidad aosa y la toxemia materna es ms frecuente el crecimiento intrauterino retardado de tipo II o extrnseco
disarmnico (neonatos distrficos), en el que la cabeza es mucho mayor que el abdomen por la redistribucin del flujo hacia los tejidos
nobles. En este tipo de CIR se recomienda la extraccin del feto tan pronto como sea maduro, pues parece ser que se desarrolla
mejor fuera de la madre; la placenta est muy deteriorada y no es capaz de suministrar el aporte necesario de oxgeno. El realizar una
cesrea inmediata independientemente del estado fetal sucede nicamente cuando est en peligro la vida de la madre. En el caso que
planteamos se aconseja la induccin del parto y su finalizacin va vaginal de ser posible.

PREGUNTA N 88

RESPUESTA: E

El desarrollo del cuadro con la aparicin de disnea y hemorragia incoercible orienta a una embolia de lquido amnitico, complicada
con una coagulacin intravascualar diseminada (CID). El pronstico es nefasto, prcticamente con un 100% de mortalidad, y se debe
al paso de lquido amnitico al torrente circulatorio materno, lo que desencadena la cascada de la coagulacin intravascular y una
embolia pulmonar casi simultneamente. El diagnstico diferencial con las otras situaciones descritas se basa en la sintomatologa
pulmonar, si bien sta puede ser minusvalorada o incluso ignorada.

PREGUNTA N 89

RESPUESTA: B

El cuadro clnico ms probable en este paciente sera una endocarditis y artritis sptica de rodilla. El germen ms frecuentemente
implicado es un estafilococo aureus, por lo que la cobertura ideal sera vancomicina y tobramicina (la cual es sinrgica con la
vancomicina).

PREGUNTA N 90

RESPUESTA: E

Muy probablemente no hallemos ante una neumona por Mycoplasma. Una de las complicaciones relativamente frecuentes es la
miringitis ampollosa. El tratamiento de eleccin es la eritromicina.

PREGUNTA N 91

RESPUESTA: C

El cuadro clnico corresponde a un shock sptico en el que es frecuente encontrar leucopenia, sobre todo si est originado por
grmenes gramnegativos y trombopenia, as como consumo de factores de coagulacin. La hiperglucemia se produce por la
descompensacin infecciosa y estrs. Probablemente est originado por una pseudomona resistente a la ceftacidima.

PREGUNTA N 92

RESPUESTA: D

La descripcin de la lcera hace pensar en un chancro duro de origen sifiltico. El chancro aparece en la primoinfeccin lutica y
posteriormente desaparece. La prueba de eleccin sera una serologa sifiltica.

PREGUNTA N 93

RESPUESTA: A

La rotura espontnea de bazo es una complicacin infrecuente pero caracterstica de la mononucleosis infecciosa. Cursa con un
cuadro de abdomen agudo y shock hipovolmico por sangrado.

PREGUNTA N 94

RESPUESTA: C

La primera causa de bronquiolitis en un lactante es el virus sincitial respiratorio. El tratamiento es de soporte con oxgeno y humedad.
En caso de patologa de base importante, se debe iniciar tratamiento con ribavirina en aerosol.

PREGUNTA N 95

RESPUESTA: C

Una vez descartado proceso vascular trombtico, de forma cierta con la flebografa, el diagnstico diferencial ms probable sera
celulitis del miembro. En la mayora de las ocasiones est producida por estafilococo, por lo que sera conveniente la cobertura
emprica con Oxacilina y ver evolucin.

Examen Simulacro :: Ciclo de Revisin en Medicina :: Sbado 4 de Agosto del 2012 :: www.estudiosmyc.com
PREGUNTA N 96

RESPUESTA: A

Los abscesos en pacientes ADVP deben ser drenados cuando tienen las condiciones para ello. Esto es, cuando comienzan a fluctuar.
El tratamiento antibitico es inefectivo si no se acompaa de drenaje.

PREGUNTA N 97

RESPUESTA: D

El cuadro clnico ms probable es el de clera por la ausencia de fiebre, dolor abdominal y la gran prdida de lquidos. Estas
caractersticas no se dan en el resto de las entidades.

PREGUNTA N 98

RESPUESTA: E

El aspergiloma es una forma clnica de la aspergilosis. Esta enfermedad sistmica fngica suele aparecer en individuos
inmunodeprimidos que, adems, son tratados con antibiticos de amplio espectro o tratamiento antifngico al cual son resistentes.

PREGUNTA N 99

RESPUESTA: D

El cuadro clnico parece corresponder a la evolucin natural de una tos ferina. El tratamiento de eleccin es la eritromicina.

PREGUNTA N 100

RESPUESTA: A

A falta de confirmar el diagnstico mediante un medulograma, el cuadro clnico parece corresponder a una Leishmaniasis visceral o
Kala-azar, la cual cursa con importante esplenomegalia con pancitopenia secundaria. Caractersticamente presenta fiebre con dos
picos diarios. El diagnstico se hace mediante la visualizacin directa de las leishmanias en la mdula sea. Se pueden utilizar
adicionalmente medios de cultivo y serologa especfica.

También podría gustarte